Presented by 15 Batch 1 Fhcs - Eusl
Presented by 15 Batch 1 Fhcs - Eusl
Presented by 15 Batch 1 Fhcs - Eusl
05) Enzymes,
a) Becomes more resistant to heat denaturation in presence of substrate
b) Active site has functional group of amino acid residues
c) Activity is measured during initial velocity of the reactions
d) Increase the rate of reaction by increasing kinetic energy of reactants
e) Bind with substrate by multiple weak attractions
SBR
08) A fatty acid is not synthesized in human body & only get from diet
a) Palmitic acid
b) Oleic acid
c) Linoleic acid
d) Stearic acid
e) Valeric acid
SEQ
1. Briefly describe on
1.1. Storage of polysaccharide [25 marks]
1.2. Secondary structure of protein [25 marks]
1.3. Protein synthesis [25 marks]
1.4. Functions of prostaglandins [25 marks]
Answers
MCq
1. A T
BT
CF
D
ET
Presented by 15th Batch 4 FHCS | EUSL
2.A.T
BF
CF
DF
EF
3.A T
BT
C
DT
ET
4.A T
BT
CT
DF
EF
5.A
BT
CT
DT
EF
SBR
6. d
7. D
8.C
9.B
10.B
1.Storage polysaccharide
Starch
• It is one of the most Important dietary sources For human beings and is Also one of the main Storage
polysaccharide of
Plants.
• We can find high content Of starch in cereals, roots And also in some other
Vegetable
Secondary
• Structure that arises as a result of interactions between backbone groups that are close to one
another in protein
Alpha helix
residues ahead
• Destabilize α helix
R groups)
B pleated
Chain – H bonds
3 .Protein synthesis
Converting the coded message in the mRNA into the amino acid sequence of the protein.
Presented by 15th Batch 8 FHCS | EUSL
Translation requires,
• mRNA
• tRNA
• Ribosomes
Peroxisomes.
➢ RER bound ribosomes – synthesize secretory proteins, SER, Golgi, plasma & lysosomes
• AAs
• Energy
4. Prostaglandins
Functions of Prostaglandins
Cellular signaling
14) Enzyme having slightly different molecular structure but perform identical activates are,
a) Apo enzyme
b) Co enzyme
c) Holo-enzyme
d) Iso enzyme
e) Pro enzyme
01.
1.1. Briefly describe the biochemical functions of carbohydrates. (25 marks)
1.2. Outline the stages of cellular respiration. (25 marks)
1.3. List the different level of structural organization of a protein. (05 marks)
1.4. Describe the two of the above mentioned levels. (45 marks)
02.
2.1. A patient with myocardial infraction was admitted to hospital. Routine clinical manifestations including ECG
were done and serum biochemical parameters were tested on the second day from the onset of infraction. Briefly
explain, what are the biochemical parameters would be elevated on the above condition? (20 marks)
2.2. Explain the biochemical basis of the followings;
a. Ethanol is used as antidote in methanol poisoning (25 marks)
b. Dietary lipids on atherosclerosis (25 marks)
2.3. Describe the double helix structure of DNA. (30 marks)
1.
A
B True
C True Pentose sugar group in nucleotides .(Ribose in NADH)
D True Glycolytic pathway of fructose bypass the major rate limiting step of the glycolysis PFK1
E True Galactose is a component of ABO antigen
2 A True
Amino acids with charge R groups not compatible with alpha helix- glutamate, aspartate , histidine , lysine , arginine
B False
C True
D True Proline is a helix breaker
E True Amino acids with bulky R groups not compatible with alpha helix.
tryptophan , valine or isoleucine
3 A False Careful denaturation can lead to renaturation
B False Decrease the water solubility , increase the viscosity , surface tension reduced and
can be easily digested
C False Occur due to physical and chemical factors
D True
E False Denaturation loss the secondary , tertiary & quaternary structure. But primary
structure cannot damaged . To cleave the peptide bonds of primary structure
11th Batch – HB 1 – EME
prolonged exposure to a high temperature with a strong acid to a long time
period needed.
04 A True Cell recognition (galactose found in ABO antigens on cell membrane )
B True
C True Cell membrane
Presented by 15th Batch 14 FHCS | EUSL
D False
E True
05 A False Bind to active site of the enzyme
B False
C True
D True
E False Change the shape of the active site.
06 A True
B True De – novo synthesis of cholesterol mainly occur in liver & other extra hepatic
tissue cells also can cholesterol synthesis.
C False Major sterol in animal
D False Amphipathic in nature
E True Precursor of bile acids, bile salts , Vitamin D & steroid hormones.
07
A False Mainly found in fish oil
11th Batch – HB 1 – EME
B False Decrease the heart attack risk
C
D True
E
08
A False m-RNA
B True Dual specificity refers to the fact that each t RNA has its particular anticodon,
which corresponds to the specific amino acid to which it can be bonded.
C True Multiple Ori regions in eukaryotes, A,T rich regions. Prokaryotes have a single
ori region
D True Introns and exons are nucleotide sequences within a gene. Introns are removed
by RNA splicing as RNA matures, meaning that they are not expressed in the
final messenger RNA (mRNA) product, while exons go on to be covalently
bonded to one another in order to create mature m-RNA. (Intrones are bad exons
are good )
E False Held together by hydrogen bonds which are between complementary bases.
09
A True
B True
C True
D Feedback regulation.
E True
11th Batch – HB 1 – EME
10 A True
B True Reduce ATP production and denaturation of body enzymes lead to death
C False Ubiquinone = Coenzyme Q --- all the electron carriers in ETC are proteins
except Coenzyme Q (coenzyme q is a liphophilic substance)
D True
E True
11 B glyceride
12 A Basic side chain amino acids--- lysine, arginine & histidine
13 C Polymerase chain reaction –test tube method for amplifying specific DNA
sequence.
14 D
SEQ
Proteins.
Connective tissues.
Prevent constipation.
•Oxidation of GLUCOSE
Cellular Respiration (4-stages)
• Glycolysis
Glycolysis
♠ RBC & nervous tissues derive its energy mainly from glycolysis
♠ Unique pathway
- Can utilize O2
If available (aerobic)
(anaerobic)
Presented by 15th Batch 17 FHCS | EUSL
Krebs cycle
Each turn of the Krebs Cycle also produces 3NADH,
1FADH2
, and 2CO2
, 4CO2
, and 2ATP
Anaplerosis is the act of replenishing TCA cycle intermediates that
Cataplerotic reactions).
Four reactions are classed as anaplerotic, although the production of oxaloacetate from
Pyruvate is probably the most important physiologically. The anaplerotic reactions are:
Pyruvate oxidation
ETC
Energy rich molecules (Glucose), are metabolized by a series of
Electron carriers
5 separate protein cplx (I—V)
Except CoQ.
Cplx I – NADH dehydrogenase
– Succinate dehydrogenase
– Cyt c reductase
– Cyt c oxidase
- ATP synthase
2) Secondary
• Structure that arises as a result of interactions between backbone groups that are close to one
Another in protein
Another in protein
Alpha helix
Residues ahead
Disrupt α helix
• Destabilize α helix
R groups)
B pleated
Tertiary
➢ aqueous environment
Such as
✓ H bonds
✓ Hydrophobic interactions
✓ Ionic interactions
✓ Disulphide linkag
eMyoglobin
Chain)
Interactions.
Lysozyme
• In functional molecule
• 2 types
a) Monomeric
b) Oligomeric
➢ H bonds
➢ Ionic linkage
➢ Hydrophobic interactions
Eg: - Hb
✓ α2 β2 –tetramer
• Sub units may function independently of each other or may work cooperatively as in Hb
Proteins Simple
Globin + haem)
2.1 MI parameters
O A=T & G= C
O A+T/G+C=k
O bases are always written in sequence from the 5’-end to the 3’-end.
4) Regarding proteins,
a. Motif is a secondary structure of protein
b. Proline is always in β bend
c. Primary structure is stabilized by H bonds
d. Tertiary structure has one or more polypeptide chain
e. β sheet exist only in anti-parallel form
13) Recently diagnosed hypertensive patient has been prescribed an ACE (Angiotensin converting enzyme inhibitor)
which is known to act by low Vmax. What is the possible mechanism to inhibit drug?
a. Cofactor
b. Competitive inhibitors
c. Non-competitive inhibitor
d. Prosthetic group
e. Suicide inhibitor
15)The enzyme that does not creates an electron carrier that produce 3 ATP with electron transport chain,
a. Gleceraldehyde-3 phosphate
b. Glycerol 3 phosphate dehydrogenase
c. Isocitrate dehydrogenase
d. Malate dehydrogenase
e. Pyruvate dehydrogenase
1.
1.1.
1.1.1. Briefly describe the salient features of an 𝛼 – helix. (25 marks)
1.1.2. Name two modified amino acids found in proteins and state their biological function.
(20 marks)
1.2.
1.2.1. Explain how the types of RNA molecules are structurally modified to do the specific functions. (30
marks)
Presented by 15th Batch 27 FHCS | EUSL
1.2.2. List five uses of molecular diagnostic techniques in Medicine with examples. (25 marks)
2.
2.1. Explain the biochemical basis of the followings;
2.1.1. Statin drugs are used to treat hyperlipidemic conditions. (25 marks)
2.1.2. Cyanide poisoning causes death. (25 marks)
2.2. Outline the functions of
2.2.1. Glycoproteins (25 marks)
2.2.2. Eicosanoids (25 marks)
Answers
1A T
BT
CT
DF
ET
2A T
BT
CT
DF
EF
3A T
BT
CT
DF
E
4A T
BF
CF
D
EF
5A F(ANTI ATHEROGENIC)
BT
CT
D
ET
6A T
BF
CT
DT
ET
7AT
Presented by 15th Batch 28 FHCS | EUSL
BT
CT
D
ET
8A T
BT
CT
DT
E
9A F PANTATHONIC ACID
BT
CT
D
EF
10A T
BF
CT
DT
EF
SBR
11. b
12.A
13.B
14.C
15. A ALL OTHERS USE NAD+
2) Glycogen
A. Can generate energy even in the absence of oxygen
B. In the muscle involves in the maintenance of blood glucose level
C. Is a heteropolysaccharide
D. Is mobilized slowly
E. Is stored in skeletal muscle and liver
4) Peptide bond
A. Charged polar
B. Has partial double bond character
C. Has trans configuration
D. Broken by heat
E. Is stable
5) Hyaluronic acid
A. Contributes tensile strength to tendons
B. Gives consistency to vitreous humour of the eye
C. Has sulphate group
D. Is a lubricant of the synovial fluid of joints
E. Is a potent anticoagulant
8) Regarding lipids
A. Provide 7Kcal/g calories
B. Insoluble in polar solutions
C. Naturally amphipathic
D. Precursor of water soluble vitamins
E.
9) Regarding glycolysis
A. ATP consumed in the step catalysed by the pyruvate kinase
B. High level of citrate inhibit glycolysis
C. It occurs only in the cytosol of the cell
D. Lactate is an intermediate
12) Pyruvate dehydrogenase requires oxidized (NAD+ ) for enzymatic activity, In the reaction Pyruvate oxidized to
acetyl CoA as NAD+ reduced to NADH2 is functioning as,
A. Holoenzyme
B. Cofactor
C. Coenzyme
D. Apoenzyme
E. Isoenzyme
14) which of the following is not diagnostic tool for acute MI?
A. alanine aminotransferase
B. Aspartate aminotransferase
C. Creatine phosphokinase
D. Lactate dehydrogenase
E. Troponin I
Answers
1
AF
BT
CF
DT
ET
2
AT
BF
CF
DF
ET
3
AF
BT
CF
DT
ET
4
AF
BT
CT
DF
ET
5
AT
BT
CF
DT
EF
6
A F LYCOPENE
BT
CT
DT
ET
7
Presented by 15th Batch 32 FHCS | EUSL
AT
B F UNCHANGED
CF
DT
E
8
AT
BT
CT
DF
E
9
AF
BT
CT
DT
EF
10
AF
B T
CT
DF
E
SBR
11 A
12 C LIPPINCOT PG 110
13 B
14 E
15 A ALT USED FIR LIVER DISEASE DAIGNOSIS
1.3.
Replication Process
• Replication occurs in 3 stages,
1. Initiation
2. Elongation
3. Termination
Initiation
• strands of the parental double helical DNA must be separated (“melt”) because the polymerases use only ssDNA
as a template
• begins at a single, unique nucleotide sequence—a site called the origin of replication (oriC).
• It is a highly conserved A:T rich repeat sequence that facilitate melting
• Two strands open forming Replication Forks (Y-shaped region)
• In eukaryotes, multiple replication sites – to rapidly replicate the great length of the eukaryotic DNA
1. DnaA (Initiation factor) - binds to ‘oriC’ and induces melting of DNA strands.
2. DnaB (Principle Helicase in E-coli) - binds to ssDNA and begins further unwinding of DNA strands – requires ATP.
3. Single-stranded DNA-binding (SSB) proteins (not enzymes) –
• binds and stabilize ssDNA.
• shift the equilibrium between dsDNA and ssDNA in the direction of ssDNA
• protect the DNA from nucleases that degrade ssDNA
4. Primase - binds and synthesizes RNA primer.
5. Topoisomerases - relieve topological strain.
6. DNA pol III - binds and begins to add dNTPs – deoxynucleotide triphosphates, to the primer.
• As the 2 DNA strands open at the origin, Replication Bubbles form
1. Prokaryotes (bacteria) have a single bubble
2. Eukaryotes have MANY bubbles
Elongation
Presented by 15th Batch 35 FHCS | EUSL
• Mainly done by DNA polymerase - III,
❖ Reads the template from 3´ to 5´ end.
❖ Synthesize new DNA strand in 5´ to 3´ end. (Polymerase activity)
❖ Has proofreading activity. (3´ to 5´ exonuclease activity)
❖ Synthesizes DNA from both strands simultaneously
❖ High processivity / high fidelity – remains bound to the template strand as it moves along, and does not diffuse
away and then rebind
❖ Processivity is the result of - β subunit forming a ring that encircles and moves along the template strand of the
DNA, serving as a sliding DNA clamp
❖ High rate of synthesis
❖ Two DNA strands grow in opposite directions.
• Helicase continues to unwind the DNA.
• SSB proteins stabilize the SS-DNA.
• Topoisomerase relieves the strain.
• Replication of one strand is discontinuous, because DNA polymerase activity is in 5’ to 3’ direction only
Leading strand
• Continuous synthesis
• Chain elongation in the same direction as replication fork movement.
Lagging strand
• Synthesized in the opposite direction to the fork movement.
• Limits of DNA polymerase III - can only build onto 3 end of an existing DNA strand
• Discontinuous synthesis.
• Synthesized short fragments – Okazakifragments.
• DNA bases arrive with their own energy source for bonding
• Pyrophosphate (PPi) is released when each new deoxynucleoside monophosphate is added to the growing chain
• Hydrolysis of PPi to 2Pi break down 2 high-energy bonds release the energy
DNA polymerase – I,
• 5´ to 3´ exonuclease activity – hydrolytically remove the RNA primer in the 5’→3’ direction
• 5´ to 3´ polymerase activity – extends strand up to adjacent Okazaki fragment and replaces RNA primer with DNA,
because new lagging strand consists with RNA-DNA unjoined pieces of RNA-DNA combinations.
• Forms the final phosphodiester bond between the 5’-phosphate group on the DNA chain synthesized by DNA
polymerase III and the 3’- OH group on the chain made by DNA polymerase1.
• ATP dependent.
Termination
• Events that occur at the terminus that result in separation of daughter stands.
2.2.1
–chylomicrons-
Skeletal & Cardiac). Formation fluctuates with load of TG absorbed. Give turbidity after a fatty meal, rapidly
catabolized and cleaved from circulation (1-2 hrs)
VLDL-
LDL-Forward transport of cholesterol produced in plasma during intra vascular metabolism of VLDL.
2.2.
• Transcription
• Translation
• Targeted proteolysis
1. Via effector molecules/Allosteric regulation allosteric enzymes – The enzymes that are regulated by effector
molecules
• Binding of an effector molecule to the enzyme creates conformational changes in the enzyme resulting in altered
activity
• Cause positive or negative influence on the enzyme Positive effectors negative effectors
• When inhibitors bind – decrease the binding affinity of the enzyme to substrate
• regulate the enzyme activity by binding at a site other than the active site of the Enzyme
• Binding of an effector molecule to the enzyme creates conformational changes in the enzyme resulting in altered
activity
2. Via phosphorylation
• Phosphorylation of enzymes by specific protein kinases is a widespread mechanism for regulation of enzyme
activity
• It allows the enzyme’s tertiary structure to rearrange so that the final active state can form.
Presented by 15th Batch 38 FHCS | EUSL
2.3.
As the electrons are transferred, some electron energy is lost with each transfer.
This energy is used to pump protons (H+ ) across the membrane from the Matrix to the inner membrane space. A
proton gradient is established.
The higher negative charge in the matrix attracts the protons (H+ ) back from the
Intermembrane space to the matrix. The accumulation of protons in the intermembrane space drives protons into
the matrix via diffusion.
Most protons move back to the matrix through ATP synthase. ATP synthase is a membrane-bound enzyme that uses
the energy of the proton gradient to synthesize ATP from ADP + Pi. This is called as oxidative phosphorylation of
etc.
2.4.
However, in specific cases, uncoupling the two processes may be biologically useful.
Membrane of brown adipose tissue—provides for an alternative flow of protons back to the inner mitochondrial
matrix.
Synthetic uncouplers (e.g., 2,4-dinitrophenol) also exist, and, at high doses, are lethal.
Maintaining body temperature for hibernating animals, although these proteins may also
01. Glycogen,
A) Can generate energy even in the absence of oxygen.
B) Is a linear homopolysaccharides.
C) Is stored in anhydrous form.
D) Is stored in the endoplasmic reticulum of the cell.
E) Has α (1-4) glycosidic bonds only.
07. Cholesterol,
A) Mainly exists as cholesterol esters
B) Maintain the fluidity of plasma membrane
C) Is a precursor of peptide hormone
D) Rich in coconut
E) Is transported free in plasma
SBR
15. What is the equation that represents the cellular aerobic respiration?
A) glucose → pyruvate + 2ATP
B) 6CO2 +6O2 +energy → glucose
C) glucose + 6O2 → 6CO2 +6H2O +energy
D) glucose → 2 lactate + 2ATP
E) glucose → 2 alcohol + 2CO2 + 2ATP
1. Briefly describe on
1.1. Alpha helix structure of protein (25 marks)
1.2. Cloning (35 marks)
1.3. Enzymes in clinical diagnosis (40 marks)
6) Denaturation of protein,
A. Change primary structure
B. Decrease viscosity
C. Usually irreversible
D. Loses biological activity
E. Facilitate the digestion
9) Thymidine,
A. Is equal to the adenosine in double stranded DNA
B. Is a pyrimidine
C. Replaced by uracil in RNA
D. Made 3 hydrogen bonds with Guanine
E. Participate in hydrophobic interactions due to its methyl group
SBR
11) which is following enzyme that require NAD+ as cofactor?
A. alpha ketoglutarate dehydrogenase
B. citrate synthase
C. fumarase
D. succinate dehydrogenase
E. succinyl CoA synthetase
13) Which is the amino acid important for buffering action of protein at physiological pH,
A. Tyrosine
B. Histidine
C. Lysine
D. Arginine
E. Alanine
1.
1.1 Briefly describe the importance of homopolysaccharides with examples. (30 marks)
1.2 State the functions of cholesterol with examples. (25 marks)
1.3 Cyanide is one of the most rapidly acting poisons in human.
Briefly explain, what effect cyanide would have on ATP synthesis. (25 marks)
1.4 Briefly describe on Alpha helix structure of a protein (20 marks)
2.
2.1. Write short notes on
2.1.1. DNA replication in prokaryotes (35 marks)
2.1.2. Restriction enzyme (30 marks)
2.2.
2.2.1. Draw the Michaelis-Menton and Lineweaver-Burk plots (20 marks)
2.1.2. Explain the significance of Michaelis-Menton constants (15 marks)
answers
Presented by 15th Batch 44 FHCS | EUSL
1. A-False
B-True
C-True
D -True
E-False
2. A-False
Substrates are bound to enzymes by multiple weak attractions.
(Binding is thought to cause a conformational change in the enzyme (induced fit model) that allows a rapid
conversion of the ES to enzyme–product (EP) complex that subsequently dissociates to free enzyme and product.)
B-True
Active Site is formed from Amino acids sequences in the polypeptide chain (3-5)
C-True
D-True
Active site is a restricted region (3D) or special pocket or cleft of an enzyme molecule which binds to the
substrate
E-True
Enzyme molecules contain a special pocket or cleft called the active site which is formed by folding of the
protein.
3. A-False
Phosphotidyl inositols act as second messengers
B-True
C-True
D-True
E-(False)
4. A-True
B-False
AMP activate PDH to produce more acetyl coA for krebs cycle
C-False
Calcium (Ca2+ ) is a strong activator of PDH phosphatase, stimulating E1 activity. This is particularly important
in skeletal muscle, where Ca2+ release during contraction stimulates the PDHC and, thus, energy production.
D-True
The acetyl CoA from FA oxidation indirectly inhibits PDH [by activation of PDH kinase]
E-False
NAD is a co substrate of PDH
5. Non-polar side chains (hydrophobic) Ex: ala, val, leu, Ile , met, Phenyl, Try, Pro, Gly
Polar (hydrophilic) Uncharged side chains: Ser, Thr, Tyr, Asn, Gln, Cys
Charged Side Chains
Acidic side chains: Glu, Asp
Basic side chains: lys, arg, His
A-False
B-True
C-False
D-False
E-True
Presented by 15th Batch 46 FHCS | EUSL
6. A-False
Denaturation results in the loss of secondary, tertiary and quaternary structure of proteins.
The primary structure of a protein with peptide linkages remains intact i.e., peptide bonds are not hydrolysed.
B-False
The viscosity of denatured protein (solution) increases (while its surface tension decreases.)
C-True
Denaturation is usually irreversible.
Careful denaturation is sometimes reversible (known as renaturation).
lrreversible denaturation results in coagulation.
D-True
E-True
Denatured protein is more easily digested .This is due to increased exposure of peptide
7. -Question 2 repeated-
8.
A-True
B-True
C-True
D-True
E-False
9. The addition of a pentose sugar to a base through an N-glycosidic bond (see p. 94) produces a nucleoside. If the
sugar is ribose, a ribonucleoside is produced, and if the sugar is 2-deoxyribose, a deoxyribonucleoside is produced
Ribonucleosides of A, G, C, U are adenosine, guanosine, cytidine, uridine
Deoxyribonucleosides of A, G, C, T have the prefix deoxy- added. (Deoxyadenosine,....)
Thymidine+phosphate=Thymine
10.
A-False(in vitro)
B-False(selected DNA)
C-True
D-True
E-False
11. A
12. B
14. B
15.
1.1 Briefly describe the importance of homo polysaccharides with examples. (30 marks)
Polysaccharides (or simply glycans) consist of repeat units of monosaccharides or their derivatives, held together by
glycosidic bonds. They are linear as well as branched polymers. Polysaccharides are of two types. They are homo
polysaccharides and hetero polysaccharides.
Homopolysaccharides on hydrolysis yield only a single type of monosaccharide. Examples for homopolysaccharides
are starch, glycogen, cellulose, inulin, dextrin, mannan, galactan
Starch is the carbohydrate reserve of plants which is the most important dietary source for higher animals,
including man. It is a homopolymer composed of D-glucose units held by E-glycosidic bonds.
Glycogen is the carbohydrate reserve in animals, hence often referred to as animal starch. It is present in high
concentration in liver, followed by muscle, brain etc. Stored glycogen serves as a fuel reserve. Because glycogen can
be rapidly metabolized, can generate energy in absence of oxygen, brain depends on continuous glucose supply.
Cellulose occurs exclusively in plants and it is the most abundant organic substance in plant kingdom. It is a
predominant constituent of plant cell wall. Cellulose is totally absent in animal body. Cellulose is composed of B-D-
glucose units linked by B (1-4) glycosidic bonds. Cellulose cannot be digested by mammals including man due to lack
of the enzyme that cleaves Beta glycosidic bonds. Cellulose, though not digested, has great importance in human
nutrition. It is a major constituent of fiber, the non-digestible carbohydrate. The functions of dietary fiber include
decreasing the absorption of glucose and cholesterol from the intestine, besides increasing the bulk of feces.
Inulin is a polymer of fructose. Inulin is not utilized by the body. It is used for assessing kidney function through
measurement of glomerular filtration rate (GFR).
Cholesterol is a steroid alcohol of animal tissues. It is widely distributed in all cells of body. There are many
functions of cholesterol.
- It is precursor of synthesis of
~ steroid hormones: progesterone, estrogen, androgen, glucocorticoid, mineralocorticoids
~ bile acids and bile salts
~ vitamin D
Cyanide is a respiratory chain inhibitor which block respiration’s in the presence of either ADP or uncouplers. It
blocks the electron flow in complex IV. Cyanide ion combines tightly with Fe3+ of cytochrome oxidase. In complex
lV 2 protons pumped from matrix to intermembrane space and water is formed. Since this is is inhibited the
consequent action of ATP synthase will be interrupted, leading to inhibition of electron transport chain. Utilization
of O2 is stopped in cyanide poisoning.
The synthesis of ATP (phosphorylation) is dependent on electron transport. Hence, all the site-specific inhibitors of
ETC also inhibit ATP formation.
There are 2 dimensional secondary structures of proteins. Those are Alpha helix & Beta sheet. The most common
type is the Alpha helix
Amino acids in an Alpha helix are arranged in a right handed helical structure. The polypeptide bonds form the
back-bone and the side chains of amino acids extend outward from the central axis.
Stabilised by H bond formed between H atom attached to peptide N , & O atom attached to peptide C. Inter chain
hydrogen bond is parallel to the peptide backbone.
Each turn is formed by 3.6 residues. The distance between each amino acid residue is 1.5 Å.
Amino acids with bulky side chains less common in alpha helix. Glycine & Proline destabilizes alpha helix.
• DNA synthesis begins at specific nucleotide sequences, called replication origins, and proceeds in both directions
(i.e., bidirectional action).
a. Replication is semiconservative; that is, each new DNA double helix contains one parental (old copy) strand and
one daughter (new copy) strand.
b. DNA is exposed to replication enzymes by action of DNA gyrase (topoisomerase II); DNA is relaxed with negative
supercoils and unwound.
• Multiple proteins participate in initiation and elongation of daughter DNA strands at the replication fork.
a. Helicase separates the two strands of the relaxed DNA helix, and single strand–binding proteins attach to each
strand; the point of separation is the replication fork.
b. Helicase unwinding causes a physical strain (positive supercoiling) immediately ahead of the moving replication
fork; topoisomerase I relieves this strain by breaking one of the DNA strands and allowing it to rotate around the
unbroken strand before rejoining the broken ends.
c. DNA polymerases catalyze the addition of 5’-deoxyribonucleoside triphosphates to the free 3’-OH end of the
growing strands with a loss of pyrophosphate (PPi ); this always leaves a free 3’-hydroxy group to accept the next
addition.
(1) The parental (template) strand is always read in the 3’—5’ direction.
(2) The daughter (new) strand is always synthesized in the 5’—3’ direction.
(3) Because of the directionality of DNA synthesis, one daughter strand is synthesized continuously (i.e., leading
strand), and the other is synthesized discontinuously (i.e., lagging strand)
• Different processes are involved in the synthesis of leading and lagging strands.
a. Leading-strand synthesis, catalyzed primarily by DNA polymerase d, proceeds continuously from a single RNA
primer synthesized by DNA polymerase a.
b. Lagging-strand synthesis proceeds discontinuously from multiple RNA primers synthesized by DNA polymerase a,
forming short, 1000 base pair Okazaki fragments.
(1) The Okazaki fragments are extended by DNA polymerase , as in leading-strand elongation.
(2) Primer is removed from the fragments by a 5’—3’ exonuclease, and gaps are filled by DNA polymerase, which
also functions in repair.
Creates fragmented, lagging strand; Okazaki fragments linked together with DNA ligase
a. The 3’—5’ exonuclease activity of the DNA polymerases removes a mismatched nucleotide, and synthesis then
continues with insertion of the correct nucleotide
One of the major molecular analysis of genomic DNA is the immense size of molecules involved. The discovery of a
special group of bacterial enzymes, called restriction endonucleases (restriction enzymes), which cleave double-
stranded DNA (dsDNA) into smaller, more manageable fragments, opened the way for DNA analysis.
Each enzyme cleaves dsDNA at a specific nucleotide sequence (restriction site) and produce restriction fragments.
These enzymes are naturally found in bacteria to protect bacteria from invasion of foreign DNA.
Restriction enzymes recognizes short stretch of DNA( 4-8 bp) that contain s nucleotide sequences. These
recognition sites are called Palindromes. Palindromes exhibit twofold rational symmetry.
These enzymes are named according to the organism from which it was isolated.
Example: EcoRI, Haelll
They may produce either cohesive ends or blind ends after the cleavage.
2.2.
Michelis constant is the substrate concentration that produces half maximum velocity of enzyme.
Enzymes with low Km: have high affinity to the substrate i.e. they act at maximal velocity at low substrate
concentration
E.g. Hexokinase acts on glucose at low concentration (fasting state)
Enzymes with high Km: they have low affinity to the substrate i.e. they act at maximal velocity at high substrate
Concentration
E.g. Glucokinase enzyme acts on glucose at high concentration (fed state)
02. Match the specific inhibitor for electron transport chain complex
A. ATP synthase- Antimycin
B. Cytochrome C oxidase- cyanide
C. Cytochrome C reductase- Oligomycin
D. NADH dehydrogenase- Rotenone
E. Succinate dehydrogenase - Carbon monoxide
SBR
13. What is the Krebs’s cycle enzyme found in inter membrane space?
A. Citrate synthase
B. Malate dehydrogenase
C. Iso-citrate dehydrogenase
D. Succinate dehydrogenase
E. α- ketoglutarate dehydrogenase
14. Production and secretion of trypsin is inhibited in pancreas disease. Hydrolysis which compound?
A. Proteins
B. Lipids
C. Carbohydrate
D. Nucleic acid
E. Phospholipid
1.
1.1. Outline the mechanism of oxidative phosphorylation (25 marks)
1.2. State the function of different types of lipoproteins (15 marks)
1.3. List three (03) consequences occurred due to deficiency of disaccharidases enzymes in human (10 marks)
1.4. List the functions of any 5 enzymes or protein required for DNA replication (15 marks)
1.5. Briefly describe the beta pleated sheet structure of protein (20 marks)
1.6. Mention five (05) enzymes used as therapeutic agent (15 marks)
8TH BATCH
8TH BATCH PROPER MCQ
3) Trytophanyl - tRNA synthatase will charge a tRNA leaving the correct anti-codon with L- tryptophan, but no D-
tryptophan or any other amino acid in protein biosynthesis. Which of the following enzymatic characteristic is seen
in above example?
A) Catalytic power
B) Specificity
C) Transformation of energy
D) Regulation
E) Competitive inhibition
4) Glycosaminoglycan are,
A) Heteropolysaccharides
B) Associated with proteins
C) Branched molecule
D) Having the ability to bind large amount of water
E) Are found in connective tissue
6) Prostaglandins
A) Supports gastric suppression
B) Elevate body temperature
C) Promote cervical dilatation
D) Are involved in inflammatory response
E) Increase blood pressure
7) Regarding RNA
A) tRNA comprise 5% of total RNA
B) mRNA is the smallest RNA
C) tRNA transfers amino acid to a growing polypeptide chain
D) tRNA is the abundant RNA
E) rRNA carries genetic information from DNA
1.0
. 1.1. Outline the basic structure of DNA & list the characteristics of the WatsonCrick model (45 marks)
1.2. 1.2.1. Define secondary, tertiary & quaternary structure of proteins (15 marks)
1.2.2. With the aid of a diagram, illustrate the main types of secondary structure. (20 marks)
1.3. Explain why maltose positively answers the benedicts test but sucrose does not. (20 marks)
Answers
2)
A) T All aromatic amino acids absorb UV rays.
B) F Glycine has 2 H atoms attached to the chiral C, therefore no optical activity and no D and L isomers.
C) T Lippincott pg. 3 – fig. 1.3
D) T Isoelectric pH; the pH at which the no. of negative charges on a population of molecules is equal; no net
charge.
E) T
3)
A) T
B) T
C)
D)
E) F
4)
A) T Heteropolysaccharide made up of the repeating unit [amino sugar + acidic sugar]
Presented by 15th Batch 60 FHCS | EUSL
B) T All GAGs except hyaluronic acid are found covalently attached to protein, forming proteoglycan monomers.
C) F Unbranched
D) T Binds large amounts of water producing the gel like matrix that forms the basis of the body's ground
substance.
E) T
5)
A) T Compete with substrate molecules for the active site.
B) F Vmax - unchanged
C) T Km - increased
D) F Competitive inhibitors compete with substrate molecules foe the active site.
Allosteric inhibitors bind irreversibly or reversibly to the enzyme not at the active site, they function by altering the
shape of the active site.
E) T
6)
A) T PGE2 - Reduce secretion of HCl from stomach
B) T PGE2 – Fever response
C) T PGE2/PGF2 – induce labour
D) T PGE2/Leukotriene
E) T PGF2/Thromboxane A2 - vasoconstriction
7)
A) F rRNA – 85% tRNA – 15% mRNA – 5%
B) F tRNA
C) T
D) F rRNA
E) F mRNA
8)
A) T Okazaki fragments; short sequences of DNA nucleotides containing approx. 150-200 base pairs.
B)
C) F Done by topoisomerase.
D) T
E) T
49)Answer – A
1.
1.1.
● There are 4 bases as adenine,guanine,cytosine and thymine.
● Adenine and guanine are purines which contain 2 cycles.
● Cytosine and thymine are pyrimidines which contain only 1 cycle.
● Deoxyribo nucleotide consist of purine or pyrimidine base,phosphate group and pentose sugar.
1.2.
1.2.1.
● Secondary structures are ordered 2 dimensional structures formed due to H binding between peptide
groups which occurs between H of amino group and O of the carbonyl groups.
● These H bonds involve only the peptide backbone, not the side chains.
● Classified into 2 classes as alpha helix and beta sheet.
● Tertiary structure is a 3 dimensional folding of a protein due to interactions of amino acid side chains.
● A globular structure.
● Interactions that stabilize tertiary structure are ionic bonds,van der waals,hydrophobic interactions ,and
disulfide bonds.
● Quaternary structure is the association of 2 or more polypeptides into one functional protein.
● Sub units are held together primarily by noncovalent interactions such as H bonds , ionic bonds and
hydrophobic interactions.
1.2.2.
a)α-helix
b)β-sheets
c)β-Bends
d)Loops & Coils
e)Supersecondary structures (motifs)
1.
1.1.
●
Regular folding and twisting of the polypeptide chain by hydrogen bonds.
●3 important kind of secondary structure
Alpha – helix
Beta – pleated sheet
Beta- bend
Alpha – helix
● Is the most common
● Spiral structure
● Consist tightly packed coiled polypeptide backbone core.
● Side chains of the amino acid extend outward from the central axis.
Ex ; keratine , myoglobin
Beta – pleated sheet
● Surface appear pleated
● Contain two or more polypeptide chains or segments of polypeptide chain
● Hydrogen bonds are perpendicular to polypeptide backbone.
● Polypeptide chains arranged in parrellel and anti-parrellel .
B –bend
● Most of proline compound in b –bend.
1.2.
● Complex of lipids and specific protein.
● Consist of chylomicron
VLDL
Chylomicrone
● TAG (90%) >Cholesterol/ cholesterol ester > phospholipid
>protein
● Need APO B -48 , APO C 2, APO E
VLDL
● TAG (60%) > Cholesterol /cholesterol ester> phospholipid
> protein
LDL
● Cholesterol/ cholesterol ester > phospholipid > protein > TAG
HDL
● Protein > phospholipid > cholesterol /cholesterol esters >
TAG
● Need APO A – 1 C – 2 and E.
● Size of lipoprotein
Chylomicrone> VLDL>LDL>HDL
● Density
HDL > LDL > VLDL > chylomicrone
● Chylomicrone deliver dietary TAG to peripheral tissues VLDL deliver endogenous TAG to peripheral tissue.
● LDL deliver cholesterol to peripheral tissues on back to
liver
● Increase risk for coronary heart disease.
● HDL deliver cholesterol from peripheral tissue to liver Good type
● Decrease risk for heart disease.
1.3.
● Test tube method
● For amplifying a selected DNA sequence
● Synthesize million of copies of specific nucleotide sequence in few hours.
1.4.
● Long, un branch ,negatively charged ,heteropolysaccharide
● Composed of repeating disaccharide unit
● Acidic and amino sugar
● But keratan sulphate not contain acidic sugar, contain galactose
● Large complex
● All GAGs have sulphate group except hyaluronic acid
● Has special ability o bind large amount of water
● GAGs are associate with large amount of protein except hyaluronic acid
● Types of GAGs , 6 types
hyaluronic acid - synovial fluid , vitreous humor of eye , loose connective tissue , cartilage
Lubricant , shock absorber
heparin – present in mast cells, anticoagulant, present sulphate large amounts .
heparin sulphate – extracellular GAG, present in basement membrane.
Dermatan sulphate – Skin , blood vessel , heart valve,Need for coagulation
chondroitin sulphate – cartilage , tendon ,ligament, arota Most abundant GAG in the body
Forms proteoglycans aggregates with hyaluronic acid
keratan sulphate – cornea , loose connective tissue,most heterogeneous GAG
Absorb mechanical shock
● Functions of GAGs
Binding and packing of tissue
Connect, anchor , support the body and its organ
Transport of metabolites between capillary and tissue
Defence against infection
Repair in injury
2) Hyaluronic acid
A) Linear polymer
B) Moisturizer
C) Found in vitreous humor of eye
D) Sulphated
E) Backbone of proteoglycan
4) Regarding lipids
A) Sphingomyaline contains a phospholipids
B) Cholesterol is precursor of bile salt and vitamin D
C) Phospholipids contains nitrogenous residues
D) All fatty acids synthesized in body saturated
E) Lipid can be tested by Seliwanoff’s test
7) RNA
A) Has short half-life than DNA
B) Not contain bind with base pairs
C) Has different purine base that DNA has
D) Functions as catalytic enzymes in same form
E) Purine: pyrimidine is not similar
Presented by 15th Batch 66 FHCS | EUSL
8) Chromatin
A) Has double helix DNA
B) Has basic proteins
C) Has acidic proteins
D) Has small amount of proteins
E) Is the basic unit of nucleosome
SBR
41) Molish’s test is done to identify
A) Cryohydrates
B) Proteins
C) Phospholipids
D) Bile salts
E) Bilirubin
42) Heparin is a
A) Carbohydrate
B) Proteoglycan
C) Glycosaminoglycan
D) Lipoprotein
E) Glycolipid
43) Enzyme
A) Activity increases with temperature
B) Increases activation energy
C) Active site is formed by assembly of protein domain
D) Changes the equilibrium
E) Activity is measured during maximal enzymatic reaction
49) ADH requires NAD+ for catalytic activity. In the reaction catalyzed by ADH, an alcohol is oxidized to an aldehyde
as NAD+ is reduced to NADH and dissociates from the enzyme. The NAD+ is functioning as a (an)
A) Apo enzyme
B) coenzyme-co substrate
C) coenzyme-prosthetic group
D) Cofactor
E) Heterotropic effector
(01)
1.1 Briefly describe the α helix secondary structure of proteins (15 marks)
1.2 State the different types of inhibition (10 marks)
1.3 Describe competitive enzyme inhibition kinetics and other features (30 marks)
1.4 Give 2 example of competitive enzyme inhibition in usefulness in therapeutics (20 marks)
01.
1.1.Briefly explain how the constituent molecules of the plasma membrane contribute to fluidity. (35marks)
1.2. Differentiate between starch and glycogen on the basis of structure and function. (30 marks)
1.3. Describe how DNA is packaged in the chromosomes with necessary diagrams. (35 marks)
02
2.1
a) Outline the levels of organization of protein structures. (25 marks)
b) Describe the bonds that stabilize each level of organization. (25 marks)
2.2
a) Define isoenzymes. (10 marks)
b) Briefly describe the diagnostic significance of the following isoenzymes. (2x20=40 marks)
i) Lactate dehydrogenase (LDH)
ii) Creatine phosphokinase (CPK)
2)
A) T Long, unbranched polysaccharide
B) T Serve as lubricant and shock absorber.
C) T Synovial fluid in joints, vitreous humor of the eye, umbilical cord, loose connective tissue and
cartilage
D) F Unsulphated
E) T Lippincott pg. 160 – fig. 14.7
3)
A) T
B) T
C) T
Fluidity due to motility of lipids and proteins
It mainly depends on lipid composition and temperature. D) T
E) T
4)
A) T Phospholipids derived from alcohol sphingosine instead of glycerol.
B) T Synthesis of bile acids Lippincott pg. 225 fig. 18.9
Synthesis of vitamin D Lippincott pg. 387 fig.28.23
C) T Glycerol + 2 fatty acids + phosphate group
Also considered as derivative of phosphatic acid (phosphoric acid, N containing base and other
substituent)
D) F
E) F Seliwanoff’s test Ketose sugar test
Sudan III test Lipid test
5)
A) F Mnemonic for essential amino acids; PVT TIM HALL
B) F
C) F
D) T
E) F
6)
Presented by 15th Batch 69 FHCS | EUSL
A) F C18 – linolenic acid
B) F C18
C) T C20
D) F C18
E) T C20
7)
A) T
B) F Single stranded but has base pairing in t RNA
C) T Uracil instead of thymine
D) T Ribozyme – 28s r RNA catalyze the synthesis of peptide bonds. I
E) T
8)
A) T Chromatin DNA + histone protein
B) T Histone protein is a basic protein
C) F Proteins are basic; Arginine and Lysine
D) T
E) F Basic unit of chromatin is nucleosome
9)
A) T PCR allows scientists to amplify small, specific segments of DNA and make millions of copies of
segment.
B) F Small specific segments
C) T DNA polymerase III (Taq Polymerase)
D) T
E) F Temperature differences
10)
A) T
B) T
C) T
D) T
E) T
11)
A)T Isoenymes are enzymes that catalyze the same reaction but have different physical properties.
B)T Different physical properties.
C) F
D)T Eg; Creatine kinase (Lippincott pg.65)
E)T Lippincott pg.65 - Fig. 5.21
41) ANSWER-A
42) ANSWER-C
43) ANSWER-C
48) ANSWER – A
49) ANSWER – B
Coenzymes- cosubstrates are small organic molecules that associate transiently with an enzyme and
leave the enzyme in a changed form. Coenzyme-prosthetic groups are small organic molecules that
associate permanently with an enzyme and are returned to their original form on the enzyme. Cofactors
are metal ions. Heterotropic effectors are not substrates.
1.
1.1
● There are 2 dimensional secondary structures of proteins.
● Those are Alpha helix & Beta sheet.
● The most common type is the Alpha helix.
● Amino acids in an Alpha helix are arranged in a right handed helical structure.
● All H-bonds in the alpha helix are oriented in the same direction.
● The N-terminus will have a positive charge & the C-terminus will get a negative charge.
● Side chain groups point outwards from the helix.
● Amino acids with bulky side chains are less common in alpha helix.
● Glycine & Proline destabilizes the alpha helix.
1.2
● Irreversible
● Reversible
Competitive
Non-Competitive
1.3
Because of the similar structure of the substrate and inhibitor, the inhibitor has to compete with the
substrate to bind with the active site of the enzyme. So it is called “Competitive Inhibition”. That binding
blocks the active site available for the substrate. So there will be no Enzyme-Substrate complex formation.
No enzymatic activity. No Enzyme-Product complex formation. Km will be increased and Vmax constant.
1.4
Inhibitor is a substance that reduces/stops the enzyme reaction & thereby interferes with catalysis.
Competitive inhibitor is a structural analogue of the substrate which competes with the substrate to bind to
the active site & inhibit the reaction. In this inhibition, Vmax is not changed. But Km increased.
Methotrexate
Methotrexate is an analogue of Dihydrofolate(DHF). So it competes with DHF to bind with DHF Reductase
enzyme. Methotrexate binds to the active site of enzymes. So, Ensyme-Substrate complex formation is
reduced resulting in decreased formation of Tetrahydrofolate(THF). Conversion of DHF to THF is essential
Simvastatin is an analogue of HMG CoA. So it competes with HMG CoA to bind with the active site of
HMG CoA Reductase enzyme. As simvastatin binds with enzymes, the active site is not free for the
formation of Enzyme-Substrate complexes. Therefore the product; Mevalonate, is reduced. This reaction
is the regulatory step in cholesterol synthesis. By inhibiting this reaction, cholesterol synthesis is reduced.
So it is used in the treatment of Atherosclerosis.
2.
2.1.
Okazaki Fragments
An Okazaki fragment is a relatively short fragment of DNA. Those are created on the lagging strand during
DNA replication.
On the leading strand, DNA replication proceeds continuously along the DNA molecule as parent double
stranded DNA is unwound. But on the lagging strand, the new DNA is made in installments which are later
joined together by a DNA ligase enzyme. This is because the enzyme that synthesises the new DNA can
only work in one direction along the parent DNA molecule & the two strands are anti- parallel.
2.2.
Structural polysaccharides
Structural polysaccharides are,
Cellulose
Chitin
Gums & Pectin
GAG
Cellulose
Major component of plant cell walls.
Polymer of beta glucose units.
Linked by β-1,4 glycosidic bonds
Provides protection & support.
Presented by 15th Batch 72 FHCS | EUSL
GAG
Unbranched polysaccharides
Polymers of uronic acid & amino sugar
Heteropolysaccharides.
Amino sugars
- D-Glucosamine
- D-Galactosamine
Uronic acids
- L-Glucoronic acid
- L-Iduronic acid
GAGs attached covalently to extracellular proteins except hyaluronic
acid.
Forms proteoglycans.
GAGs can be found
- Synovial fluid of joints.
- Vitreous humour of the eye.
- Arterial walls.
- Bones & cartilages.
Major component of extracellular matrix.
Functions
1. As a cushion against mechanical shocks.
2. Act as molecular sieves.
3. Give elasticity to substances.
4. Lubricate joints both of the surface of cartilage & in synovial fluid.
Types of GAGs
1.Hyaluronic acid
Synovial joints
Loose connective tissue
2.Chondroitin sulphate
Tendons
Ligaments
3.Keratan sulphate
Cornea
Cartilage
4.Dermatan sulphate
Skin
Blood vessels
5.Heparin
Lining of arteries of liver
6. Heparan sulphate
Skin
Fibroblast
2.3
Protein synthesis has 2 steps.
1. Transcription
2. Translation
Translation takes place in the cytoplasm.
Presented by 15th Batch 73 FHCS | EUSL
It is the process of decoding the m-RNA into a polypeptide chain.
Ribosomes read m-RNA 3 bases or one codon at a time & construct the proteins.
Transcripted m-RNA enter the cytoplasm.
m-RNA transcript start codon AVG attaches to the small ribosomal sub unit.
Small sub unit attaches to large ribosomal sub unit. 05 stages
1) Activation of amino acids
2) Initiation - start codon (AVG)
3) Elongation
4) Termination – stop codon (UAG)
5) Folding & processing
1.
1.2.
starch glycogen
1.3.
● Found in the forms of chromosomes
● Associate with the tightly bound basic protein – ( histone, + charged , so they bind with – charged
DNA)
● Histones H1 , H2A , H2B , H3 , H4
● Two molecules from H1 , H2A , H2B , H3 , H4 form the structural core of individual nucleosome
“beads”
● A segment of DNA double helix is twisted (wound) around the core.
● Nucleosome are joined by linker DNA and H1
● Nucleosomes are tightly packed to form a nucleofilament ( polynucleosomes )- a coil 30 nm fiber.
● The fibres are organised into loops and anchored to scaffold proteins.
2.0
2.1
a)Four organizational levels of protein structure
Primary
Primary structure
Secondary structure
Regular arrangements of amino acids that are located near to each other in the linear
sequence.
α-helix
β-sheet
Β-bend/ reverse
Loop & coils
Super secondary structure
Secondary structural elements are stabilized by extensive hydrogen bonding.
Tertiary structure
Quaternary structure
The arrangement of polypeptide subunits is called the quaternary structure of the protein
results from the aggregation (combination) of two or more polypeptide subunits held together by non-
covalent interaction like H- bonds, ionic or hydrophobic interactions.
Subunits may either function independently of each other , or may work cooperatively , as in hemoglobin.
Ex- Hemoglobin is a globular protein with four polypeptide chains (tetrameric)
b)
Primary structure
peptide bond
Secondary structure
peptide bond
hydrogen bond
Tertiary structure
Hydrogen bonds
ionic bonds
van der waals
hydrophobic interactions
disulfide bonds
Quaternary structure
non-covalent interaction like H- bonds, ionic or hydrophobic interactions.
ii) CK
10TH BATCH
10TH BATCH PROPER MCQ
2) Glycogen
A) Degradation results mainly glucose -1-phosphate
B) Formed from fatty acid
C) A steroid hormone
D) It is major type of storage carbohydrate
E) Directly use as GTP in muscle tissue
3) Buffer is the mixture of weak acid and its conjugate base. Regarding weak acid
A) it’s protonated at the pH below pka value
B) Acetic acid is an example
C) It has conjugate acid
D) It completely dissociate in the water
E) It behaves according to the Henderson Hasselbalch equation
4) 27yr old woman urinolysis high concentration neutral amino acid and has hartnup disease. Which are found in
urine
A) Arginine
B) Histidine
C) Lysine
D) Phenylalanine
E) Serine
5) Regarding peptides
A) Polymer of Amino acid
B) Positive to biuret reaction
C) Aspartame is a commercial dipeptide
D) Glutathione is dipeptide
E) When written nitrogen terminal on left hand and hydroxyl on right hand
6) Regarding coenzymes
A) Are precursor forms of enzymes
B) Inorganic substances
C) Not bound to the enzyme
D) Thermo stable
Presented by 15th Batch 78 FHCS | EUSL
E) Usually regenerate
-6 fatty acid
- linoleic acid
B) α-linolenic acid
C) Oleic acid
D) Arachnoid acid
E) Gamma-linolenic acid
8) All nucleotides
A) Contain a pentose monosaccharide
B) Function as co–enzymes
C) Have 3 phosphate groups
D) Contains a nitrogenous base
E) Are stored in the cell nucleus
41) A young black man entered his physician’s office complaining of bloating and diarrhea. His eyes were sunken
and the physician noted additional signs of dehydration. The patient’s temperature was normal. He explained that
the episode had occurred following a birthday party at which he had participated in an ice cream eating contest.
The patient reported prior episodes of a similar nature following ingestion of a significant amount of dairy products.
This clinical picture is most probably due to a deficiency in
A) Salivary α-amylase
B) Isomaltase
C) Pancreatic α-amylase
D) Sucrase
E) Lactase
01. Bio molecules play extremely important roles in the functioning of all body tissues.
1.1 Briefly describe the functions of carbohydrates. (35 marks)
1.2 Outline the process of protein denaturation and factors influencing it. (30 marks)
1.3 Briefly describe the types of plasma lipoproteins and their role in human body. (35 marks)
Answers
1)
A) T
B) F
C) F
D) F
E) T
2)
A) T glycogen is converted to glucose-1-phosphate by glycogen phosphorylase
B) F a glucose polymer
C) F is a storage polysaccharide
D) T other storage polysaccharide, starch, inulin
E) T
3)
A) T At pH values less than the pKa, the protonated acid from (CH3-COOH) is predominant.
B) T CH3-COOH
C) F
D) F 𝐶𝐻3𝐶𝑂𝑂𝐻 ⇔ 𝐶𝐻3𝐶𝑂𝑂− + 𝐻+
E) T pH = pKa + log [A-]/[HA]
4)
A) F Hartnup disease – characterised by defective absorption and defective transport of tryptophan.
B) F
C) F Neutral amino acid- Glycine, Alanine, Valine, Leucine, Glutamine, serine, Threonine, proline, tyrosine,
Asparagine, Isoleucine, cysteine, methionine, phenylalanine, tryptophan
D) T basic- Histidine, Arginine, lysine
Presented by 15th Batch 80 FHCS | EUSL
E) T acidic-Aspartate, Glutamate
5)
A) T peptides are polymers of 2-50 amino acids bound by peptide bonds. (Proteins > 50)
B) T biuret test -;
Reagent’s CuSO4 / KOH,
Original colour – blue
Turns into violet in the presence of peptide bonds.
C) T aspartate-phenylalanine dipeptide, Non-saccharide sweeter.
D) F Glutathione is a tripeptide, an antioxidant
E) T
6)
A)F coenzymes are small organic molecules that help enzymes in their reactions.
1. Co-substrate – transiently bound
2. Prosthetic group – permanently bound
B) F if the non-protein moiety is a mental ion such as Zn2+ or Fe3+ (inorganic) co-factor
C) F
D) T because it is a non-protein thermo stable.
E) T NAD+ ⇔ NADH
7)
A) T
B) F
C) F
D) T
E) T
8)
A) T
B) F NAD, FAD function as coenzymes.
C) F 2P-ADP, 1P-AMP.
D) T
E) F Found in other organelles such as mitochondria
9)
A) F Topoisomerase - unwinds DNA
Helicase- breaks H bonds
Polymerase- forms complementary DNA strand in 5’-3’ direction.
B) T DNA polymerase 1 – remove RNA primers and replace them with DNA.
C) T DNA polymerase 3 – add nucleotides to the 3' end and build a new stand in 5’-3’ direction.
D) F 5’-3’ direction
E) F polymerase 3 –synthesis in 5’-3’ direction – leading stand
41) ANSWER – E
The physical symptoms suggest a deficiency in an enzyme responsible for carbohydrate degradation. The
symptoms observed following the ingestion of dairy products suggest that the patient is deficient in
lactase.
42) ANSWER – B
43) ANSWER – C
β-Bends often contain proline, which provides a kink. The α-helix differs from the β-sheet in that it always
involves the coiling of a single polypeptide chain. The β-sheet occurs in both parallel and antiparallel
forms. Domains are elements of tertiary structure. The α-helix is stabilised primarily by hydrogen bonds
between the –C =O and –NH– groups of peptide bonds.
44) ANSWER – B
Arachnoid acid, (ω-6 acid contains 20 carbon & double bones) is immediate precursor of predominant type
of prostaglandin.
1.
1.1
● Carbohydrates act as a storage form of energy.
● They are glycogen, starch and insulin.
● Starch is the main storage polysaccharide of plants.
● It is a polymer of α glucose and consists of 2 glucose units called amylose and amylopectin.
● Glycogen is the storage polysaccharide of animals.
● Inulin acts as the storage polysaccharide in some plants like onion, garlic etc.
● It is a polymer of D- fructose units.
● Carbohydrates act as structural components.
● Glycosaminoglycans serve as structural components in humans.
● They are long unbranched polysaccharides.
● Functions of glycosaminoglycans are binding and packing of tissues repair of injury, defence
against infections, transport of metabolites between capillaries and tissues and connect, anchor
and support the body and its organs.
● Cellulose serves as a structural component of plants.
1.2.
● Protein denaturation results in the unfolding and disorganisation of a protein’s secondary, tertiary
structures and quaternary structures without the hydrolysis of peptide bonds.
● Denaturing agents include
Heat
Organic solvents
Strong acids or bases
Detergents
Ions of heavy metals such as lead
● Denaturation may under ideal conditions be reversible such that the protein refolds into its original
native structure when the denaturing agent is removed.
● However most proteins, once denatured, remain permanently disordered.
● Denatured proteins are often insoluble and precipitate from solution.
1.3.
● Lipoproteins are molecules which contain lipids, proteins and phospholipids.
● They are water soluble molecules.
● Outer Shell of lipoprotein is made of a phospholipid layer.
● Apolipoproteins which are embedded in layers stabilise the structure. Lipoproteins are classified
into chylomicrons, very low density lipoproteins, intermediate density lipoproteins, low density
lipoproteins, high density lipoproteins .
● Chylomricrones consist of apo B-48 apo E, apo Cii.
● Chylomicrons carry dietary triacylglycerol, cholesterol, fat soluble vitamins, and cholesterol esters
to peripheral tissues.
● LDL consists of apo B-100, apo E .
● Low density lipoproteins contain much less TAG than VLDL. And have a high concentration of
cholesteryl esters.
● They provide cholesterol to peripheral tissues.
● High density lipoproteins consist of apo A-1, apo Cii, apo E.
● They supply apolipoproteins for chylomicron an VLDL metabolism. They uptake the unesterified
cholesterol.
● HDL also reverses the cholesterol transport.
2.
2.1.
● Competitive inhibitors compete with substrate molecules for active sites of enzymes.
● Bind reversibly.
● Inhibitory action is proportional to its concentration.
Presented by 15th Batch 83 FHCS | EUSL
● So, the effect of the inhibitor is reversed by increasing substrate concentration.
● At high substrate concentration, velocity reaches Vmax observed in
absence of inhibitors.
● So, Vmax is constant.
● Increase Km for a given substrate.
● So, more substrate is needed to achieve 1⁄2 Vmax.
Drug therapy
● Statin drugs competitively inhibit rate limiting steps in cholesterol biosynthesis.
Inhibit HMG- CO A reductase
Are structural analogues of natural substrate.
So, lower plasma cholesterol level
Ex: atorvastatin
Pravastatin
2.2.
● DNA transcription is the 1st stage of protein synthesis.
● It is the process of copying the sequence of one strand of DNA to form mRNA.
● Require RNA polymerase.
Initiation.
● RNA polymerase binds to a region of DNA known as promoter.
● It signals the start of a gene.
● RNA polymerase does not need a primer and there is no error correcting ability .
● Transcription factor assemble at promoter to form a transcription initiation complex
Elongation
● RNA polymerase unwinds the DNA by breaking the H bonds.
● Separate two strands and form a transcription bubble.
● Base pairing occur between incoming RNA nucleotide and DNA
nucleotides of genes.
● RNA uses uracil instead of thymine.
● RNA polymerase catalyses the bond formation between nucleotides.
● Bubbles move forward and rewarding behind the DNA strand.
Termination
● Terminator region signals the stop of the gene.
● RNA polymerase releases the mRNA and DNA.
mRNA editing
● a post transcriptional process.
Introns - non functional segments
They cut off
Exons - coding region of DNA
They region by ligase
● A guanosine triphosphate cap is added to 5’end of the new Mrna.
● A poly A tail is added to 3’end of mRNA.
● mRNA leaves through pores into cytoplasm.
01.Alpha helix
A) Left hand
B) Disturb by proline
C) Is stabilize by H bond between the C=O in main chain
D) All globular proteins at same percentage
E) Play mechanical role in stiff bonding of fibres in keratin.
03.
04.Irreversible enzyme inhibitors are
a) CN
b) snake venom
c)nerve gas
d)organophosphates
e) statin drugs
07.Regarding glycolysis
A. Occurs in cytoplasm
B. Lactate is an intermediate
C. hexokinase enzyme is used for the committed step
D. pyruvate kinase reaction require ATP
E. high level of citrate inhibits glycolysis
01.
1.1. Outline the role of glycosaminoglycans in the human body. (25 Marks)
1.2. Briefly describe the functions of biologically active di and tri peptides. (20 Marks)
1.3. State the functions of kreb’s cycle. (25 Marks)
1.4. Outline the role of different types of RNA in a eukaryotic cell. (30 Marks)
Answers
1)
A)F right handed spiral structure
B)T
C)T
D)T
E)T
3)
A)F Mnemonic-PVT TIM HALL
B)F
C)F
D)T
E)F
4)
A)T
B)
C)T
D)T
E)F
5)
A)F right handed double helix
B)T
C)T
D)T
E)
6)
A)F same reaction
B)
C)T
D)T
E)T
7)
A)T
B)F
C)F fructose 6-phosphate → fructose 1,6- bisphosphate.
D)F
E)T
8)
A)T
B)T
C)T
D)F
E)T
9)
Presented by 15th Batch 87 FHCS | EUSL
A)F
B)T
C)
D)T
E)T
42) ANSWER D
43) ANSWER A
45) ANSWER E
1.
1.1
• GAGs are highly polar and can attract a large amount of water. Thereby produce a gel like matrix which
forms the basis of ground substance. Fibrous proteins make up an extracellular matrix.
• GAGs are highly negatively charged with extended conformation that gives high viscosity to the solution.
• Their low compressibility makes these molecules ideal for lubricating fluid in the joints(synovial).
• Rigidity provides structural integrity to cells and passageways for cell migration.
1.2
• Dipeptides are two amino acids joined by a peptide bond. For example Aspartame- aspartic acid with
phenyl alanine. Used as a sweetening agent replacing sugarcane.
• Tripeptide is three amino acids joined by two peptide bonds. Example Glutathione formed with glutamic
acid, cysteine and glycine. It helps in absorption of amino acids, protects against haemolysis of RBC by
breaking H2O2 which causes cell damage.
1.3
• Provides final common pathway for the oxidation of carbohydrate, fat and
protein.(Integration of major metabolic pathways.)
• Produce energy either directly as ATP or as reducing agents NADH or FADH2 oxidized in ETC
• Provides substrates for the ETC
• Source of biosynthetic precursors. Example –porphyrin from succinyl CoA, amino acids from OAA and
alpha ketoglutarate
• Some intermediates exert regulatory effects on other pathways. Example- citrate inhibits PFK-1 in
glycolysis.
1.4
1. mRNA–function as messengers which carry information in a gene to the protein synthesising ribosomal
RNA
2. rRNA- structural component of ribosome. Necessary for ribosomal assembly and play a key role in
binding of mRNA to ribosomes and its translation.
3. tRNA- carry the specific amino acid to the ribosome.
1. Glycosaminoglycans,
a. expand in nature due to negativity
b. unbranched heteropolysaccharides
c. keratan sulfate softens vitreous
d. chondroitin sulphate is the highest in ECM
e. dermatan sulphate lubricates synovial fluid
3. Covalent modification,
a. Alter the quantity of enzyme
b. Mediate by hormones
c. Occur in amino acid residues contain hydroxyl residues
d. Of digestive enzymes by proteolytic cleavage
e. Of glucagon synthase regulate glycogen synthesis
5. Alpha helix,
a. H bonds occur between electro negative atoms of side R chains of amino acid
b. H bonds are perpendicular to axis
c. The structure is stabilized by salt linkage
d. Branches of amino acids are exterior
e. Proline stabilize the helix
6. T form of Hb
a. Deoxy form of Hb
b. Has high oxygen affinity form
c. Has low oxygen affinity form
d. H bonds and ionic bonds limit the movement of monomer
e. The binding of oxygen destabilizes some of the H bonds and ionic bonds particularly between alpha – beta dimers
8. Regarding leukotrienes
a. Increase vascular permeability
b. Induce chemotaxis
c. Destroyed neutrophils
d. Induce muscle contraction
e. Synthesis from arachnoid acid
13TH BATCH
13TH BATCH PROPER MCQ
1. Starch
A. Pink colour appearance with iodine
B. its a homopolysaccharide
C. high in leafy vegetables
D. predominantly it contains amylopectin
E. Structure support by disulphide bond
3. PDH complex
A. Catalyze the oxidative decarboxylation of pyruvate
B. Activated by acetyl co A
C. Activated by insulin
D. Vitamin B12 need for action.
E. Produce 1 NADH molecule as a product
6. Regarding Isoenzymes
A. identify by electrophoresis
B. catalyze same reactions
C. have different kinetics
D. present in different subcellular organs
E. genetically determined Primary stucture are same
7. Regarding DNA,
A. Containglycosidic bonds between nucle.
B. H formation occur between purine & pyramids .
C. Hydrophobic bases locate inside the helix.
D. Uracil present in DNA.
E. Denature in high temperature.
44. A person recently diagnosed with a urinary tract infection was prescribed with sulfa drug which alters Km value.
What is the type of inhibition?
A. Allosteric
B. Competitive
C. Feedback
D. Non competitive
5.
5.1. Write short notes on restriction endonucleases (25 marks)
5.2. 5.2.1. Briefly describe the general structure of hemoglobin (15 marks)
5.2.2. Give the biochemical basis for sickle cell anaemia (30 marks)
5.3 5.3.1 What is genetic code ? (10 marks)
5.3.2 Explain the characteristic features of genetic code (20 marks)
6.
6.1. Describe the functions of carbohydrates that are important to humans (35 marks)
6.2. Outline the structure, location and functions of collagen in the human body (30 marks)
6.3. List posttranscriptional ad posttranslational modifications (15 marks)
6.4. State the main functions of Kreb’s cycle (20 marks)
14TH BATCH
14TH BATCH PROPER MCQ
02. Glycogen,
A) Degradation result mainly glucose 1 phosphate
B) Formed from fatty acids
C) A steroid hormone
D) It is a major type of storage carbohydrate
E) Directly used as GTP in muscle tissue
29. True/false
A) Disulfide bond require cysteine to be adjacent in the primary structure
B) Protein consisting of a single polypeptide chain has quaternary structure.
C) The peptide bonds are always trans
D) The primary driving force for protein is the hydrophobic effect.
E) Beta sheets are antiparallel.
57. The retained segment of RNA when pre mRNA becomes a mature mRNA
A) Codon
B) Exon
C) Intron
D) Nucleotide
E) Splice boundaries
59. In citric acid cycle substrate level phosphorylation occurs at the conversions of?
A) Alpha ketoglutarate to succinyl co A
3.
3.1. Briefly describe the functions phospholipids with examples. (20 marks)
3.2. Write a note on
3.2.1. Protein denaturation. (25 marks)
3.2.2. Clinically important isozymes. (35 marks)
3.2.3. Pyruvate kinase deficiency. (20 marks)
4.
4.1. List the functions of proteins and enzymes involved in DNA replication. (20 marks)
4.2. Briefly describe the role of eukaryotic RNA polymerases in transcription. (30 marks)
4.3. Briefly describe on regulation heme biosynthesis. (30 marks)
4.4. State 5 conditions that will decrease the plasma albumin level (20 marks)
15TH BATCH
15TH BATCH PROPER MCQ
54) In the study of enzymes, a sigmoidal plot of substrate concentration versus reactions velocity may indicate?
A. Competitive inhibitors
B. Cooperative binding
2.3 State the inhibitors of each of the enzyme complexes of electron transport chain (10 marks)
2.4 Briefly describe the functions of the electron transport mechanism (15 marks)
2.5 Briefly discuss the importance of essential fatty acids in human with examples (25 marks)
3.
3.1. Mention the functions of amino acids in human (15 marks)
3.2. Briefly describe 3.2.1. the regulation of enzyme activity (35 marks)
3.2.2. DNA replication (30 marks)
3.2.3. the basic steps in DNA cloning with plasmids (20 marks)
SBR
11. In which phace of the cell cycle will you find chromosomal condensation inactivation & not transcribed to
mRNA.
A. G_0 Phase
B. G_1 Phase
C. G_2 Phase
D. S phase
E. M phase
02. Urine was collected in a subject at 30 minutes interval for 2 hours after drinking one litre of water rapidly and its
volume and specific gravity were measured. Results are shown below:
2.1. Comment on the above readings. (20 marks) 2.2. Explain the Physiological basis of the deviations from the
normal (80 marks)
1.
1.1 Briefly describe the defect of cell division that causes Down syndrome (30)
A genetic disorder causing developmental and intellectual delays. Due to abnormal cell division resulting extra
chromosome 21, trisomy of 21 chromosome.
Two types,
1. Trisomy 21 (96%)- Extra copy of 21 in every cell. Maybe due to either nondisjunction of homologous
chromosomes in meiosis 1 or nondisjunction of sister chromatids in meiosis 11
2. Translocation (4%) – Due to Robertsonian Translocation. Children have only extra part of chromosome 21. 46
total chromosomes but 13/14/15/21/22 chromosomes has an extra chromosome 21 attached. Down’s syndrome
causes distinct facial appearances, intellectual disability and developmental delays. Maybe associated with heart
and thyroids.
1.2 Describe the fluid mosaic model of the eukaryotic cell membrane structure (40)
• Resemble a continually moving sea of fluid lipids that contain a mosaic of many different proteins. 7.5- 10nm
thick.
• Basic structural framework is the phospholipid bilayer.
• Thin, pliable, elastic structure.
Presented by 15th Batch 101 FHCS | EUSL
• 2 back-to-back layers made up of three types of lipid molecules—phospholipids (75%), cholesterol (20%), and
glycolipids (5%) .
• Amphipathic- have both,
→ Polar hydrophilic head (outward)
→ Nonpolar hydrophobic tail (inner)
• Membrane fluidity depends both o on the number of double bonds in the fatty acid tails of the lipids that make
up the bilayer (Each double bond puts a “kink” in the fatty acid tail) , and o on the amount of cholesterol present.
• Different proteins responsible for uniqueness of different membranes
• Proteins can be just on the surface,
1. peripheral proteins
→ Loosely bound to surface, often attached to integral proteins (eg.- enzymes) • Or embedded in the membrane,
2. integral proteins,
→ A few integral proteins are tightly attached to one side of the bilayer by covalent bonding to fatty acids. Like
membrane lipids, integral membrane proteins are amphipathic.
→ Some transmembrane proteins which span the thickness of bilayer.(channel pores, carrier proteins)
• Carbohydrates makeup about 3 %
• Combined with proteins or lipids (glycolipids, Proteoglycans)
• Form glycocalyxes
1. Reception;
two components ‐ Receptor is a protein molecule that receive chemical signal. They trigger signalling cascade. Can
be upregulated or downregulated. Trigger signalling cascade.
I. Intracellular receptors
II. Cell surface receptors; Channel- linked receptors Enzyme linked receptors G- protein coupled receptors ‐
Ligand is the molecule that bind to receptor.(Eg.-Neurotransmitter)
2. Transduction ‐ Relay molecules resulting from signal receptor binding ‐ Second messengers pass the signal
initiated by the first messenger (Eg.- Ca2+, IP3 )
3. Response Can be,
l. Gene expression
ll. Cellular metabolism
‐ Gene expression is the process by which information from a gene used to produce functional protein
‐ Example for cellular metabolism is Adrenalin preparing the body for short term emergencies
3) Cyclin dependant protein kinases (CDK’s) regulate the cell cycle. That check,
a. Microtubule attached to the centromere
b. DNA damage
c. Cell size and grow
d. Number of times the cell can divide
e. Cell size and nutrients
6) Which one of following brings the body temperature to normal when it is high?
a. Raising of skin hair
b. Redistribution of blood flow to periphery
c. Shivering
d. Sweating
e. Thermogenesis
SBR
11) What is the predominant buffer in interstitial fluid,
a. Cl-
b. HCO3-
c. PO43-
d. Proteins
e. Sulfate ion
13) Plasma and interstitial fluid are included in extra cellular fluid. Most significant feature seen in plasma
compared to interstitial fluid,
a. Plasma contain high amount of sodium ion than interstitial fluid.
b. Plasma has higher buffering capacity
c. Plasma has more protein than ISF
d. Plasma is electrically more negative
e. Plasma is more viscous than that of interstitial fluid
14) A patient is admitted to the hospital emergency treatment ETU.A blood gas analysis was performed the results
are follow
P O2 12.4 (11- 13)
P CO2 55(47-60)
Ph 7.29 (7.35- 7.45)
HCO3- 15 (22-36)
a. Compensated metabolic acidosis
b. Metabolic acidosis
c. Metabolic alkalosis
d. Respiratory acidosis
e. Respiratory alkalosis
01)
1.1 Briefly describe the formation of Trisomy 21 during the meiosis. (30)
1.2 Briefly describe the role of protein in plasma membrane
1.3 Outline the types of cell surface receptor with example(15)
1.4 briefly explain the role of free radicals in cellular aging (25)
2. 2.1 List the four starling forces
2.2 Using diagram, illustrate the role of these forces in environment
2.3 Cardiac failure results in increased capillary venous pressures
2.4 Outline the clinical features of dehydration on physiological basis
1.4 briefly explain the role of free radicals in cellular aging (25)
Many changes that occur in our body are caused by free radicals. Free radicals have a free electron which bind to
other essential molecules to stabilize themselves. This cause cells not to perform their normal functions. They can
damage DNA, proteins, lipids, cell membranes and other parts of human cells. Damage to DNA results in production
of ineffective proteins and cell death. Damage to proteins causes disease directly. Therefore it accelerate the aging
process.
1) Cell Membrane
A. Consist of lipid bilayer
B. A constant composition of protein throughout life of the cell
C. Cell membrane is a dynamic structure
D. Is freely permeable to protein
E. Protein structure and enzyme content is varies cell to cell
4) Nucleus
A. Controls activities in the cell
B. Has a nuclear organizing region to reform the nucleolus in telophase
C. Has more euchromatin in metabolically active cells
D. There are pores in nuclear membrane to transport ribosomes
E. Present in mature RBC
SBR
11) What Contributes least to the osmolarity of plasma
A. Cl
B. Glucose
C. Protein
D. Sodium
E. Urea
13) Which from the followings is incorrect about the total body water
A. Is measured by the calculating the dilution of source in the compartment
B. Constitutes about 55% of the body weight in adult male
C. Mainly in intracellular compartments
D. Proportionally high in infants than elders
E. Proportionally low in women than men
15) Which following method that loss water without losing solutes?
A. Faces loss
B. Gastric juice
C. Insensible perspiration
D. Pancreatic juice
E. Sweat
1.
1.1. Write an Account on the cell cycle including the check points (30 Marks)
1.2.
1.2.1. Briefly describe the functions of eukaryotic cell membrane. (25 Marks)
1.2.2. outline the G protein coupled mechanism in the signal transduction pathway with an example (25 Marks)
1.2.3. state the Factors Affecting cellular ageing process (20 marks)
2.
2.1. Explain he physiological basis of formation of oedema in heart failure (50 marks)
2.2. Explain briefly the effect of adding following solutions to the extra cellular fluid
2.2.1. Isotonic saline (25 marks)
2.2.2. Hypotonic saline (25 marks)
01)
AT
Presented by 15th Batch 108 FHCS | EUSL
BF
CT
DF
ET
02)
AT
Transporting substances that could not penetrate the layer
BT
Water, water soluble substances, ions can diffuse between the cells. Also have selective properties .
CT
DF
ET
Receptors for peptide hormones.
03)
AT
BT
CF
Smooth endoplasmic reticulum contains enzymes for metabolism and detoxification of substances in the
hepatocytes such as drugs, hormones and alcohol.
DT
Presence of polyribosomes on the cytoplasmic surface give them a rough granular appearance.
EF
SER of hepatocytes contains glucose-6-phosphatase enzyme which convert the glycogen in to glucose in liver cells .
04)
AT
BT
During the cell division the nucleoli disappear and around the nuclear organizing region, nucleoli reform during
telophase of cell division.
CT
Euchromatin is metabolically active and involve in protein synthesis.
D
EF
05)
AT
BF
CT
DF
ET
06)
AF
Total body water -: 60% of body weight ➢ Intra cellular fluid: 2/3 of TBW ➢ Extra cellular fluid: 1/3 of TBW:
interstitial tissue 75% of ECF : plasma 25% of ECF
BT
CT
Presented by 15th Batch 109 FHCS | EUSL
DF
ET
07)
A F Content of ORS: ➢ Sodium chloride ➢ Glucose ➢ Potassium chloride ➢ Trisodium citrate
BT
CT
DT
ET
08)
AF
❖ Isotonic dehydration Loss of water and electrolytes in equal proportion.
EG-: burns, vomiting, haemorrhage, diarrhea Treatment -> replace the loss of isotonic solution. 0.9 % saline ,
Hartmann’s solution , 5%dextrose
❖ Hypertonic dehydration Loss of water excess of solutes.
EG-: fever , severe sweating , diabetes insipidus Treatment -> infusion of hypotonic solutions. 5% dextrose (later
become hypotonic) , 0.45% saline
❖ Hypotonic dehydration Loss of electrolytes in excess water . water moves in to the cells . EG-: excessive use of
diuretics , hyperglycemia Treatment -> hypertonic saline
BT
CT
DF
ET
09)
AT
Blockage of lymph return cause for oedema.
BT
Increase capillary permeability
CT
DT
Increase capillary pressure due to high venous pressure.
E
10)
AT
BT
CT
DT
EF
11)
Answer E
12)
Answer C
fluid moves in to the cells – swelling of cells causes for lysis .
13)
Presented by 15th Batch 110 FHCS | EUSL
Answer B
Total body water – 60% of body weight Intracellular fluid 2/3 of TBW Extra cellular fluid 1/3 of TBW
❖ PHYSIOLOGICAL VARIATIONS IN TBW
Age – TBW as a percentage of body weight decrease with age. At birth TBW – 80-85% of body weight. Male >
female
Fat content – greater fat content lesser the TBW as a % of bodyweight. HYDROPHOBIC
14) Answer B
A molecule that bind to a receptor is called a ligand , and can be a peptide ( short – protein) or another small
molecule such as a neurotransmitter, hormone, oharmaceutical- drug, toxin or parts of the outside of a virus or
microbe.
15) Answer E
Loss of water excess of solute – Hypertonic dehydration ECF Water moves from ICF to ECF ICF Volume decreased
decreased osmolality increased increased
10. Correct the match components of cellular membrane and their role,
A) Cholesterol-control fluidity of cell membrane.
B) Glycolipid-cell recognition.
C) Integral proteins-enzyme
D) Peripheral protein-control transport of substances.
E) Sphingolipid-signal transmission
SBR.
11. Because of the respiratory obstruction disease, the Pco2 level of the plasma increases (40-60) and plasma PH
reduced from 7.4-7.0. What could be the result of the
condition?
A) Decrease plasma HCO3
B) H+ Concentration in the urine is increased.
12. Which process is involved, when transporting glucose from the intestinal tube to the epithelial cell is inhibited,
and the Na+ transportation across the cell membrane along the concentration gradient is also disturbed?
A) Co transport
B) Counter transport
C) Facilitated diffusion
D) Secondary active transport
E) Simple diffusion
13. Homeostasis is achieved through feedback mechanism. Which is not true regarding positive feedback
mechanism?
A) Positive feedback mechanism results in amplification of output signals.
B) Involve in regulation of blood clot formation pathway.
C) Involve in regulation of homeostasis which constrained in time frame.
D) Involve with minimizing the initial intensification of stress.
E) It involves in breakdown of homeostasis of the system.
15. What are the factors that does not allow formation of free radicals.
A) Environment contamination.
B) Electron transport chain.
C) Excessive consumption of processed foods.
D) High intake of fruits and vegetables.
E) Ionizing and ultraviolet radiation
1.
1.1. Briefly describe the possible errors occur during the cell division. (30 marks)
1.2. Describe the role of second messengers in cell signaling pathway with examples.
(30 marks)
1.3. Explain the biochemical basis of cellular aging process. (40 marks)
2. Disturbance in Starling forces lead to accumulation of fluid in extracellular compartment.
Explain the physiological basis of following condition in relation to disturbance in the
above forces.
2.1. Dyspnea (shortness of breath) and limb oedema with patient suffer with congestive
heart failure. (40 marks)
2.2. Generalized oedema with patient suffered with end stage liver. (cirrhosis) (20 marks)
2.3. Explain the physiological basis of using oral rehydration solution in Diarrhea.
(40 marks
Presented by 15th Batch 113 FHCS | EUSL
14TH BATCH EME – MCQ ANSWERS
01) Answer - T, F, T, F, T
c) theory
02) Answer - T, T, F, T, T
b) small in size comparing with parent cell initially after mitosis due to equal division of cytoplasm.
a) d) main function of mitosis. helps to replace dead cells and wound healing.
d) This is why secondary active is called as an active process though it doesn't use energy directly. Concentration
gradient of particular ion which is produced by active ion pumping is used to transport another substance against
its own concentration gradient.
04) Answer T, T, F, T, F
b) Initially its higher in infants than childhood but later ICF becomes dominates.
a) ECF>ICF infant
d) water content is inversely proportional to body fat content. so high fat in obesity results in low water content.
e) theory
05) Answer F, F, T, F, F
a) Osmoreceptors are stimulated under such condition which are present in hypothalamus.
b) Contribution of proteins to osmolarity is less than 2 m osmoles. So, there won't be much change.
c) Due to the secretion of ADH in response to increased osmolarity will result in reabsorption water in renal tubules
in an increased manner so low water more substances ---> concentrated urine.
d) Already 0.45% is a hypotonic solution. Since it's taken as 2l almost it'll be more hypotonic in nature. Also,
retention of NaCl solution in blood will result in hypo-osmolar plasma.
a) e) Decrease in osmolarity will result in reduced release of ADH to elevate the osmolarity by excreting a dilute
urine (water>solute).
06) Answer F, T, T, T, F
a) In case of a solution of NaCl, it'll result in increasing blood osmolarity hence increasing ADH secretion which will
eventually lead to fluid excess.
c) fluid overload and changes in net filtration pressure could lead to edema.
d) In case of glucose administration, it could lead to overload of glucose in cell and pyruvate via glycolysis and could
result in increased lactic acid formation.
07) Answers T, T, T, F, F
a) In case of metabolic acidosis large acids are added or loss of base will result in excess of H+. So, the pH of blood
will be reduced.
b) In order to neutralize the excess H+, HCO3- which presents in blood will accept H+ and forms H2CO3. Hence it
will continue toform CO2 and water at lungs. So, CO2 will be exhaled via expiration. So, this will reduce the PCO2 of
arterial blood after circulation via lungs.
c) Since neutralizing H+ via HCO3- will be reduced and will be compensated from bicarbonate reservoir later.
d) Reduced plasma pH will simulate the respiratory centers and will result in hyperventilation or increased
respiratory rate and depth to remove H+ via CO2.
e) Here's a condition to conserve HCO3- to compensate the loss of HCO3- during the neutralization process occurs
in blood. So, for that renal tubular absorption of HCO3- will be enhanced.
08) Answer: T, T, T, F, F
Other than E all are physiological buffers present in body as intrinsic and extrinsic buffers. though triglycerides can
function as a buffer invitro it’s not a buffer available in vivo.
Presented by 15th Batch 115 FHCS | EUSL
09) Answer T, F, F, F, T
a) Lipophilic molecules are able to diffuse through the plasma membrane easily unlike water soluble molecules
which requires special transport carriers.
b) Binding of a signaling molecule to the receptor results in activation in extracellular receptors. But in case of
intracellular they're mostly accompanied by release of molecules which alters gene expression.
e) e) upon binding release of inhibitory complex and that receptor protein binds with genome and alters gene
expression.
10) Answer T, T, T, T, T
d) theory
because this is a condition called respiratory acidosis due to the obstruction of air way and reduced removal of CO2.
will result in favoring the bicarbonate buffer equation in forward direction hence will increase more HCO3- and
arterial chemoreceptors will remain stimulated despite the inability to inhale breath due to the obstruction. H+ will
be excreted via kidney and HCO3- will be preserved. And H+ are not buffered by non-bicarbonate buffer system at
this movement.
12) Answer A
The reason is since the both Na+ and glucose transported in same direction and transport occurs only if glucose is
present without any inhibition refers as co-transport.pumping activity of Na/K ATPase pump forms low intracellular
Na+ concentration hence Na+ concentration gradient across the membrane towards inside (+). By using this
gradient Na+ is transported following its concentration gradient (passive) but glucose is transported against its own
gradient, so whole process is active since the required Na+ gradient achieved by the active pumping of Na/K ATPase
pump.
Because positive feedback mechanism amplifies output signals, involves in blood clot formation by releasing more
and more serotonin, and involves in homeostasis which should happen with a constrained time frame. also, it
14 )Answer is C
15) Answer-D
a) environmental contamination means chemicals pesticide provokes damages of bio molecules and yielding free
radical
b) free electrons reacting with molecular oxygen in absence of enough H could result in ROS formation.
c) fruits and vegetables comprise antioxidants hence they help to get rid of free radicals.
2) Regarding meiosis,
A. It generates genetic diversity
B. it is a cell division that produce haploid(n) sex cells
C. It occurs after DNA replication
D. Produce 2 daughter cells with same genetic materials
E. Responsible for subsequent growth and development of organisms
5) Simple diffusion,
A. Facilitates the movement of blood gasses through the cell membrane
B. Facilitates the movement of fat-soluble vitamins across the cell membrane
C. Is an active process
D. Moves the substances through the electrochemical gradient
E. Rate decreases in higher temperature
6) Plasma osmolarity,
A. Increase will decrease the release of ADH by pituitary gland
B. Increase with hypoproteinemia
C. Mainly determined by blood Na+, glucose and urea
D. Measures the hydration status of the body
E. Range between 280-300mOs/kg
7) Regarding dehydration,
A. It is caused by hyperventilation
B. Can occur hypovolemic shock leads to dysfunction
SBR
11) What is the following are important for glucose absorption into the intestine?
A. Cotransport
B. Counter transport
C. Facilitated diffusion
D. Primary active transport
E. Simple diffusion
14) Which of the statement is not true regarding composition of body fluid?
A. 1,3,2,4
B. 1,4,2,3
C. 3,2,1,4
D. 3,1,4,2
E. 4,2,3,1
1.
1.1 State the function of microtubules in cell division. (15marks)
1.2 Briefly describe the different composition of eukaryotic cell membrane and their role. (25 marks)
1.3 State the importance of telomeres on cellular aging process. (10 marks)
1.4 Explain the physiology behind the conditions
1.4.1 Person having higher ADH secretion due to Syndrome of Inappropriate Antidiuretic Hormone
secretion (SIDAH) develops seizures due to acute oedema in brain cells. (25marks)
1.4.2 A patient admitted with severe episodes of vomiting was found to have high arterial HCO3
and PCO2
03) False – membrane bound flattened sacs which connects nucleus is RER
True
True- modification of protein ( glycoprotein, lipoprotein)
False – protein synthesis occurs in ribosome
True- lysosome is secreted by golgi
04) True
True- fluidity depends on number of unsaturated fatty acids, amount of cholesterol,
temperature
True- fluidity allows interaction with plasma membrane, enables the movement, self steal role
False- increase with temperature
False- cholesterol
06) False- if osmolarity increased water should be reabsorbed , so ADH release increases
False- low level of protein decrease the osmolarity
True- osmolarity is determined by electro chemicals , glucose and urea
True
True
07) True – hyperventilation can cause dehydration
False- hypovolemic shock leads to hypovolemia not dehydration
True- during dehydration ECF water reduced , so osmolarity increased so water flows from ICF to ECF , so
both the ECF,ICF volume will be reduced
False-hypertonic extra cellular fluid
True- increased osmolarity cause increased ADH , so high amount of water will reabsorbed, so
concentrated urine . so it will lead to oliguria .
Oliguria means low urine output
08) True
True
False – localized edema
False- localized edema
True
10) True
False- G protein is an integral protein
False- alpha subunit binds with G protein
True
True – adrenergic , muscarinic ,metabotropic glutamate receptors and receptors for peptide hormone act
through G protein receptors
11) Answer A
In the intestine glucose is absorbed by co transport
12) Answer E
Vomiting cause depletion of H + ions , it cause metabolic alkalosis
13) Answer C
In protein malnutrition capillary oncotic pressure is decreased . so fluid reabsorption will decrease , so
filtration is increased
14) Answer B
Intracellular anion concentration is higher than plasma , so intracellular is more negative.
15) Answer D
1.1.
1.2 .
Composition of cell membrane
Protein -55%
Phospholipid -25%
Cholesterol -13%
Other lipids -4%
Carbohydrates -3%
Sphingolipid –
Protect from harmful environmental factor
Signal transmission
Adhesion site for extra cellular proteins
Cholesterol _
Controls and maintain the membrane fluidity
Proteins-
Act as channel proteins and carrier proteins
Act as receptors for peptide hormones
Act as enzymes
Support in signal transduction
Cell cell recognition
Carbohydrates –
Play a key role in cell cell recognition
Act as antigen
Important in organ and tissue development
1.3.
Telomeres are protective structures present at the end of the chromosome .
Telomeres shorten as a result of cellular replication .
When the telomeres become too short , the cell enters the senescence stage .
Telomeres regulate how many times an individual cell can divide.
Telomeric sequences shorten each time the DNA replicates.
Once the telomere shrinks to a certain level , the cell can no longer divide.
It metabolism slows down .
It ages and dies .
1.4.
1.4.1.
This person has higher ADH secretion .
Normaly ADH reabsorp water in distal convoluted tubule and collecting duct in nephron.
Higher ADH secretion will lead high reabsorption of water in kidney.
So water level in blood will increase.
Blood osmolarity is decreased than normal level.
So the water will shift into cell and interstitial fluid.
So there is an abnormal accumulation of fluid in interstitial fluid and intracellular fluid.
It will lead to edema.
This occurs in brain cells also in SIADH .
So the brain cells will swallow and cause edema in brain cells and develops seizures.
1.4.2.
Patient admitted with severe episodes of vomiting .
02. Match the specific inhibitor for electron transport chain complex
A. ATP synthase- Antimycin
B. Cytochrome C oxidase- cyanide
C. Cytochrome C reductase- Oligomycin
D. NADH dehydrogenase- Rotenone
E. Succinate dehydrogenase - Carbon monoxide
SBR
11. Precursor of eicosanoids
A. C18-C20 saturated fatty acid
B. C18-C20 polyunsaturated fatty acid
C. C20-C22 saturated fatty acid
D. C20-C22 polyunsaturated fatty acid
E. C20-C22 saturated fatty acid
13. What is the Krebs’s cycle enzyme found in inter membrane space?
A. Citrate synthase
B. Malate dehydrogenase
C. Iso-citrate dehydrogenase
D. Succinate dehydrogenase
E. α- ketoglutarate dehydrogenase
14. Production and secretion of trypsin is inhibited in pancreas disease. Hydrolysis which compound?
A. Proteins
B. Lipids
C. Carbohydrate
D. Nucleic acid
E. Phospholipid
1.
1.1. Outline the mechanism of oxidative phosphorylation (25 marks)
1.2. State the function of different types of lipoproteins (15 marks)
1.3. List three (03) consequences occurred due to deficiency of disaccharidases enzymes in human (10 marks)
1.4. List the functions of any 5 enzymes or protein required for DNA replication (15 marks)
1.5. Briefly describe the beta pleated sheet structure of protein (20 marks)
1.6. Mention five (05) enzymes used as therapeutic agent (15 marks)
Answers
01.
A. (T) - Leads to heavy loss of water through expiration
B.
C. (F) - More water loss than solute. So it increase plasma osmolarity
D. (T) - Both ICF & ECF depletion leads to dehydration
E.
02.
03.
A. (T) - It has mitochondrial DNA
B. (F) - Glycosylation of proteins happen intracellularly
C. (T) - It has ribosomes. It synthesis protein for its own growth
D. (F) - Golgi body synthesis lysosomes
E. (T) - It’s a double membranous organelle
04.
A. (T) - Macromolecules are captured in vesicle on one side of the cell, drawn across the cell & ejected on the other
side
B. (F) - Gases pass through diffusion
C. (T) - Secondary active transport (Bulk transport)
D. (T)
E. (F) - Against the electrochemical transport (Active transport)
05.
A. (T)
B. (T)
C. (T)
D. (F) - When it’s cold they are found closer together. So decrease in low temperature.
E. F - cholesterol regulates the fluid bidirectionaly
06.
A. (T) - In our body
Solids- 40%
Liquid -60%
ECF – 20% Here Interstitial fluid -80%
Plasma-20%
ICF-40%
B. (F) - Na+ and k+ is high in the interstitial fluid but protein is very low so cl- is very low
C. (F) - In plasma, plasma proteins are there. But Interstitial fluid low Protein are there (low Protein is right
D.
E. (F) - proteins are high in the cell not ECF .
07.
A. (F) - congestive heart failure – It leads to increase increase capillary hydrostatic pressure mainly in the venous
fluid movement is increased Into interstitial fluid ( But here it is generalized edema ,not localized
B. (T) - When flea bites swelling is happen on that area
C. (T)
D. (F) - generalized edema … reduced capillary oncotic pressure
E. (T) - fluid accumulation of interstitial fluid is high
09.
A. collagen proteins
B.F
C.T
D.T
E.T
10.
A. T
B. F
C. T
D. T
E. T
11. answer B
12. (D)
Protein malabsorption leads to Decrease plasma oncotic pressure so capillary filtration will increase
13. (D)
When we Intake Isotonic solution which increase the cardiac output, Increase the flow rate of kidney, filtrate
increase,excreate urine will increase
14. (B)
Testosterone is a hydrophobic signaling molecule and Adrenaline is example for hydrophilic molecule.
Adrenaline actives G-Protein coupled receptors in many different tissues.
15. (A)
Cytokinesis is the physical process of cell division, which divides the cytoplasm of a parent cell into two daughter
cells.Karyokinesis is the step during cell division where the nucleus divides to form 2 daughter nuclei.
Role of phospholipid
lipid bilayer acts as a barrier to the passage of molecules and ions into and out of the cell.
The lipid layer in the middle of membrane is impermeable to water soluble substances (ions, glucose and urea) 2 &
permeable to lipid soluble substance.
Role of Sphingolipids on CM
Presented by 15th Batch 134 FHCS | EUSL
• Structural components
• Protection from harmful environmental factors
• Signal transmission
• Adhesion sites for extracellular proteins
Role of cholesterol in CM
It helps in determine the degree of permeability of the bilayer to water soluble constituents of body fluid and
Controls much of the fluidity of the membrane
1.3) Proto-oncogene
An oncogene is a gene that has the potential to cause cancer. In tumor cells, these genes are often mutated, or
expressed at high levels. Most normal cells will undergo a programmed form of rapid cell death when critical
functions are altered and malfunctioning.Proto-oncogenes normally regulate cell division, but can be changed into
oncogenes through mutation, which may cause cancers to form.
• gene mutation causes a change that increases the activity of a positive regulator.
1.4) A Patient diagnosed with chronic obstructive lung disease has the following arterial
blood values;
PH = 7
PCO2 = 50mm Hg [Normal range (35-45 mm Hg )]
[HCO3- =Normal range (22-26 mEq/L)]
Identify the acid base imbalance and explain the reason for observing above changes in the
blood test.
Chronic obstructive lung disease is a group of disease that cause airflow blockage and breathing related problems. It
includes emphysema, chronic bronchitis and asthma.
Normal arterial blood, PH = 7.35 -7.45
PCO2 = 35-45mmHg
[HCO3-]= 22-26 mEq/E
According to the patient’s blood test arterial blood pH is lower than the normal level. Therefore it is an acidosis
condition. But PCO2 and [HCO3-] are higher than the normal level. Therefore it can identify as a Respiratory
Acidosis.
The reasons for observing this changes are can’t expel CO2 properly, rate of CO2 generation exceeds than the rate
of CO2 removal, homeostasis disturbed by increasing PCO2 caused by hypoventilation.
1.5) Brain edema is caused by excess accumulation of fluid in intracellular or extracellular spaces of brain.
Hypertonic fluids have a greater osmotic pressure than the cell. When infused it raises the serum osmolarity
pressure which pulls fluid from cells and interstitial tissues into vascular space. It will reduce swelling of brain tissue
and will correct cerebral edema
PHASE 01 QUESTIONS
9) Cell membrane
A) Selectively permeable
B) Discharges particles by a process of vacuolization
C) is flexible due to presence of saturated fatty acids
D) Process Trans membrane proteins extending between cytosol and ECF
E) Consists of cholesterol molecules exposed to ECF
14) Which of the following are correctly matched regarding body fluid compartment?
A) ECF- radioactive chromium
B) ECF- Inulin
C) ICF- sucrose
D) Plasma- radioactive iodine
E) Total body water- Deuterium
SBR
46) Mitochondria
A) Are immobile organelles
B) Receives more protein from cytosol
C) Are abundant in hepatocytes
D) Are spare in muscles
E) No enzymes in its outer membrane
54) Edema is a feature in lymphatic obstruction. What is the main disturbance to starling forces in lymphatic
edema?
A) Decrease hydrostatic pressure interstitial space
B) Increase capillary hydrostatic pressure at arteriolar end
C) Increase capillary hydrostatic pressure at venular end
55) Interstitial fluid is different to plasma though both are belongs to ECF. What is the most significant feature seen
in plasma?
A) Contain more Na+ than interstitial fluid
B) Higher buffering capacity than interstitial fluid
C) Has more protein than interstitial fluid
D) Electrically more negative than interstitial fluid
E) More viscous than interstitial fluid
1) A) T
Buffer is a mixture of a weak acid with a strong base or a strong acid with a weak base.
B) F HCO3
C) T
3 important buffers in urine; bicarbonate, di-basic acid phosphate and ammonia.
D) T is the major blood buffer
E)
9)
A-T
B-T vesicular transport-active
C-F Due to the presence of unsaturated fatty acids.
D-T
E-F
10)
A-T
11)
A-F
12)
A)T
B) T
C) F Primary active
D) T
E) F 3Na+ and 2K+
13)
A) T GLUT 4
B) T Through channel proteins.
C) F Secondary active
D) T
E) F SGLT 1 – secondary active.
14)
A)F ECF – inulin
RBC volume – radioactive Cr
B)T
C)F Cannot be measured (calculated as TBW – extracellular fluid volume)
D)T Radioactive I labeled protein, Evans blue dye
E)T
15)
A)T
B) F
46)ANSWER – C
Abundant in liver, muscle and kidney
47) ANSWER – C
Provide physical binding of the epithelium to the underlying tissue and physical support.
54) ANSWER – E
In lymphatic obstruction, the filtered proteins do not enter the blood reducing the plasma oncotic pressure.
55) ANSWER – C
8) Chromatin
A) Has double helix DNA
B) Has basic proteins
C) Has acidic proteins
D) Has small amount of proteins
E) Is the basic unit of nucleosome
19) Factors contributing to negative in inner side of the cell membrane include
A) Presence of Na/K channel
B) Semi permeability of cell membrane
C) Relative permeable Na/K across leaky channels
D) Secondary active transport of Na+
E) Presence of more PO3- inside the cell
SBR
4) Functions of SER except
A) Glycoprotein production
B) Phospholipid synthesis
C) Remove phosphate from glucose 6 phosphate
D) Detoxification of harmful chemicals
E) Synthesize enzymes
9th SEQ-Repeat
1.
1.1. Briefly explain how the constituent molecules of the plasma membrane contribute to fluidity. (35 marks)
5.A 7 year-old child came with a history of diarrhea. It was found that child was moderately dehydrate and
breathing rapidly. The child was treated with Jeevani. (ORS made according to WHO formula)
5.1 State the expected acid-base disturbance and explain the mechanisms (15 marks)
5.2 Explain the Physiological basis for rapid breathing in this child? (15 marks)
5.3 What are the basis for having Na+ and glucose in ORS. (35 marks)
5.4 What are the limitations of ORS as a rehydration solution (35 marks)
3) Buffer is the mixture of weak acid and its conjugate base. Regarding weak acid
A) it’s protonated at the pH below pka value
B) Acetic acid is an example
C) It has conjugate acid
D) It completely dissociate in the water
E) It behaves according to the Henderson Hasselbalch equation
SBR
45) What is the most likely acid-base disturbance caused in a patient with insulin dependent diabetes mellitus .His
plasma PH=7.2 HCO3 - =17mmol/l Pco2 = 2-20mmhg
A) Metabolic acidosis
B) Metabolic alkalosis
C) RD) Respiratory alkalosis
E) None of the above
10th SEQ-Proper
3)
A) T
At pH values less than the pKa, the protonated acid from (CH3- COOH) is predominant.
B) T
CH3-COOH
C) F
D) F 𝐶𝐻3𝐶𝑂𝑂𝐻 ⇔ 𝐶𝐻3𝐶𝑂𝑂 − + 𝐻 +
E) T
pH = pKa + log [A~ ]/ [HA]
11)
A) T
centromere
a pair of centrioles
Centrosome matrix
B) T
RER — synthesis proteins.
OuteC) F
ribosomes consists of 2 subunits (large, small). These sub units are produced in nucleolus and exit together to
cytoplasm.
Presented by 15th Batch 143 FHCS | EUSL
D) T
microfilaments from micro villi.
E) F
in inactive state—chromatin appears as beads of nucleosomes referred as heterochromatin. For transcription
occur, DNA uncoil and uncoiled state (active) is called Euchromatin.
12)
A) T
B) F
C) T
D) T
E) F
13)
A) F
B) F
C) T
atty acids and steroid hormones.
D) F
sarcoplasmic reticula not found in liver.
E) F
takes place in the cytosol.
15)
A) F
passive (along the concentration gradient
B) F
-permeable membrane from a less concentrated solution into a
more concentrated one.
C) F
passive (along the pressure gradient)
D) T
E) T
45) ANSWER – A
Normal pH – 7.35- 7.45 HCO3 - = 2 Pco2 = 44 mm/Hg (venous) 40mm/Hg (arterial)
47) ANSWER – B
Genetic variability enhanced
- Crossover which redistributes genetic material.
- Random distribution of homologues chromosome to Daughter cells.
48) ANSWER – A
Plasma membrane is a phospholipid bilayer.so lipid soluble substance can easily diffuse through it.
2.1.
• Cell membrane is surrounded the cell from its outside.
• Thickness 5-8nm.
• Selectively permeable.
• Composed of a) Lipid phospholipids (75%) Cholesterol (20%) Glycolipid (5%) b) Protein integral protein (
transmembrane) Extrinsic protein ( peripheral) c) Carbohydrates
• Singer and Nicholson proposed the fluid mosaic model.
• Phospholipid molecules
Amphipathic
Hydrophilic
– polar head
Hydrophobic
– non polar tail
• Phospholipid arrange as a bilayer, facing their head outward.
• Fluidity and flexibility of membrane is increased by UFA.
• Cholesterol molecule among phospholipid molecules.
• They stabilize and regulate the fluidity of bilayer ( reduce)
• Transmembrane protein have a hydrophobic central zone amd hydrophilic zone.
• Peripheral protein attach to inner and outer membrane.
• Mosaic model of membrane ids due to protein.
• Protein make up almost ½ of total mass of membrane.
• Function of transmembrane protein,
I. Communication
II. II. Cell matrix adhesion
III. III. Formation of pores
• Short chain polysaccharide contact with protein and lipid molecule forming glycocalyx.
• Interphase is in a high metabolic activity and does most of its growing and synthesis of organelles.
• In M phase ( mitosis) duplicated chromosomes and organelles goes to daughter cells
• M phase is the periods ofcell division.
• M phase is consist of 4 phases.
• They are
10. Mitosis
A) Causes growth of tissues
B) Has 2 cell divisions
C) Regulated by cyclin dependent protein kinases
D) Occurs in secondary spermatocytes
E) Gives rise to 2 identical daughter cells
12. Diffusion,
A) Accounts for the greatest amount of substance transported across the cell membrane
B) Is an active transport
C) Is the process by which gases move across alveolocapillary membrane.
D) Takes place against concentration gradient.
E) Takes place due to random movement of molecules.
SBR
41. Important buffering activity is provided by
Presented by 15th Batch 146 FHCS | EUSL
A) Bicarbonate ion in blood
B) Hemoglobin
C) Ammonia in urine
D) phosphate in urine
E) plasma protein
46. Nucleus
A) disappear in prophase of meiosis 1
B) increase Heterochromatin in metabolic active cells
C) continue as SER
D) found in mature RBC
E) in osteoclast cells as single structure
50. A patient was transfused with a high volume of a solution which made his RBC lysis, this solution is mostly like?
A) Hypertonic Nacl
B) Hypertonic Urea
C) Hypotonic urea
D) Isotonic mannitol
E) Isotonic Nacl
59. Edema is a feature in lymphatic obstruction. What is the main disturbance to starling forces in lymphatic
edema?
A) Decrease hydrostatic pressure in interstitial space
B) Increase capillary hydrostatic pressure at arteriolar end
C) Increase capillary hydrostatic pressure at venular end
D) Decrease oncotic pressure in interstitial space
E) Decrease plasma oncotic pressure
11th SEQ-Proper
02. 2.1. List the types of cell surface receptors with examples. (15 Marks)
1. Ligandgated ion channels. Example- neurotransmitter receptor
2. Enzymelinkedreceptors- receptors for growth factors
3. G-proteincoupledreceptors- beta adrenergic receptor
12th Proper-MCQ
15. Mitosis,
a. Give non identical daughter cells
b. Repair damaged tissues
c. High rate in tumor cells
d. Can cause mutations
e. May cause loss of chromosome parts
20. Endocytosis
a. Active method of cell transport
b. Phagocytosis
c. Intra cellular vesicles fuse with cell membrane
Presented by 15th Batch 147 FHCS | EUSL
d. Remove cell debris
e. Intracellular substances are removed to extra cellular area
SBR
44. Which of the following is responsible for the seasonal allergic reactions such as runny nose, nasal congestion
and wheezing
a. Monocytes
b. Neutrophils
c. Mast cells
d. Lymphocytes
e. Basophils
1.1 Mechanisms by which chemical messengers (hormones or neurotransmitters) modify the cellular activities in
their target tissues
1.2 Functions of different components of eukaryotic cell membrane
1.3 Events of Meiosis occur in ovary
1.4 Defects of meiosis that cause Down syndrome
15
A.F give identical cells
B.T replace and growth tissues divide from mitotic
C.T
D.F mutation mostly occur in meosis
E.F in meosis
20)
A.T
B.T endocytosis include pinocytosis and phagocytosis
C.T
D.F from exocytosis
E.F extracellular substance take into intracellular
44. C
Mainly allergies are due to mast cells which releases allergic inducing substances.
50. B
Abundant positive ion – K+
Presented by 15th Batch 148 FHCS | EUSL
Abundant negative ion – HPO4-2
K+ is higher than HPO4-2
12. Serum is different from plasma. What is the main feature of serum?
A. it has higher osmolarity.
B. It is devoid of fibrinogen.
C. it has increased calcium concentration.
D. It has no antibodies.
E. It contains platelets.
SBR
47. Which of the following is wrong about mitosis?
A.It is regulated by the cyclin dependant protein kinases.
Presented by 15th Batch 149 FHCS | EUSL
B. It leads to formation of genetic variations.
C. It occurs in repair and replacement of celss.
D. It occurs rapidly in keratinocytes.
E. Rapid mitosis leads to occurance of mutations in daughter cells
54. Which of the following capillaries have the highest hydrastatic pressure ?
A. Pulmonary capillaries
B. Glomerular capillaries
C. capillaries in Liver
D. capillaries in Skeletal muscle
E. Capillary in intestinal villi
13th Proper-SEQ
1)
1.4. State 3 genetic importance of meiosis (15 marks)
08
A) True
Cholesterol – maintain the fluidity
B) False
Cell recognition - glycoproteins (ABO antigens in RBC membrane –contain galactose)
C) True
Integeral proteins act as- channel proteins ,carrier proteins, enzymes, Receptors
D) True
Peripheral protein functions
1.enzymes
2.control the transport of substances through cell membrane across the pores
E) True
Sphingophospholipids –ex.Sphingomyelin Myelin predominantly found in neuron cell membrane.
09
A) True
B) True
Trimeric protein – Alpha, beeta and gamma sub units
C) False
G-protein active when bind to GTP
D)
E) False In many times cAMP activates protein kinase
11
A) True
B) False
Filtrate from blood capillaries , excess interstitial fluid drain by lymphatic system.
C) False
ECF+ICF = 60% of Total body weight
D) False
Low protein . (consider as 0mmHG oncotic pressure in interstitial fluid) E True
Presented by 15th Batch 150 FHCS | EUSL
47. B Cross over in meiosis –genetic variant formation
54. B
14th Proper-Mcq
SBR
41. p53 protein is a key regulator of the progression of the cell cycle. It provides opportunity for own repair to occur
via the arrest of the following phase. which arrested for DNA repair before DNA replication by p53
A) G1 phase
B) G2 phase
C) S phase
D) Metaphase
E) Prophase
42. Which organelles have the ability to self replicate for the energy demand?
A) Centrosome
B) Lysosome
Presented by 15th Batch 151 FHCS | EUSL
C) Mitochondria
D) Peroxisome
E) Ribosome
14th Proper-SEQ
2.4. Outline the role of G protein coupled receptors on cell signaling pathway with illustration. (25 Marks)
5.
5.1. Hypoalbuminemia leads to:
5.1.1. hyponatremia
5.1.2. hypovolemia
Explain the physiological basis of the abnormalities stated in 5.1.1 and 5.1.2. following hypoalbuminemia.(50marks)
01)
AT
Different cell types express specific kinds of intermediate filaments. They are composed by different proteins. (
Nuclear lamins protein forms the intermediate filaments in Nucleus ) Specific type of proteins is used to determine
cells of origin of metastatic tumors
BT
Finger like cell extensions = microvilli → Microfilaments provide mechanical support
CF
Position of cell organelle → Intermediate filaments
D T Centrosome = Pair of centrioles + Pericentriolar matrix Composed of microtubules
E T Spindle fibers consist of microtubules synthesized from the protein Tubulin
Presented by 15th Batch 152 FHCS | EUSL
14)
AF
Endocytosis (pinocytosis, phagocytosis) and exocytosis are active transport methods
BF
CF
DT
ET
15)
AT
Na+ can transport through channels. It is passive
BF
Inside the membrane is negative and outside the membrane is positive
CF
Permeability depends on the substance. So permeability is not equal for different ions.
DT
Gradient is made from high to low concentration of ions
EF
Only move water. Water move from solution to cell.so cells swell.
36)
AT
Prevent the shortening of telomeres
BF
C F Presents at the ends of linear chromosomes
DT
ET
41) A
DNA replication is occurred in S phase. G1 is the phase before S phase
42) C
Mitochondria and Chloroplasts are two self-replicating cell organelles. Main function of mitochondria is ATP
production.
48) D
Exocytosis is an active process. It uses energy
49) D
Protein is not lipophilic. It is not diffusible. Ions transport through channels and glucose is transported through
facilitated diffusion
50) C
SBR
44) A woman has plasma osmolarity of 290mos/L & urine osmolarity of 1100mosm/L. What is the reason?
A. Central diabetes insipidus
B. Dehydration
C. Drinking large volumes of water
D. Nephrogenic diabetes insipidus
E. SIADH
45) normal cell function is dependent on folding of the cell membrane. which one of the following increases the
membrane fluidity?
A. Increased body cold stress
B. Increased integral membrane protein content
C. Increased peripheral membrane protein
D. Increased saturate phospholipid content
E. Increased unsaturated phospholipid content
46)Nucleus
A. contains nucleoproteins synthesized inside it
B. contains ribosomes
Presented by 15th Batch 154 FHCS | EUSL
C. is bounded by single layered membrane
D. is surrounded by a membrane showing the identical features of plasma membrane
E. nucleoli which produce mitotic spindles
15th Proper-SEQ
5.2. Explain the physiological basis of using oral rehydration solution (ORS) in chronic diarrheic patient (25 marks)
(06) Fibroblast,
a. Has an extensive RER & free ribosomes.
b. Release histamine & associated with allergies & inflammation.
c. Secretes proteins that become fibers in the connective tissue matrix.
d. Has the ability to provide repair after injury in connective tissue.
e. Is responsible for scar tissue formation.
(08) Bone,
a. Rich in blood supply.
b. Serve as a source of mineral ions for homeostasis.
c. For growth in thickness has growth plate.
d. Has a good regenerative potential to regenerate through out life.
e. Disorder caused by vit D deficiency leads to reduced strength.
(09) Skin,
a. Is 15% of total body weight in normal adults.
b. Is the storage organ for vitamin D.
c. Always has an acidic pH.
d. Is impermeable to drugs.
e. Epidermis is continuously replaced.
1 .Briefly describe the sequence of event at the neuromuscular junction when a motor nerve is stimulated with
reference to ,
1.1 Event at the terminal buttons (35 marks)
1.2 Nature of transmission from nerve to muscle (35 marks)
1.3 Replace of motor end plate (30 marks)
1.3 Compare and contrast the structure of skeletal and cardiac muscle(30 marks)
1.4 Describe the process of ossification (40 marks)
2.2.1 Briefly describe the importance of ‘Oxygen debt’ after exercise(20 marks)
2.2 Repeated stimuli at short intervals produces tetanus in skeletal muscles but not in cardiac muscle
2.2.1 Define the term tetanus(20 marks)
2.2.2 Briefly explain the physiological basis of the above difference
3) Membrane potentials,
a. Is an unique feature of excitable tissues
b. Development to form skeletal system
c. Induce the neural tube forming
d. Formed from mesoderm
e. Net effect of concentration gradient and electrical gradient between plasma membrane
5) Smooth muscle,
a. include tropomycine
b. single nucleated
c. contain sarcomere
d. contain satellite cells
e. Have unstable membrane potential
6) Regarding muscles,
a. Refractory period of cardiac muscles is longer than skeletal muscles
b. Cardiac muscles show fatigability
c. In cardiac muscles force of contraction is inversely proportional to the end diastolic volume
d. Skeletal muscle velocity great at resisting membrane potential
e. Membrane potential of smooth muscle is unstable
8) Endochondral ossification,
a. Ends at birth
b. Create medullary cavity
c. Can be seen in clavicle
d. Need the formation of hyaline cartilage
e. Spongy bone firstly form
10) Dermis,
a. Apocrine sweat gland involving thermogenesis
b. Contain collagen fibers
c. More cells than epidermis
d. Is damaged by 1st degree burn
e. Is vascular
11) What is the common feature of epithelial tissue & connective tissue?
a. Cells are closely packed
b. Cells lies on basement membrane
c. Have rich blood supply
d. Occur throughout the body
e. Possesses underlining intracellular materials
13) Plasma and interstitial fluid are included in extra cellular fluid. Most significant feature seen in plasma
compared to interstitial fluid,
a. Plasma contain high amount of sodium ion than interstitial fluid
b. Physostigmine drug increase the action potential of sarcomere
c. Physostigmine drug increase the acetylcholine in synaptic cleft
d. Physostigmine increase ca+2 removal from t-tubules
e. Physostigmine increase speed of impulse
14) A tough sheet of dense irregular tissue, that surround bone surface ,bone growth assist in bone repair,
nourish bone tissue and serve attachment site of tendon, which structure,
a. Epiphysis
b. Endosteum
c. Metaphysis
d. Diaphysis
e. Periosteum
1.
1.1. State the components of the Haversian system. (20 Marks)
1.2. Outline the characteristic of the epidermis of thick skin. (25 Marks)
1.3. Illustrate the structure of a sarcomere. (20 Marks)
1.4. Describe the growth of cartilage. (35 Marks)
2.
2.1. Generation an action potential is a feature of excitable tissues
2.1.1. Illustrate the action potential in a neuron using a diagram. (40 Marks)
2.1.2. Explain the events that take place during the action potential in a neuron. (40 Marks)
2.2. Muscle contraction requires enormous amounts of energy in the form of ATP.
Outline the sources of energy for the muscle contraction in the human body with examples. (20 Marks)
9) Fibro cartilage
A. Has abundant collagen type I
B. Has multiple chondrocytes in a lacunae
C. Is highly flexible
D. Lacks perichondrium
E. Provides support to the joints
12) Which of the following feature Is mostly likely responsible for transitional epithelium to prevent the leakage of
urine from bladder to underlying tissues
A. Lies on the basement membrane
Presented by 15th Batch 162 FHCS | EUSL
B. Multi-layer
C. Desmosome junctions between cells
D. Tight junctions between cells
E. Umbrella cells
1.
1.1. State the importance of tension lines in skin (15 Marks)
1.2. Outline the process of nerve regeneration (20 Marks)
1.3. Describe the growth of a long bone (35 Marks)
1.4. Compare and contrast the structure of skeletal and cardiac muscle (30 Marks)
2.
2.1. Outline the disease related to collagen synthesis (20 marks)
2.2. Define nerve impulse (20 marks)
2.3. Explain the ionic basis of propagation of nerve impulse (20 marks)
2.4. Explain briefly how receptor potential differs from action potential (20 marks)
2.5 Explain briefly how myelination increase velocity of conduction (20 marks)
Answers
01.
A. False
B. False
C. False
D. True
E. True
✓ Cellularity
E.
03.
A. F
B. F
C. T
D. T
E. T
04.
A. F
B. T
C. T
D. F
E. F
✓ Non polar
• Mutation in fibrillin 2 results Marfan’s syndrome
Collagen protein
• Long, rigid
• Three alpha polypeptides/triple stranded /triple helix
• Collagen related diseases :scurvy, ehler’s danlos syndrome
• ECM, vitreous humor in eye, tendons, cornea of eye
05.
3- causes a depolarization of the terminal button, which causes calcium channels to open.
5- Ca2+ makes the vesicles to fuse with the surface of the presynaptic cell.
*excess NT is uptaken by either the glial cells or the presynaptic cell's terminal buttons.
11- The left over shells from the vesicles are fatehed back from the presynaptic cell's plasma.
A. False (Electrically stimulated Presynaptic membrane depolarizes. It results in activation of calcium channels)
B. True
C. False (Ach is stored in vesicles. Not in the sarcoplasmic reticulum. Calcium ions stores in sarcoplasmic reticulum)
06.
A. True
B. False
C. True
D. False
E.
Electrical potential of skeletal muscle fibres
07.
A.
B.
C. True
D. False
E. True
08.
A.
B. False
C. True
D. True
E. True
*Consists of glycine and arginine
*Stored in skeletal muscle
Free ATP -
Low levels of ATP exist within the muscle fibers and can immediately provide energy for contraction. However, the
pool is very small and after a few muscle twitches will be exhausted.
Phosphocreatine -
Phosphocreatine, also known as creatine phosphate, can rapidly donate a phosphate group to ADP to form ATP and
creatine under anaerobic conditions. Enough phosphocreatine is present in the muscle to provide ATP for up to 15
seconds of contraction. The reaction of phosphocreatine + ADP to ATP + creatine is reversible. During periods of
rest, the store of phosphocreatine is regenerated from ATP.
Glycolysis -
Glycolysis is the metabolic reaction which produces two molecules of ATP through the conversion of glucose into
pyruvate, water, and NADH in the absence of oxygen. The glucose for glycolysis can be provided by the blood
supply, but is more often converted from glycogen in the muscle fibers. If glycogen stores in the muscle fibers are
expended, glucose can be created from fats and proteins. However, this conversion is not as efficient. Pyruvate is
continually processed into lactic acid. With pyruvate accumulation, the amount of lactic acid produced is also
increased. This lactic acid accumulation in the muscle tissue reduces the pH, making it more acidic and producing
the stinging feeling in muscles when exercising. This inhibits further anaerobic respiration, inducing fatigue.
Glycolysis alone can provide energy to the muscle for approximately 30 seconds, although this interval can be
Presented by 15th Batch 166 FHCS | EUSL
increased with muscle conditioning.
Cellular Respiration -
While the pyruvate generated through glycolysis can accumulate to form lactic acid, it can also be used to generate
further molecules of ATP. Mitochondria in the muscle fibers can convert pyruvate into ATP in the presence of
oxygen via the Krebs Cycle, generating an additional 30 molecules of ATP. Cellular respiration is not as rapid as the
above mechanisms; however, it is required for exercise periods longer than 30 seconds. Cellular respiration is
limited by oxygen availability, so lactic acid can still build up if pyruvate in the Krebs Cycle is insufficient. Cellular
respiration plays a key role in returning the muscles to normal after exercise, converting the excess pyruvate into
ATP and regenerating the stores of ATP, phosphocreatine, and glycogen in the muscle that are required for more
rapid contractions.
09.
A. True
B. False
C. False
D. True
E. True
Hyaline cartilage
Elastic cartilage
✓ Consists of Collagen type two
✓ Single chondrocyte in one lacunae
✓ Gives support
✓ Maintain shape
✓ Allows flexibility (elasticity)
Fibro cartilage
✓ Consists of Collagen type one
✓ Parallel rows of chondrocytes alternating with collagen fibres
✓ Highly compressible
✓ Great tensile strength
10.
A.
11. D
Epithelial type
1) Simple squamous - Diffusion, Filtration, Secretion(serosa), form Blood brain barrier
I.Ciliated - Propulsion
II.Non ciliated - Protection, Absorption, Secretion
5) Stratified squamous - Protection
6) Stratified cuboidal - Protection
7) Stratified columnar - Protection, Secretion
8) Transitional - Stretch, Permits distension of sacs
12. D
Tight junctions – Inhibit passage of substances between cells. Prevent contents of organ from leaking into blood or
surrounding tissues.
13.
A.
B.
C. False
D. False
E. False
14. A
Nerve conduction velocity (CV) is the speed at which an Allows cell electrochemical impulse propagates down a
neural pathway.
1. Myelination amount
Myelination results in soltatary propagation which is more faster than continuous propagation. Therefore velocity
increases with the amount of Myelination
2. Axon diameter
Presented by 15th Batch 168 FHCS | EUSL
Velocity increases with the increasing of axon diameter.
3. Temperature
Velocity increases with temperature.
❖ A fibres – thick myelinated
❖ B fibres – thin myelinated
❖ C fibres-unmyelinated
❖ Preganglionic fibres – myelinated
15. B/D
Factors affecting bone growth and remodelling
A. Nutrition
a) Minerals
Calcium ions /Phosphate ions – Need vitamin D calcitriol to absorb
Fluoride ions
Magnesium ions
Manganese ions
Ferric ions
b) Vitamins
Vitamin C- Collagen synthesis
Vitamin D-Absorb Calcium & Phosphate
- Increase blood calcium level
Vitamin K & B 12-Protein synthesis
Vitamin A-Stimulation of osteoblastic activity
B. Diet changes
C. Lifestyle
1.
1.1. (15)
Surgical cuts and incisions make along tension lines because they heal and repair scars quickly
1.2. (20)
Axons and dendrites are associated with neurolemma. Following the damage, Nissls granules break into granular
masses (chromatolysis). Even though the axon and myelin sheath degenerate, the neurolemma remains.
Degeneration of the distal portion of the axon and myelin sheath is called Wallerian degeneration. Following
chromatolysis, signs of recovery in the cell body become evident. Macrophages phagocytize the debris. Synthesis of
RNA and protein accelerates, which favours rebuilding/regeneration of the axon. The Schwann cells on either side
of the injured site multiply by mitosis, grow toward each other, and may form a regeneration tube across the
injured area. The tube guides growth of a new axon from the proximal area across the injured area into the distal
area previously occupied by the original axon. New axons cannot grow if the gap at the site of injury is too large or if
the gap becomes filled with collagen fibers. During the first few days following damage, buds of regenerating axons
begin to invade the tube formed by the Schwann cells. Axons from the proximal area grow at a rate of about 1.5
mm (0.06 in.) per day across the area of damage, find their way into the distal regeneration tubes, and grow toward
the distally located receptors and effectors. Thus, some sensory and motor connections are reestablished and some
functions restored. In time, the Schwann cells form a new myelin sheath.
1.3. (35)
• Long bone growth by endochondral ossification.
• First, a cartilage model of the future bone is developed by the mesenchymal cells. Cells convert to chondroblasts,
secrete cartilage ECM and perichondrium developed.
• When branches of epiphyseal artery enter epiphysis, secondary ossification centre develops outward (around
time of birth). Spongy bone remains and no medullary cavity.
• Hyaline cartilage surrounding epiphysis become articular cartilage.
Scurvy
Due to vit.C deficiency
Vit.C coenzyme for hydroxylation
Collagen synthesis impaired
Osteogenisis imperfecta
Genetic disorder
Problem in mutation of genes responsible for production of collagen type
Brittle bones
Ehlers danlos syndrome
Genetic disorder
Affect production of collagen in blood vessels,skin
2.2.
Propagation of action potential
2.3.
Due to strong stimulus Na+ leaky channels open RMP increase upto threshold due to Na+ influx Voltage gated Na+
channels open,Na+ influx-depolarization. Then K+ channels open, K+efflux -repolarization. At threshold K+ channels
start to close. Still some K+ efflux- hyperpolarization. Na+/K+ ATPase action regenerate RMP. Electrical circuits
between nearby areas lead to propagation of AP
2.4.
Action potential generate if stimulus reach firing level, receptor potential (RP) is a membrane potential AP generate
simplistically, RP is a graded potential
2.5.
In myelinated nerves impulses transduced from one Ranvier nodes to next Ranvier node. Because myelinated nerve
impulses jump over areas myelinated, they are faster than unmyelinated ones
10. A trained 400m sprinter obtain energy for the event by,
A) Anaerobic glycolysis
B) Kreb's cycle
C) Oxidation of fatty acids
D) Phosphagen system
E) Stored ATP
SBR
Presented by 15th Batch 172 FHCS | EUSL
11. Which gland secretes sticky and waterproof secretion to prevent entry of foreign particles and insects to ear
canal?
A) Apocrine gland
B) Ceruminous gland
C) Eccrine sweat gland
D) Goblet or unicellular glands
E) Sebaceous glands
12. Which of the following cells regulate the exchange of nutrients and metabolites from the neuron cell bodies in
ganglia?
A) Astrocytes
B) Ependymal cell
C) Microglial cell
D) Satellite cell
E) Schwann cell
1.
1.1. Compare and contrast the structure of both involuntary type of muscles. (30 Marks)
1.2. Outline the characteristic features of the epidermis of thin skin. (30 Marks)
1.3. Describe the process of ossification of humerus. (40 Marks)
2.
2.1. Compare and contrast the structure of collagen and elastin proteins in the extracellular matrix. (20 Marks)
2.2. Briefly describe the role of skin in
3.2.1. Thermoregulation (40 Marks)
3.2.2. Immunity
01.
A- T it’s the specialization of epithelial surface to increase surface are for the absorption.
B- T prevents free movement of substances and forms tissue blood barriers: blood brain barrier.
C- T made of single layers of cells facilitates easy diffusion.
D- T due to abrasion lost cells replaced by epithelial cells with high mitotic index.
E- F has abundant nerve supply and nerve endings.
02.
A- T unicellular glands are presents. Goblet cells.
B- T developed from surface epithelium and the connection maintains to form exocrine ducts.
C- F if having high generating ability could result in cancer.
D- F adenocarcinoma is a cancer arises from endocrine tissues.
E- F via ducts secretions is released.
03.
A- T
B- T since having mitochondria which have UCP- (uncoupler protein) results in heat generation rather than ATP
production.
C- F
D- T
E- T refer wheatear’s histology
04.
A- F cell to cell or cell to receptor.
B- F present in all cells.
C- T theory
D- T energy needed for Na/K ATPase pump to form resting membrane potential.
05.
A- F only a stimulus with intensity higher than RMP
B- F easy to record in nerves and muscles.
C- T trigger of muscle contraction.
D- F also in muscles as well.
06.
A- T loss of cartilages causes abrasion of bone by friction and arthritis
B- F blood supply by diffusion.
C- T theory
D- F fibrocartilages
E- T presence of collagen fibers provides considerable flexibility.
07.
A- F red bone marrow present in spongy bone.
B- T theory
C- F refer to the spongy bone. Refer histology of wheatear’s
D- F in single longitudinal direction.
08.
A- T theory
B- T theory
C- F
D- ?
E- ?
09.
A- T
B- F
C- T
D- F
E- F
10.
A- T
B- T
C- F
D- T
E- T
12. A is the answer. By forming blood brain barrier with perivascular end feet of astrocytes.
13. E is the answer presence of Ranvier forms myelin cover only at those locations. Nerve impulse transmitted only
at Ranvier nodes via jumping AP.
1.
1.1.
Branched cylindrical fibres with one centrally Fibres thickest in middle, tapered at each end and
located nucleus with one centrally positioned nucleus
Actin and myosin are arranged in a sarcomere Actin and myosin are not organized to a
sarcomere
Have moderately developed sarcoplasmic reticula Have poorly developed sarcoplasmic reticula and
and tranverse tubule no transverse tubules
Intercalated discs contain gap junctions and Gap junctions in viscera smooth muscle, none in
desmosomes multi-unit smooth muscle
Regular protiens - troponin and tropomyosin Regular protiens - calmodulin and myosin light
chain kinase
High Ca2+ storage and Ca2+ binds to troponin Low Ca2+ storage and Ca2+ binds to calmodulin
1.2.
The epidermis of thin skin is consisted of 4 major layers from bottom to top, Stratum basale, stratum spinosum,
Stratum granulosum and stratum corneum.
The deepest layer; stratum basale is the layer where continuous cell division occurs and make all other cell layers.
Stratum spinosum includes 8-10 layers of keratinocytes which includes keratohyalin. Stratum granulosum includes
lemellar granules which are rich in glycolipids. Stratum corneum is composed of many sublayers of flat, dead
keratinocytes that are continuously shed and replaced by cells from deeper strata. (Stratum spinosum + stratum
basale --> stratum germinativum). Stratum spinosum --> in the process of protein synthesis.
1.3.
The process of bone formation is reffered as ossification. Bone formation occurs in 4 situations, bone formation in
an embryo, growth of bones until adulthood, remodeling of bone, repaire of fractures. There are 2 types of
ossification, they are membranousossification and endochondrial ossification. Long bones as humerus follow the
endochondrial ossification. It uses hyaline cartilage models and thus requires breakdown of hyaline cartilage prior
to ossification.
Cartilage template is formed with the shape the future bone. Primarily offification centers appear in the shaft or
diaphysis. Thin bone collar appears around the diaphysis by intramembranous ossification. So, that the
perichondrium of cartilage become periosteum. Deep to newly formed collar cartilage matrix begins to calcify.
Chondrocytes hypertrophy and die. Blood vessels invade the diaphysis interior and bring in associated
mesenchymal and osteoprogenitor cells. Then the erosich of cartilage in the centre is occured and primitive marrow
cavity or medullary cavity is formed. Secondary ossification centres appear. And subsequently articular cartilage
and the epiphyseal plate is formed. Interstitial proliferation of remaining chondrocytes cause the two ends to grow
longitudinally. Condrocytes are arranged in columns on both sides of central region. Thus forms the epiphyseal
plate at junction between epiphysis and diaphysis.
At the primary ossification centre, spongy bones are forms,other osteoblasts below the periosteum makes compact
bone. Sondary occifications centres appear as diaphysis. The growth of long bones in length involves 2 major
events. They are the growth of cartilage on the epiphyseal plate and replacement of cartilage by bone tissue in
epiphyseal plate. Until the epiphyseal plate is there the growth of long bones continues. But after puberty at a
certain age they dissapear and replaced by bone tissue. Then the lengthening stops. Epiphyseal plate cartilage
organises into 5 functional zones.
- zone of resting- Small clusters of flattened, rounded and randomly arranged, quiescend chondrocytes.
- zone of proliferation - Chondrocytes undergo rapid mitosis under influence of growth hormone.
- zone of hypertrophy - Miroses ceases and cells and their lacunae enlarge.
- Richly vascularized primary marrow extends into the newly opened spaces and osteoblasts
differentiate from mesenchymal cells in the marrow.
2.
2.1.
Collagen Elastin
Main structural protien found in skin and other Major structural protien in the connective tissue
connective tissue. of elastic structures.
Long, stiff, triple stranded, helical structure Rubber like structure, highly hydrophobic, not
glycosylated.
3 collagen polypeptide chains (alpha chains) are Contain little hydroxyproline and hydroxylysine.
wound around one another in a roperike super Insoluble protien.
helix by inter chain H bonds.
Extremely rich in proline and glycine. Hydroxy Composed of 2 large types as hydrophobic
proline is present. Hydroxylysin is absent segments and alanine and lysine rich alpha-
helical segments
2.2.
2.1.1.
Skin plays a major role in thermoregulation. Normal body temperature of a healthy individual is 37°C (369°C). When
the body temperature is varied than than the body metabolic processes alter their action. So, It is very important to
keep the temperature in their proper range. Skin is playing important role there. When the body is 37°C, it should
be increased by heat gain. Shivering, releasing of epinephrine and norephrine and vasoconstriction of cutaneous
blood vessels make it happen. When the body is exposed to a cold environment, heat loss from the body should be
decreased cutaneous vasoconstriction decreases cutaneous blood circulation and decreased the heat loss.
Goosebumps in a cold weather occurs due to constriction of arrector pili muscle. That also create some heat by
muscle action but it is very small to consider. But the erection of hair In the skin makes a heat protecting shield over
the skin by the air trapped among the hair shafts. When the body temperature is increased, it should get back to
normal level. It is normal to have increased body temperature during excercises as the muscle action is increased
during exercises. But apart from exercising having high temperature is pathological. To loss the heat Skin has many
mechanisms. Evaporation of sweat, radiation, conduction and convection, cutaneous vasodialation etc. Sweet
glands secretes increased amount of sweat when there is increased body temperature. By the increased heat sweat
Presented by 15th Batch 178 FHCS | EUSL
on the skin surface is evaporated and heat is loss by that action. Cutaneous vasodilation increased the cutaneous
blood supply and make more blood to come near the surface to loss heat by conduction and convection. Radiation
from skin also can loss heat. Receptors for temperature changes are located in the skin mainly. Warmth and cold
receptors from skin takes signals to heat loss centre and heat gain centre in hypothalamus.
When the skin receptors are stimulated by increased heat, preoptic and anterior hypothalamic nuclei are
Stimulated and it inhibit sympathetic nervous system to decrease heat production and increase heat loss via
posterior hypothalamus by the actions mentioned before. When the cold receptors in the skin are stimulated by
decreased body temperature, preoptic and anterior hypothalamic nuclei are inhibited and posterior hypothalamus
activates sympathetic nervous system to activate heat gain mechanisms of skin.
2.2.2.
If any foreign substance to enter the body, it should first contact with the skin. So, the skin plays a major role as a
first line barrier for pathogens to enter. It does the 1st line defense by being a barrier to the pathogens to enter.
Skin takes out innate immunity. Skin has many layers. Top most layer is dead cells and keratin. They are tightly
packed. They are continually shedding. It prevents body fluids to leak through the skin. So that makes a mechanical
barrier along with the flushing of sweat to get away from pathogens. Sweat glands and sebaceous glands also helps
to make the skin moist to prevent it from mechanical attacks and pathogens. Skin has sweat by sweat glands and
sebum by sebaceous glands to work as chemical barriers. Sebum contains of fatty acid and it lowers the pH to
inhibit fungi and some bacteria. Sweat by sweat glands contain lysozyme and dermicidin lysozyme attacks
peptidoglycans which make the bacterial cell membrane. If any how the pathogens enter the skin, 2nd line defence
is taken place by cellular barriers to kill the microbes. Dendritic cells in the skin are phagocytic and intergrated
among epidermal cells. Langerhan cells present in epidermis are a unique populatation of tissue resident
macrophages that form a network of cells across the epidermis, but which have the ability to migrate from the
epidermis to draining lymph nodes. Their location in skin barrier also help to conduct immune functions. After
microbe is infected, inflammatory responses are occurs. The inflammatory responses are also give signals to that an
inflammation has started due to infection. Redness, pain, heat and swelling are almost always can be observed
from the skin. So, it is important to take steps to prevent further infection.
*Skin
-physical barrier
01) The velocity of conduction of action potential along a nerve is determined by,
A. Body temperature
B. Diameter
C. Length of nerve fiber
D. Myelination of axon
Presented by 15th Batch 179 FHCS | EUSL
E. Stimulation of Na+, K+ pump
05) Following is/are an inhibitory neurotransmitter in the central nervous system (CNS)?
A. Glycine
B. Histamine
C. Norepinephrine
D. Serotonin
E. Aminobutyric acid (GABA)
7) Regarding Elastin,
A. During biosynthesis, cross linking takes place via glycine
B. It is highly hydrophobic protein
C. It is protected by elastase enzymes
D. It is a dominant extra cellular matrix protein in arteries
E. Undergoes hydroxylation reaction as post translational modification
08) Which one of the following matches correctly with the function?
A. Ciliated columnar - Transportation
B. Simple cuboidal - secretion
C. Simple squamous - Protection
D. Stratified Cuboidal - Secretion
E. Transitional – Stretch
SBR
10) Neurogila cells its function correct Mach,
A. Astrocyte - Absorption of neurotransmitters
B. Ependymal cells - Secrete of celebrospinal fluid
C. Microglia cells – Phagocytosis
D. Oligodendrocytes - Facilitate the rapid nerve conducting
E. Satellite cells - Myelin sheath formation
11) The correct temporal sequence for event of the neuromuscular junction is?
A. Action potential in motor neuron, depolarized muscle end plate, uptake of Ca2+ in to presynaptic nerve terminal
B. Release of acetylcholine, action potential in the motor neuron, action potential in the muscle.
C. Release of acetylcholine, action potential in the muscle end plate, action potential in the muscle.
D. Uptake of Ca2+ into motor end plate, action potential in the motor end plate, action potential in the muscle.
E. Uptake of Ca2+ into the presynaptic terminal, release of Ach, repolarization of muscle end plate.
12) Repeated stimulation of skeletal muscle fiber cause a sustained contraction (tetanus) accumulation of which
solute in intracellular fluid is responsible for the tetanus.
A. ATP
B. Ca2+
C. Na+
D. Tropomyosin
E. Troponin
13) Which is the wrong pair regarding the effects of the hormone during the bone remodeling and growth?
A. Parathyroid hormone - Increasing bone resorption
B. Gluco corticoids - Stimulates osteoclast activity
C. GH hormone - Stimulates protein synthesis
D. Insulin like growth factor - Promote cell division in the epiphyseal plate
E. Oestrogen - Increase osteoblast activity
14) A location where you find the fibro cartilage would be?
A. Intervertebral disc
B. Nasal septum
C. Tracheal rings
D. Ear canal
E. Epiglottis
15) A cross sectional view of a skeletal muscle fiber through the H-zone would reveal the presence of,
A. Actin
B. Actin and troponin
C. Dystrophin
D. Myosin
1.
1.1.Explain the ionic basis of action potential on a neuron. (30 marks)
1.2. 5-Year old woman diagnosed with myasthenia gravis has been found with increased muscle stregth when she
is treated with an acetylcholinesterase (AChE) inhibitor. Explain the physiological basis of the above condition. (40
marks)
1.3. Outline the biochemical basis of
1.3.1. Muscle cramp occurs during sprinting (15 marks)
1.3.2. Ontogenesis imperfect (15 marks)
Answers
01)
1.Amount of myelination
AP propagate more rapidly along myelinated axons than along unmyelinated axons.
2.Axon diameter
Larger diameter axons propagate action potentials faster than smaller ones.
Larger – diameter axons have larger surface area, short absolute refractory period of about
0.4msec(1000impulses/s)
Small – diameter axons have absolute refractory period as long as 4msec(250 impulses/s)
3. Temperature
4. Pressure
5. Hypoxia/ Anesthesia
A) T
B) T
C) F
D) T
E) F
•Opening of K+ channels
A) F
B) F
C) T
D) F
E) T
03)
B) T
C)
D)
E) F
04)
Tunica media of blood vessels has both visceral and multi-unit smooth muscle types.
C) F unlike unitary smooth muscle, multi unit smooth muscle is non syncytial and contractions do not spread
widely through it.
D) T
E) T
05)
A) T
B) F
C)
D) T
E) T
06)
A) T
C) F
D) T
E) T
07)
B) T
D) T
E) T
08)
Location: Glands and ducts (pancreas & salivary), kidney tubules, covers ovaries
Functions : • Absorption, and filtration • Not effective protection – single layer of cells.
Location : Walls of capillaries, air sacs in lungs, Form serous membranes in body cavity
Function: Protection
Location: largest ducts of salivary gland, mammary gland and sweat gland
E) T Transitional epithelium
Function: • Stretches readily and permits distension of urinary organ by contained urine.
• acts as a permeability barrier and protects tissues from noxious effects of urine Location: Lines the
ureters, urinary bladder, and part of the urethra.
B) T
D) – in skeletal muscle
• Satellite cells
E) T
10) Astrocytes
Functions:
• Provide the appropriate chemical environment for the generation of nerve impulses
Ependymal cell
• metabolite exchange between CSF and extracellular spaces of the brain and spinal cord
• Choroid plexus
Microglia
Oligodendrocytes
11)
Neuromuscular Transmission
-Acetylcholine diffuse through synaptic cleft and bind with cholinergic receptors on motor end plate
-End plate potential reach to firing level(Generation of action potential in muscle fiber)
12) B
• Pubic symphysis
Hyaline cartilage
• Epiphyseal plates
Elastic cartilage
• external ear
auditory tubes
epiglottis
15) D
1.1.
A. Depolarization
( Na+ INFLUX
B. Repolarization
( Resistance for Na+ influx increases as membrane potential becomes more positive, reversing the electrical
gradient for Na+ .
( When MP rises from -70mV to 0mV, voltage gated K+ channels open. – They are slow to open and slow to close.
( After depolarization, K+ voltage gated channels close via a negative feedback mechanism
C. Hyperpolarization
( RMP is restored. But a large amount of Na+ remains within the cell & a large amount of K+ has escaped.
What is AP?
Threshold stimulation
- Myasthenia Gravis results by formation of circulating antibodies against muscle type of Nicotine cholinergic
receptors.
- These antibodies destroy some of the receptors & bind others to neighboring receptors triggering their removal by
endocytosis.
- In a normal person repetitive stimulation at neuromuscular junction causes decline in number of acetyl choline
molecules.
- Additional receptors are translocated to post synaptic membrane to up regulate the receptors.
- In myasthenia gravis Ig G type antibodies are produced which can bind with ACh receptors.
Cholinesterase catalyse the hydrolysis of the neurotransmitter acetylcholine into choline and acetic acid.
Compensate for the normal decline in released neurotransmitters during repeated stimulation.
1.3.
1.3.1.
Muscle contraction requires huge amounts of energy in the form of ATP. The amount of ATP stored in the cell only
last for 5 seconds. The rest of the ATP must be regenerated using one of three pathways.
Creatine phosphate
Anaerobic glycolysis
Aerobic respiration
Creatine phosphate provides energy up to 10 seconds. After that muscles get energy from anaerobic glycolysis.
Anaerobic glycolysis produces 2 ATP and lactic acid as a waste product. Accumulation of lactic acid causes muscle
fatigue and cramp.
1.3.2.
• Most common mutation causes the replacement of glycine with bulky side chain (prevent triple helix structure)
• Type 1 most common; mild bone fragility, hearing loss and blue sclera
Presented by 15th Batch 190 FHCS | EUSL
• Type II ; most severe form, lethal in perinatal period
02. Which cells of supporting tissue are responsible for secreting extra cellular matrix
A. Adipocytes
B. Chondrocytes
C. Fibroblast
D. Myofibroblast
E. Osteoclast
SBR
11. Which of the following sports, training program for increasing maximal muscle glycogenolytic enzyme activity
would be less important?
A. 200m track
B. 800m track
C. Basketball
D. High jump
E. Swimming
12. At node of Ranvier of myelinated axon, voltage gated Na+ channels are inactive, voltage gated K+ channels are
active and the membrane potential is in fall. What phase of neural action potential is occurring here?
A.
B. Depolarization
C. Hyperpolarization
D. Repolarization
E. Resting potential
13. O
A. Decreases the action potential velocity
B. Depolarize the neuron
C.
14. A tough sheath of dense irregular connective tissue the surround the bone surface, function bone growth, assist
in fracture repair, help to nourish the bone tissue and serve as an attachment site for tendons, what is the most
possible structure mentioned in the above statement?
A. Diaphysis
B. Endosteum
C. Epiphysis
D. Metaphysis
E. Periosteum
1.
1.1. A 42-year-old woman presents with the history of episodes of weakness in limbs. MRI reveals multiple sclerosis,
a demyelinating disease damages the myelin sheath of the nerve cells in the brain and spinal cord.
Explain how the demyelination causes muscle weakness in the above condition. (30 marks)
1.2. Cardiac muscles functions as syncytium. Explain the phenomenon in relates to synaptic transmission (15 marks)
1.3. Briefly describe the factors affecting bone growth and remodeling. (15 marks)
1.4. List all the cells that are found in the bone matrix. (10 marks)
1.5. State one function of each of the cells mentioned in 1.4 (15 marks)
Answers
01.
A- T
B- F
C- T
D- T
02.
A- T(mainly collagen)
B- T
Presented by 15th Batch 193 FHCS | EUSL
C- T
E- F
03.
A- T
C- T(facilitates the transfer of metabolites to fuel neurons and support the integrity of myelinated axons)
D- T
04.
A- T
B- T
C- T
D- F(cardiac muscle cells have long refractory period,so no summation or tetanus seen in cardiac muscles)
E- T
05.
C- T
D- F
E- F
06.
A- F
C- T
D- T
E- T
07. ?
A-
B- F (sodium channels)
C-
D-
E-
08.
Collagen
❖ Types;
➢ Type 1 – skin, bone, tendon, blood vessels, cornea
➢ Type 2 – cartilage, intervertebral disc mineralized
➢ Type 3 – blood vessels, foetal skin
➢ Type 4 – basement membrane
B- F
C- T
D-
Presented by 15th Batch 196 FHCS | EUSL
E- T
09.
B- F
C- T (in the proliferative stage, the osteoblast synthesis a type 1 collagen-rich matrix, termed osteoid, to support
ECM formation)
D- T (germinal layer- contains chondrocytes in a relatively quiet state that will serve as the source of cells for
growth)
E- F
10.
Phosphagen system
o Creatine phosphate (CP) is stored in muscles
o CP contsins a high energy bond.
o Bond is cleaved reversibly to reconstitute ATP.
o CK has different isoforms in different muscle types and it is useful as a cardiac biomarker.
A- T (glycine+arginine+methionine)
C- T
D- T ( serum creatinine levels are influence by muscle mass. And creatinine is a derivative of creatinine phosphate)
E-
12. D
13. ?
14. E
15. C
1.
1.1.
Demyelination interrupts the flow of nerve impulses in the spinal cord or (less frequently) in the brain that keep a
muscle or group of muscles functioning properly. As a result, muscles lose strength.
Myelin allows nerve fibers to transmit signals very rapidly (40-60 meters/second). Loss or damage to myelin can
cause slowing or blockage of the nerve signals and can lead to loss of nerve fibers. This causes weakness, paralysis
and/or impairment in motor function, especially of the arms and legs.
Presented by 15th Batch 198 FHCS | EUSL
1.2.
Synaptic transmission is the biological process by which a neuron communicate with the target cell across a
synapse. Chemical synaptic transmission involves the release of a neurotransmitter from the pre-synaptic neuron,
and neurotransmitter binding to specific post-synaptic receptors. Electrical synapse transmission involves the
transfer of electrical signals through gab junctions. Cardiac muscle act as a syncytium through gab junctions. They
conduct signals by electrical synapse transmission.
1.3.
♠Nutrition
*Calcium (Ca2+)
*Phosphate (PO43-)
*Fluoride (F-)
*Magnesium (Mg2+)
*Iron (Fe3+)
*Manganese (Mn2+)
2. Vitamins
*Vitamin C
*Vitamin D
♠Hormones
1. GH
5. Insulin
6. Parathyroid hormone
7. Glucocorticoids
8. Calcitriol
♠Exercise
♠Life style
♠Changes in diet
1.4.
1.5.
17) Collagen
A) Is synthesized by fibroblasts
B) Rich in glycine and proline
C) Type ІІІ contributes to form the basement membrane
D) Type ІV is hyaline
E) Has 4 types
18) Osteoblast
A) Involves in bone formation
B) Stimulated by calcitonin
C) Secrete inorganic calcitonin
D) Multinucleated
E) Concern with bone resorption
24) Conducting system for contraction of muscle fiber A) Motor end plates innervate two to three muscle fibers
B) The neurotransmitter of somatic neuromuscular is acetylcholine
Presented by 15th Batch 201 FHCS | EUSL
C) Each myofibril is surrounded by sarcoplasmic reticulum and T tubules into it
D) Depolarization release Ca2+ from sarcoplasm into the sarcoplasm surrounding myofilaments
E) Ca2+ activate the sliding filament mechanism of myofibril
45) Adipose tissue has the following features EXCEPT? A) Has sympathetic nerve endings
B) Bear receptors for insulin
C) Act as an insulator, by serving as a heat generator
D) Has cushioning effect in soles and palms
E) Has the capacity for regeneration than hypertrophy in mature state
56) Tetani is seen in skeletal muscles but not in cardiac muscles) Best explanation for that is
A) Mechanical event on cardiac muscle is almost complete in refractory period
B) Refractory period of action potential in cardiac muscles longer than skeletal Muscles
C) Slow opening Ca2+ channels in action potential of cardiac muscles
D) There is a plateau phase in cardiac muscle
E) Tetani of cardiac muscles will lead to death
16.
B) F
D) F
E) F
17.
A) T
B) T
D) F - Type II
E) F - about 25
18.
A) T
B) T
D) F - Single nucleus
E) F- Bone mineralization
19.
A) F - Osteoclast
B. T
C. T
D. T
E. T
20.
A) F - Central nucleus
B) F - Vertical
C) F - Single nucleus
E) T
22.
A) T
Presented by 15th Batch 203 FHCS | EUSL
B) F - Nissle’s granules are RER
C) T
D) T
24.
A) T
B) T
C) T
D) T
E) T
25.
B) F - Regular patterns
D) T
E) T
44) E
substantia nigra
45) E
48) B
Answers
3.
3.1
Presented by 15th Batch 204 FHCS | EUSL
located in myocardium of heart.
28) Bone
A) Rich in blood supply
B) Serve as a source of mineral ions for homeostasis
C) Thickness has growth plate
D) Has a good regenerative potential to regenerate throughout life
E) Disorder caused by vit D deficiency leads to reduced strength
52) Lamella Granules are rich in glycolipid & released by keratinocyte from a barrier to matrix. Which are mostly
epidermis?
A) Stratum Corneum
B) Stratum Lucidum
C) Stratum Granulosum
D) Stratum Spinosum
Answers
A) T
B) F - Epithelial tissues
D) T- Bones tissue
E) T - Blood
14)
A) F
B) T
C) T
D) F - In GAGs
E) T
15)
A) T
Microglia Cells
Neuroglia cells are supporting cells in
Astrocytes
Oligodendrocytes
Microglia
Ependymal
Schwann cells
Satelite cell
B) T - Astrocytes
E) T
16)
A)F
B) F
C) T
D) F
E) T
17)
B) T
C) T
D) T
E) T
23)
A) F
It is in a polarized state
B) T
But after depolarization stage even a sub threshold 2nd stimuli can elicit a 2nd action potential (supernormal
period)
C) T
D) T
E) F
D) T -Purplish color
25)
A) T- Sympathetic cholinergic
B) T
E) T
26)
B) T
C) T
D) T
E) T
27)
A) T
B) T
fibers.
E) T
28)
C) F
D) F
35)
A) T
reticulum.
B) T
D) T
E) T - Has enlarged terminal cisterns in close contact with the T system at the junction between the A and I bands.
52) C
01.
a.
1.2 Write a brief account on microscopic feature of skeletal muscle fiber (35 marks)
b.
Answers
1.
a)
1.1
Presented by 15th Batch 212 FHCS | EUSL
Small model of the long bone is formed by solid hyaline cartilage.
1.2
2. Troponin
3. Tropomyosin
1. Myosin
globular.
-binding
sites.
b.
1.3
to neighboring receptors triggering their removal by
endocytosis.
junction causes decline in number of acetyl choline molecules.
membrane to up regulate the receptors.
can bind with ACh receptors.
Presented by 15th Batch 214 FHCS | EUSL
ut no up regulation of receptors.
( Ach esterase break down Ach into acetyl & choline)
evel in neuromuscular junction (NMJ)
NMJ is enough to normal function.
1.4
efflux
the cell
electronegative
ent
energy
-) charge in the cell
gradient for K+ increases.
gradient (Normally K+ concentration in intracellular fluid >
extracellular fluid)
become more negative
reach the threshold level easily.
increased.
10) Collagen
A) Synthesized in fibroblast
B) Has triple helix structure
C) Rich in lysine
D) May be enzymatically glycosylated
E) Synthesis is affected by ascorbic acid deficiency
16) Skin
A) Is 15% of total body weight in normal adults
B) Is the storage organ for vitamin D
C) Always has an acidic PH
D) is impermeable to drugs
E) Epidermis is continuously replaced
18) Fibroblast
A) Has an extensive RER & free ribosomes
B) Release histamine & associated with allergies & inflammation
C) Secretes proteins that become fibers in the connective tissue matrix
D) Has the ability to provide repair after injury in connective tissue
E) Is responsible for scar tissue formation
25) Myelination
A) Found in neurons inside the CNS
B) Complete at end of the fetal life
C) Swan cells involved in myelination
D) It slow down the nerve conduction velocity
E) Nerve conduction velocity inversely proportional to diameter of the
cell
50) A tough layer of dense irregular connective tissue that surround the bone surface functioning in bone growth
assist in fracture repair nourish, attachment for tendons, what's the structure
A) Periosteum
B) Diaphysis
C) Epiphysis
D) Endosteum
E) Metaphysis
10)
A) T -
enzymatically glycosylated.
14)
D) T
16)
BW)
soluble substances
17)
E) T- cuboidal—secretion
and inflammation
19)
capillaries.
D) F-
E) T - functions of lymph,
tissues
Recirculation of lymphocytes
Exposure of antigens to the immune system
Formation of lymphocytes
Absorption and transport of fatty acids and fats as
chylomicrons from digestive system
20)
B) T -
D) F -
E) T -
blood
Ca2+ homeostasis
21)
C) T - endochondrial ossification -
A) T
B) T
C) F
D) F
E) F
23)
A) T -
B) F -
muscle.
stimulus.
E)
24)
cardiac muscles.
25)
A) T -
maturity.
C) T
E) F- diameter α velocity.
35)
A) T
C) T- bone density
D) F
E) T
49)D
50)A
51)C
52) E
3.1 Briefly describe the structure at the neuro-muscular junction. (40 marks)
3.2 Briefly describe the sequence of changes during excitation – contraction coupling. (60 marks)
Answers
2.1
Cell cycle
Endochondral ossification
Epithelium of GI tract
The epithelial lining, covering the oral cavity divided into 2 types.
Non keratinized stratified squamous epithelium covers the inner lining
of lips, cheeks, soft palate, floor of mouth and undersurface of the
tounge.
Rest of the tongue, gingivae and hard palate is covered by keratinized
stratified squamous epithelium.
Oesophagus is lined by non keratinized stratified squamous epithelium.
In stomach-oesophageal junction the abrupt change from non
keratinized stratified squamous epithelium of oesophagus to simple
columnar epithelium of stomach.
Duodenum, jejunum, ileum of small intestine is lined by simple
columnar eoithelium covering the villi.
Glands are lined by columnar cells, paneth cells and enteroendocrine
cells and the crypt of liberkuhn can be seen.
Large intestine consist of ceacum, appendix, ascending, transverse,
descending, sigmoid colons, rectum and anal canal.
Except middle and lower parts of anal canal , the other parts are lined
by simple columnar epithelium.
Villi and plicae circularis are absent in mucosa and consist of straight
interstinal glands (crypts),goblet cells and paneth cells are only present
in ceacum and appendix.
Upper part of anal canal consist of simple columnar epithelium, middle
part of tha anal canal consist of stratified squamous non keratinized
epithelium and lower part covered by true skin.
3.
3.1
membrane enclosed
sarcolemma.
3.2
contraction.
3. Opening of these channels allow flux of Ca2+ ions inside the neuron.
7. Ach bind with ligand gated ion channel coupled receptors specific
into muscle
fibers occurs)
10. Spreads throughout the membrane of the fiber and within the T
Presented by 15th Batch 226 FHCS | EUSL
tubules of the muscle fibers deep inside the fiber.
complex.
elicit contractions.
binding state.
7. The myosin then hydrolyses the ATP and uses the energy to move
16. Nerve
A) Has blood nerve barrier
B) Covered by connective tissue layer
C) Has regeneration capability in CNS
D) Collection of axons of neurons
E) Supported by satellite cells
Answer
15.
A. True
B .True
E -True
16 )
D .True
E .True
17)
A.
E.
18)
A.
B.
C.
D. True
19)
A. True
B.
C.
D. False - RMP -20 to -65mV
Muscle
21)
A. True
B. False
C. False - No O2 require
D. True
E. False - Give energy only for 8 to 10 seconds
48)
49)
B.
3.1 Explain the term ‘tension lines and state their importance (15 marks)
3.2 Compare and contrast the structure and functions of involuntary types of muscles.(30 marks)
Answers
2.2
Factors affecting bone remodelling are nutrition, hormones, exercise and lifestyle.
• Also need many vitamins for regulation of the process. Vit C to synthesize collagen,
Vit D ; calcitriol helps in absorption of calcium and phosphate ions and Vit A
• Hormones also play a great role; Growth hormone stimulates protein synthesis and
cell growth, sex hormones responsible for increased osteoblastic activity and
synthesis of bone ECM. Thyroid and Insulin also increase bone formation and
• According to exercise, bone adjusts its strength in proportion to the degree of bone
3.1
• Incision made along the skin tension lines heal with a minimum scarring. They result
Incision made across them can gap and increase the time needed for healing and Tension lines are due to
arrangement of collagen fibres in the dermis.
• Incision made along the skin tension lines heal with a minimum scarring. They result
Incision made across them can gap and increase the time needed for healing and resulting increased scar tissue
formation.
3.2
3.3
45. Which of the following cell regulate the exchange of nutrient metabolite from the neuron cell bodies in dorsal
root ganglia
a. Astrocyte
b. Ependymal cells
c. Microglia
d. Satellite cells
e. Schwann cells
3.2.
3.2.L. Define the term "motor unit". (10 marks)
3.2.2. When a single stimulus is applied to a nerve bundle it records several action potentials
(compound action potential) at different intervals. Explain the physiological basis of the
compound action potential in response to a single stimulus. (40 marks)
Answers
16.
A. T
the wall of vagina consists of
- a mucosal layer lined by stratified squamous epithelium
- a layer of smooth muscle
- an outer adventitial layer.
C.F present in sites what need mechanical abrasion, stress and tissue protect
D.T Basal cells are active in mitosis and produce the cells of the more superficial layers
E.F Basal cells are cuboidal or columnar. Surface cells are flattened
17.
A.T Nails are composed of hard, keratinized epidermal cells located over the dorsal
surfaces of the ends of fingers and toes
B.T
D.T width, thickness pf nail plate is determined by the size, length and thickness of the
matrix
E.T
Each nail consists of:
• free edge
• transparent nail body (plate) with a whitish lunula at its base
• nail root embedded in a fold of skin
• nail matrix
18.
A.F white is more abundant
B.T
C.T
45. D
Astrocytes – Support CNS & BBB
Ependymal – Like epithelia
Microglia – Immunity
Satellite – Nutrient exchange & Protection
Schwann – Myelination
48. B
Cardiac muscle behaves as a functional syncytium due to presence of intercalated discs,
although it is composed of individual cells.
51. D
Prolong refractory period prevent the tetanus formation of cardiac muscle
52. E
Presented by 15th Batch 235 FHCS | EUSL
Skeletal muscle contract voluntary ...but smooth and cardiac muscles are involuntary
A,B,D are not only present in the skeletal muscle
C is a feature of cardiac muscle
57. B
58. C
A. desmosome contain Cadherine protein
B. abundant
C. hemidesmosome present in skin where tend to friction
D. in invertebrate
E. transmembrane channels present
Femur is a long bone. It grow in thickness by appositional growth and lengthen by the
addition of bone material on the diaphysial side of the epiphyseal growth by Interstitial
growth.
Interstitial growth
Epiphyseal cartilage (close to the epiphysis) of the epiphyseal plate divides to create more
cartilage, while the diaphysial cartilage (close to the diaphysis) of the epiphyseal plate is
transformed into bone. This increases the length of the shaft.
Growth in length continues until puberty under the influence of growth hormone, thyroid
hormone, parathyroid hormone, and androgens.
• Zone of proliferation
-chondrocytes undergo rapid mitosis under influence of growth hormone.
-daughter cells are stacked into columns, resembling stacks of coins, and are parallel to the
long axis of the future bone.
• Zone of hypertrophy
Presented by 15th Batch 236 FHCS | EUSL
-mitosis ceases, and cells and their lacunae enlarge.
-Mature chondrocytes arranged in columns.
• Zone of calcification
-Dead chondrocytes are resorbed; lacunae erode.
-Richly vascularized primary marrow extends into the newly opened spaces, and osteoblasts
differentiate from mesenchymal cells in the marrow.
• Zone of ossification
-Osteoblasts secrete osteoid, which becomes mineralized.
-Forms new diaphysis.
Appositional growth
Increase in thickness and remodeling of all bones by osteoblasts and osteoclasts on bone
surfaces.
Osteoblasts beneath the periosteum secrete bone matrix on the external surface of the bone.
This obviously makes the bone thicker. At the same time, osteoclasts on the endosteum
break down bone and thus widen the medullary cavity. This results in an increase in shaft
diameter even though the actual amount of bone in the shaft is relatively unchanged.
1.4 Compare and contrast skeletal muscle and smooth muscle (20 marks)
Cells with obvious striations, due to organization of Lack striations, myofibril meshwork anchored to
myofibrils dense bodies
Single motor neuron and all the muscle fibers innervated by it is called motor unit. It is the
functional unit of muscle
3.2.2 When a single stimulus is applied to a nerve bundle it record several action potentials (
compound action potential) at different intervals. Explain the physiological basis of the
compound action potential in response to a single stimulus (40 marks)
Action potential recorded from a group of nerve fibers (e.g. sciatic nerves) or a nerve trunk
is called compound action potential as it is the summated action potentials of different types
of nerve fibers having different conduction velocities. The multi-peaked shape of the
compound action potential is due to the activity of the different nerve fibers of varying
conduction velocity. Most nerves are composed of myelinated nerve fibers of various
diameter and also unmyelinated fibers of quite large number.
It is conventionally split into three waves called A, B, C. The A wave itself is divided into
alpha, beta, gamma and delta sections.
The A wave represents activity in myelinated axons and the C wave that in unmyelinated
axons. The B wave may represent both types. The reason for the spreading out of the waves
lies in the differing conduction velocities of the fibers. If the recording electrode is close to
the stimulating electrode, only a single wave can be detected. It is possible to activate
selectively the axons of different sizes. With small shocks, only the wave is apparent. As the
Presented by 15th Batch 238 FHCS | EUSL
shock strength is increased, so other components of the A wave appear, followed by the
Band C waves. The largest, fastest-conducting fibres thus have the lowest threshold to
stimulations. If a maximum shock (one which fires all the fibers) is used, and pressure is
carefully applied between the stimulating and recording electrodes, another phenomenon
becomes apparent. The pressure blocks, conduction in the largest fibers first. The alpha
wave is the first to go followed by beta, gamma, delta and B until only the C wave is left.]*
49. Which of the following cell junctions allow the passage of small molecules between adjacent cells to coordinate
and synchronize cardiac muscle contraction
A..Adherens junction
B. Desmosome
C. Gap junction
D..Hemidesmosome
E. .Tight junction
1.
1.1. Compare the microscopic features of the type of cartilage present in costal cartilage and
intervertebral disc. (25 marks)
1.2. Briefly describe the structural arrangement of a sarcomere in smooth muscle (35 marks)
1.3. Describe how the meiosis differs in male and female (25 marks)
1.4. State 3 genetic importance of meiosis (15 marks)
Explain the physiological basis of the above changes after denervation. (2 * 25 marks)
Answers
15.
A. F
B. T
Contains more mitochondria
C. F
D. T
Fat storage
E. T
16.
A. F
Spongy bone
B. T
C. F
E. F
It is the function of spongy bone
17.
A. T
B. T
increase
C. T
D. F
Oligodendrocytes in CNS
Shwann cells in PNS
E. T
oligodendrocyte can myelinate up to 60 axons
19.
A. T
Need repeated stimulation. But seems like a single stimuli
B. T
C. F
No merging of action potentials
D. T
Wave merging can be seen
E. T
49. C
Gap junction
50. B
Specialized sweat glads- found in external auditory canal (outer 1/3) Classed as apocrine
gland
weakest type of cartilage and can be fractured. It is highly compressible and has great tensile
strength
Fine collagen fibers (Type II)embedded in a gel Contains alternating layers of hyaline cartilage
matrix matrix with thick layers of dense collagen fibres,
orientated in the direction of the functional stresses.
Chondrocytes lie in lacunae (cluster of 2-4 cells) Chondrocytes are arranged in rows between the
dense collagen layers within lacunae in the
glycoprotein matrix
1.2. Briefly describe the structural arrangement of a sarcomere in smooth muscle (35 marks)
• In smooth muscle actin and myosin-II are present, they slide on each other to produce
contraction
• Like Z lines in skeletal muscle, the contain dense bodies in cytoplasm, attached to cell
membrane.
• Less extensive sarcomere and few mitochondria present, compared to skeletal muscle and
cardiac muscle
Deve 4 sperms from single meiosis Develop only one egg from single meiosis
Won’t get arrested in metaphase-II Arrested in metaphase 1 until the arrival of sperm
• Assures a different genetic make up for the next generation as a result of crossing over and
new combination of genes
3.2.1
• When the muscle start to stretch, it will activate muscle spindles, start to contract intrafusal
fibres.
3.2.2
• When a muscle or any organ cease its action for a long period, it will enter atrophy state
• So the nutritional supply (blood supply) will start to reduce for the muscle
• That is not enough to rebuild the muscle protein destructed during normal wear and tear
20. Creatinine,
A) Level points out urinary function
B) Converted to creatine phosphate by creatine kinase
C) High molecular compound
D) Found in plant food
E) Depends on muscle mass in the body
SBR
43. Which type of connective tissue present in aorta for recoiling adaptation?
A) Dense elastic CT
B) Dense irregular
C) Dense regular D) Loose areolar
E) Loose reticular
44. Which gland secrete the oil like substance that prevent dehydration of the hair and skin?
A) Apocrine sweat gland
B) Ceruminous gland
C) Eccrine sweat gland
D) Goblet cell or uni cellular gland
E) Sebaceous gland
1.
1.2. State the components of the Haversian system. (20 Marks)
1.3. Compare and contrast the structure of skeletal and cardiac muscle. (30 Marks)
1.4. Explain the adaptations of alveolar epithelium for its functions (20 Marks)
2.
2.5. Strenuous exercise eventually builds up a significant oxygen debt that forces an athlete to terminate the
exercise session.
2.5.1. What is oxygen debt? (10 marks)
2.5.2. State the importance of repaying oxygen after a strenuous exercise. (15 marks)
5.
5.2. High frequency stimulation of skeletal muscle produces tetany (summation / sustained contraction). But not
in cardiac muscle.Explain the physiological basis of
5.2.1. tetany in skeletal muscle on high frequency stimulation. (25 marks)
5.2.2. inability to produce tetany in cardiac muscle (25 marks)
04.
A. T
B. F
Endochondral ossification
C. T
D. F
Endochondral ossification
E. T
05.
A. T
Single chondrocyte in lacuna
B. F
Has a perichondrium [ Lacks perichondrium→fibro cartilage ]
C. T
D. T
Found in the external ear, auditory tubes and the epiglottis
E. T
Elastic fibers enable it to withstand repeated bending
16.
A. T
Cells of epithelial tissue are attached to a basement membrane. Supported by connective
tissue.
B. F
Epithelial tissues are avascular but innervated.
C. T
Simple epithelium has single cell layer Allows passage of materials by diffusion and
filtration
D. F
Stereocilia are the projections in the pseudostratified columnar epithelium cells. They are
cylindrical non-motile. [ Epididymis and vas deferens ]
Presented by 15th Batch 246 FHCS | EUSL
E. T
Adhesion junctions contain plaque on the inside of the plasma membrane Attaches both to
membrane proteins and to microfilaments of the cytoskeleton
17.
A. T
Skeletal – multinucleated
Smooth and cardiac – mono nucleated
B. F
Axon of a motor neuron branches many times.
Each branch extending to a different skeletal muscle fiber.
[ MOTOR UNIT = somatic motor neuron and all the skeletal muscle fibers it innervates ]
C. F
Functional syncytium→cardiac muscles [ gap junctions located in intercalated disc ]
D. T
E. F
Contractile proteins → actin, myosin
18.
A. T
Involuntary – cardiac and smooth
voluntary - skeletal
B. F
Cardiac muscle contraction is initiated by SA node [ pacemaker cells ] Autonomic nervous
system can control the cardiac contraction
C. T
Automaticity/Autorhythmicity [ pacemaker cells undergo spontaneous depolarization ]
D. F
Longer refractory period→250ms
E.
20.
A. T
Creatinine is a breakdown product of creatine phosphate from muscle and protein
metabolism
C. F
Both creatinine and creatine are small molecules D
D. F
Normally creatine found in animal tissues
E. T
Creatine exists as creatine phosphate in skeletal muscles.
32.
A. T
Conduction velocity→A>B>C
B. T
C. T
Groups A and B are myelinated, and group C are unmyelinated
D. T
Myelination increase the conduction velocity of nerve fiber. [ saltatory conduction ]
E. T
Peripheral nerve fibers can regenerate. Central tracts can’t regenerate
37.
A. F
Rich in glycine, proline and hydroxyproline
B. T
Contains glucose and galactose
C. T
D. T
E. F
Marfan syndrome due to defect in elastin fibers.
43. A
Allows recoil of tissue following stretching
60. E
Osteoclasts involve in bone resorption
Compact bones are also known as cortical bones which makes the dense outer layer. It is
important to the resistance of the stress and supportve protection functions.
The major components of the compact bones are harvesian systems. They are also known as
osteons.
Connecting each central canals, volkman’s canals are present horizontal manner.
1.3. Compare and contrast the structure of skeletal and cardiac muscles.
Mono nucleated. Centrally located nucleus. Multi nucleated. Peripherally located nucleus.
Branched Unbranched
Only endomysium and perimysium present as Endomysium, perimysium and epimysium are
connective tissue components. present.
Similarities
Alveolar epithelium is composed of simple squamous epithelium. Its mean there is single
layer of alveolar cells present in alveolar epithelium. Main function of the alveolar
epithelium is gas exchanging. So single layer of alveolar cells reduces thickness of the gas
exchanging epithelium. This leads to facilitate gas exchange across the alveolar epithelium
efficiently. There are 2 types of alveolar cells present in alveolar epithelium. Type 1
alveolar cells participate in gas exchanging. Type 2 alveolar cells produce surfactant which
reduces the surface tension. So surfactant prevents lungs from collapsing. Also alveolar
epithelium is surrounded by network of blood capillaries. This network of blood capillaries
bring sufficient amount of blood for gas exchanging. We all know simple squamous
epithelium has flattened cells like fried egg. So surface area of the simple squamous
epithelium is high. As I said above alveolar epithelium consists of simple squamous
epithelium. Large surface area facilitates gas exchange more efficiently.
2.5)strenuous exercise eventually builds up a significant oxygen debt that forces a athlete to
terminate the exercise session.
Oxygen is needed for conversion of lactic acid in to H2O + CO2 80% of lactic acid enter in
to the blood stream and converted in to pyuruvic acid in kidneys ,liver and cardiac muscles.
Also oxygen is needed for the replenishing of the phosphagen system. Repaying of oxygen
debt also helps to remove the CO2 which accumulate in the blood during exercises
5.2. High frequency stmulaton of skeletal muscle produces tetany (summaton / sustained
contracton). But not in cardiac muscle.
• Skeletal muscle has a refractory period “ Period of time during which a cell is incapable
of repeating an action potential ”
• There are 2
Absolute RP—> completely impossible for another acton potential to be activated
Relative RP —> high frequency stimulus can stimulate a second response.
• In skeletal muscles,
Duration of acton potential and refractory period [ absolute ] is shorter than the duration of
contraction phase.
• When a certain frequency of acton potential is reached, the muscle contract before
relaxation
• So the muscle is in sustained contraction.
• This occurs when a muscle's motor unit is simulated by multiple impulses at a sufficiently
high frequency.
• There are 2
Absolute RP —> completely impossible for another acton potental to ee activated
Relative RP —> high frequency stimulus can simulate a second response.
• Therefore the muscle is always relaxing state when absolute refractory period is over.
• So cardiac muscles cannot be sustainly contracted or tetanioed.
Presented by 15th Batch 252 FHCS | EUSL
• This is a safety measure for cardiac muscles.
11) elastin
A. Is hydrophilic protein
B. Is glycosylated
C. Is protected by alpha 1 anti-trypsin in lung
D. Is the dominant extra cellular matrix protein in arteries
E. Mutation in gene results in narrowing of aorta
60) For which of the following sport would training program for increasing maximal muscle gluconeogenetic
enzyme activity be less important
A. 200m, track
B. 800m, track
C. Basketball
D. High jump
E. Swimming
Answers
11. elastin
B. F
Highly hydrophobic protein
B. F
Not glycosylated
C. T
Alpha 1 anti trypsin protects lungs from neutrophil elastase
D. T
E. T
Mutations in the elastin gene causing a deficiency of the protein resulting in narrowing of
aorta or other arteries as a result of the excessive proliferation of smooth muscle cells in the
arterial wall.
B. T
C. T
D. F
E. T
B. T
C. F
contraction phase longer than refractory period so tetanus can occur in skeletal muscles
D. T
E. T
creatine phosphate is use to convert ADP to ATP in the muscles for muscle contraction
B. F
typical presynaptic terminal contain about 200synaptic vesicles
C. F
they are most abundant in neuronal cell bodies moderately present in dendrites and rare in
axon
D. T
Nissle substance is RER in neuronal cell body.it is primary site of protein synthesis
E. T
nucleus of the neuron is large, round and usually located in centrally
B. T
It is associated with decreased glycosaminoglycan content of the matrix accompanied by
increased water content – Matrix degradations
C. T
D. F
Extracellular matrix contains
• Fibers – collagen, elastin fibers
• Ground substances – chondroitin sulphate
• Water - 70%-75
E. T
C. F
D. F
E. F
C. T
D. F action potential of cardiac muscle causes Ca2+ release to the cell from the
sarcoplasmic reticulum
E. F -90mV
47) Answer E
Compact bone is made up of parallel bony columns. Each column made up of a unit called
osteon or Haversian systems.
The osteocytes are thus arranged in concentric rings within the lamellae.
Haversian canal contain blood vessels, lymphatic vessels, nerves.
Surrounding central canal are concentric rings of osteocytes along with the calcified matrix.
It is called concentric lamellae. Diameters of rings increases from the central canal.
Between the lamellae are small spaces called lacunae which contain osteocytes.
Canaliculi are tiny canal like extension radiates in all directions from the lacunae. They are
filled with extracellular fluid and the plasma membrane extensions of the osteocyte. They
connect lacunae, forming a system of interconnected canals. And provides routes for
nutrients and oxygen to reach the osteocytes.
Interstitial lamellae is mineralized circular plates. They lie between the neighboring osteons.
They are fragments of older osteons. They have lacunae with osteocytes and canaliculi.
Circumferential lamellae is arranged around the entire outer and inner circumference of the
shaft of a long bone. They develop during initial bone formation
- lie deep to the periosteum - outer circumferential lamellae
- line the medullary cavity - inner circumferential lamellae
• Oligodendrogliocytes
• Fewer branches than astrocytes
• More numerous in white matter than in grey matter of the CNS
• Form myelin sheath in CNS
• one oligodendrocyte can myelinate up to 60 axons
• Microglia –
• Thorny bushes in appearance and the smallest glia
• Phagocytic function in CNS
• Constitute the brain’s immune system
• Originate from monocytes
• Ependymal cells
• Simple cuboidal to columnar epithelium line ventricles and central cavities of brain And
spinal
• prevents passage of potentially neurotoxic Substances to the brain
• bears apical microvilli to increase surface Area
• has motile cilia that project into the Ventricular lumen
a. sweep foreign particles.
b. metabolite exchange between CSF and Extracellular spaces of the brain and spinal cord
• Choroid plexus
Presented by 15th Batch 260 FHCS | EUSL
i. Lined by modified ependymal cells
ii. Secrete components of the CSF
5.3. A patient with muscle weakness is evaluated and his arm muscles become progressively
weaker during repeated lifting of a weight. Clinical nerve conduction tests indicate direct
stimulation of the muscle produces normal muscle action potentials and contractions and
direct application of acetylcholine to the motor end plate leads to normal muscle responses.
5.3.1. Based on these test results, what could be the reason for the muscle weakness in this
patient? Justify your answer (30 marks)
Direct stimulation of the muscle producing a normal action potential indicates that there is
no problem with the muscle. The problem lies at the neuromuscular junction. Based on the
symptoms and tests it can be suggested that the patient has myasthenia gravis. It is an auto-
immune condition in which the body produces antibodies against nicotinic acetylcholine
receptors on the motor end plate of the muscle. A reduction in the number of receptors is
seen. When acetylcholine binds to these receptors sites, it causes for the influx of Na+ ions
and an action potential is generated locally. With fewer receptor sites available, the muscles
receive fewer nerve signals, resulting in weakness. Further, Repetitive stimulation of nerves
leads to a reduction in quanta of acetylcholine therefore progressive weakness is seen due to
the number acetyl choline molecules getting reduced. At this state there are fewer receptors
as well as fewer acetyl choline molecules, thus further weakness is observed.
5.3.2. How could the cholinesterase inhibitors improve this above condition? (20 marks)
02) .
a) .
b) Sertoli cells assist in the release of the mature spermatozoa.
c) Abnormal sperm lack of normal motility.
d) It is influenced only LH.
e) The duration of the process is longer than oogenesis.
SBR
11) The most common form of gene expression regulation in eukaryotes is,
a) Translation control
b) Transcription control
c) Post transcription control
d) Post translation control
e) Control of the passage from the nucleus
14) What is the structure remaining active in the mature umbilical cord?
a) Vitelline duct
b) Umbilical vein
c) Allantois
d) Intestinal loop
e) Amniotic cavity
1. Placenta is a feto maternal organ which plays an important role in the maintenance of fetal growth and
development
1.1 Describe the formation of placenta (40)
1.2 Illustrate the structure of fully developed placenta(15)
1.3 State the composition of placental barrier and list 3 functions of it (25)
1.4 Explain the features of fetal membranes in monozygotic twins (20)
2.
2.1 Write an account on
2.2.1Autisomal recessive disorders(20marks)
2.2.2 Structural anomalies of choromosomes.(40marks)
2.2 Briefly describe the steps of Eukaryotic gene expression with illustration.
11th mcq
1)
A)T
B)F granulosa cells in primary follicle
C) T
D)
E)
2)
A)
B)T
C)T
D) F
E)T
3)
A)The appearance of notochord & the prechordal mesoderm
induces the overlying ectoderm to thicken to form neural
plate.
B)T
Neural tube defect
• It results when neural tube closure fails.
• If closure fails in the cranial part: anencephaly
Prevnted by termination of pregnancy
• If closure fails caudally at any part: spina bifida
• Common site lumbosacral region
4)
The epiblast, through the process of gastrulation, is the source of all
of the germ layers
A)
B)
C
D)
E)
5)
Ectodermal layer give rise to organs & structures that maintain contact
with outside world.
1. CNS
2. PNS
3. Sensory epithelium of ear, nose & eye
4. Epidermis including hair & nail
5. Sebaceous glands
6. Mammary glands
7. Pituitary gland
8. Enamel of teeth
A)T
B)F
C)
D)F
E)T
6)
A)T
B)
C)
D)
E)
7)A)
B)
C)
D)
E)
8)
A)F
B)T
C)T
D)T
E)F
9)
A)F
10)
A) F Monosomy 45,XO
B)T
C) F 96-98% do not survive to birth
D) F common in females
E)
Cytogenetics
• It is a branch of genetics is concerned with how the
chromosome relate to cell behaviour.
• The commonest laboratory investigations used is the
Karyotyping
11)B
genes are only expressed when they needed often regulated at transcription
12)D
13)A
14)B
When the allantois and the vitelline duct and its vessels are
also obliterated, all that remains in the cord are the
umbilical vessels surrounded by the jelly of Wharton.
15) E
others all are autosomal recessive
1. Placenta is a feto maternal organ which plays an important role in the maintenance of fetal growth and
development
1.1 Describe the formation of placenta (40)
‐ Placenta has maternal & fetal components.
‐ Fetal part derived from chorionic plate.
Maternal part from uterine endometrium.
1.3 State the composition of placental barrier and list 3 functions of it (25)
Hemochorial barrier.
• Placental barrier initially has 4 layers.
1. Syncytiotrophoblast
2. Cytotrophoblast
3. Connective tissue
4. Endothelial lining of fetal blood vessels.
• From the 4th month, cytotrophoblast & connective tissue degenerate- endothelial lining comes in intimate
contact with the syncytiotrophoblast. Facilitating diffusion.
Presented by 15th Batch 269 FHCS | EUSL
• Placental barrier is not a true barrier-many substance pass through it freely.
• Passes nutrients from mother to fetus -Eg : AAs, free Fatty Acids, Carbohydrates, Vitamins
• Exchanges gases between mother & fetus . Eg:oxygen,CO2,CO
• Allow maternal antibodies to cross it-eg:IgG
• Separate fetal & maternal blood.
3. Isochromosome-it is formwd when a chromosome with 2 chromatids splits at right angles to normal lengthwise
separation at cell division.Resulting chromosomes have either both short arms/both long arms .also net loss will be
an entire long/short arm
4. Inversion- chromosome break at 2points & broken segments turns round 180° to reverse the order of chromatin.
2.2 Briefly describe the steps of Eukaryotic gene expression with illustration.
• Gene expression is the process by which information from a single gene is used to synthesis of a functional gene
product.
• These products are often proteins or functional RNAs.
• Multi steps
Transcription
RNA processing/Splicing
Translation
Post translational modification
2) Primitive streak,
a. Initiate the formation of neural plate
b. Is thickening of ectoderm
c. Is located immediately caudal to precordal plate
d. Is needed for the formation of extra embryonic mesoderm
e. Involves in the of cells
3) Notochord,
a. Is formed during the third week of developed
b. Development to form skeletal system
c. Induce the neural tube forming
d. Formed from the mesoderm
e. Formation intravertible disc of axil skeletal
4) Regarding placenta,
a. It begin to form at 9th week development
b. It expel from body 30 minutes after birth
c. Maternal surface consist 30-35 cotyledons
d. It entire covered by chronic plate on the fetal side
e. Can allow the entry virus the fetus
5) Regarding somites,
Presented by 15th Batch 272 FHCS | EUSL
a. 1st pair appeared 20th day
b. Scelrotome form axial skeleton
c. At 20th day appear 1st pair of somites
d. At 4th week appear 44 somites
e. Somites are formed by notochord
SBR
11) What isn't post transcriptional modification?
a. glycosylation
b. protein degradation
c. protein folding
d. removal on intron by splicing
e. trimming
12) What is the structure internally surrounding the oocyte immediately following ovulation?
Answers
MCQ
1.
A. True
B. True spermatogenesis is regulated by LH porduction by pituitary gland.
C. False. 74days
D.True
E. False. If the oocyte fertilized, meiosis 2 is completed.
2.
Presented by 15th Batch 274 FHCS | EUSL
A. F notochord and prechordal mesoderm
B. F narrow groove with slightly bulging region on either side on the surface of ectoderm.
C. F. Notochord is there between.
D. T epiblast cells migrating through the caudalmost parr of streak+ primitive yolk sac
3.
A. T
B. T become nucleus pulposus of intervertebral disc
C. T. Notochord+prechordal mesoderm
D. F prenotochordal cells from epiblast
E. T
4.
A. T. Start of 9th week or end of 2nd month
B. The placenta should be delivered within 30 to 60 minutes after having the baby. C. F 15to 20
D.T
E. T
Maternal steroidal hormones,synthetic progestin, thyroxine(slow), synthetic estrogen,
viruses(rubella,cytomegalovirus,coxsackie,variola, varicella,measels, poliomyelitis virus) , most drugs
5.
A. T page 82
B. T. Vertebrae,ribs,tendon,cartilage
C. T
D. F 20th day - 1pair; 3pair each day; 28 day(4th week)- 25to 35pair approximately E. F. Paraaxial
mesoderm
6.
A. T
B. T. Cytoplasm and nucleus.
C. T RNA polymerase will read the information sequence on
DNA template from 3′→ 5′ direction, so RNA is synthesized antiparallel to DNA template i.e. from 5′→3′
direction.
D. F. Protomer region is recognized by sigma factor. Termination region is recognized by rho factor.
E. T
7.
A. T
B. T
C. T
D. T
E. T
8. Multifactorial disorders happen due to genetic defect and environmental influences such as nutrition,
medical condition.
A. T
B. F chromosomal disorder
C. T acquired hemophilia
D. T E. T
10.
A. T
B. F estrogen and progesterone
C. T
D. T luteal cells of corpus luteum continue to secrete progesterone which keeps the pregnancy until
end of 4th month. After 4th month, placenta secrete progesterone to keep pregnancy.
E. T
SBR
11. There is an error in the question. It should be post translational instead of transcriptional.
So, we can get the answer, D
12. Answer may be zona pellucida. But,they asked internally
13. C during late stage bilaminar disc separation
14. C In mitochondrial inheritance,Only mother can transmit traits to both son and daughter.
15. D
SEQ
1. Write an account on,
At 13th day of embryonic development, the connecting stalk is formed by extra-embryonic Mesoderm.
At 3 week, Allantois, a diverticulum from yolk sac extends in to connecting stalk. With embryonic folding,
connecting stalk & allantois become ventral of embryo. From 19th day of development, embryonic blood
vessels appear. At the 5th week of development, an oval reflection between amnion & embryonic
ectoderm is the Primitive Umbilical ring.
1. Connecting stalk
• Allantois
In 2nd week of the development of embryo, primitive yolk sac appears. Flat cells from Hypoblast form a
thin membrane (Heuser membrane). This membrane lines inner surface of cytotrophoblast forming the
lining of primitive yolk sac. This occurs about 9th day of development.
At 13th day Hypoblast provides an Additional cell layer. These cells migrate along the inside of exocoelomic
membrane /Heuser membrane. These cells proliferate & make a new cavity within exocoelomic cavity. It
is the secondary or definitive yolk sac.
Its’
At day 16, yolk sac send a diverticulum (allantois) in to the connecting stalk. As embryo folds, yolk sac
become ventral. Closure of lateral body wall except for the umbilical region, where connecting stalk & yolk
sac duct remain attached. Gut tube formed by incorporating a large portion of endoderm in to embryo’s
body. Definitive yolk sac remain outside of the embryo. Mid gut of the gut tube communicates with yolk
sac by broad vitelline duct/vitello inteatinal duct. With further embronic development this duct become
narrow. In the 5th week, yolk sac duct, allantois, with Umbilical vessels are in umbilical cord.
Vitellointestinal duct fibrosed & degenerated at the end of 6th week. Yolk sac detached from mid gut. In
2% of Adults ,Its proximal intraabdominal part persists as “Meckel diverticulum”. At 10th week definitive
yolk sac lies in chorionic cavity between amniotic & chorionic sacs. As pregnancy advances, definitive yolk
sac atrophies.
1.1.3
Autosomal dominant Inheritance (25 marks)
• Special features.
a. New mutations – increased paternal age may be associated with new mutations. Eg:
Achondroplasia
b. Reduced penetrance
c. Variable expressivity- degree of expression of a trait. Individuals in family may show mild to
severe forms of disease. Eg: Marfan syndrome
d. Variation in age-adult polycystic kidney disease, Huntington’s disease show features later in
life
1.2 Briefly explain the biochemical basis of Sickle cell disease with regard to mutation(20 marks)
• Sickle cell anemia is a genetic disorder, caused by a point mutation/single gene mutation.
• Mutant gene for beta globin of hemoglobin. Mutant B globin chain is designated HBs
• Hbs contains 02 normal alpha globin chains & 02 mutant B globin chains.
• Mutation changes the 06th codon “GTG”, which codes for glutamic acid in to “GAG” which codes for
Valine.
• At the low oxygen tension, deoxy Hbs polymerize inside the RBC & form an insoluble fibrous
network .
• Sickled RBCs can block the blood flow in narrow capillaries& interrupt the oxygen supply to tissues
causing pain & eventually death of cell.
1) Regarding spermatogenesis
A. It is beging before puberty.
B. Influence by LH
C. Leydig cells assist in the mature spermatozoa
D. Duration of the process is longer than oogenesis
E. It is a continuous process
3) Notochord
A. Derived from mesoderm
B. Is form cranial to the prechordal plate
C. Is produce migrate epinblast through blastopore
D. Influences formation of neural tube
E. Derivatives in adult is the nucleus pulposes
5) External features of embryo seen during the 2nd month of gestation include
11) Which of following congenital malformation will most predictable result oligohydramnions
A. Anencephaly
B. Spinal brivida
C. Renal agenesis
D. Tracheoesophageal fi stula
E. Urethral atresia
13) Which of the following cross the placental barrier from mother to fetus in a slow rate
A. Maternal steroid hormones
B. Thyroxine
C. Cocaine
D. Megala virus
E. Drug metabolites
B True Regulated by LH. LH binds the receptors on Leydig cells stimulate testosterone production promote
spermatogenesis. Testosterone binds to sertoli cells
C False Sertoli cells support and protect the germ cells,supply nutrition, assist in the release of mature
spermatozoa.
D False Spermatogenesis duration=74 days approximately. Oogenesis duration=begins before birth. Will not
complete the Meiosis before fertilization.
E True
mesoderm)
3 Notochord
A False Derived from endoderm
B False Prenotochordal cells move forwards cranially in the midline until they reach to the prechordal plate
C True
D True
E True
5 External features of embryo seen during the 2nd month of gestation include
A True
❖ Increase head in size.
❖ Formation of the limbs ,face ,ears, nose and eyes.,crest cells derivatives ❖ Counting of smites
becomes difficult. therefore the age of the embryo indicated as crown rump length (CRL) ❖ Paddle shaped
limb buds
chromosome 22.
❖ Cryptochydism
❖ Post axial polydactyly
❖ Facial defects
• Micrognathia
• Central or unilateral clefts in lip and palate
• Eye defects (hypotelorism)
• Micropthalmia/ anopthalmia /cyclopia( single central eye)
B True
C True
D True
E False In turner syndrome.
SBR
11 C
❖ Oligohydramnios means decreased amount of amniotic fluid (<400ml) ❖ It is may result
Causes for hydramnios are idiopathic causes, maternal diabetics, congenital malformations [ CNS
disorders (anencephaly), GIT defects (atresias )]-they prevent the infant from swallowing the fluid.
12 D
• Inserting or deleting of one or more nucleotides.
• Changes the reading frame.-shift the reading frame of genetic message.
• Proteins built incorrectly.
• Ex:- Tay-sachs disease
13 B
❖ Steroidal hormones readily cross the placental barrier.
❖ Other hormones such as thyroxine do so only at a slow rate.
14 C
• Reduce penetrance means although only one mutated copy is needed, not all individuals who
15 D
• Most common
• Simultaneous shedding of 2 oocytes and fertilized by different sperms.
• 2 zygotes have totally different genetic constitutions.
• The zygotes implant individually in the uterus.
• Each has its own placenta, amnion, chorionic sac
Monozygotic twins
• Monozygotic/ identical twins
• Developed from a single fertilized ovum
• Result from splitting of the zygote
❖ During Two cell stage-2 separate zygotes developed. Blastocysts implant separately.
Has its own placenta and chorionic sac. ❖ During early blastocyst stage- inner cell
mass split into 2 within the same blastocyst cavity. 2 embryos have common placenta,
common chorionic cavity but separate amniotic cavities. ❖ During bilaminar germ disc
SEQ
01
1.1 Describe the formation of the placenta (30 marks) Placenta has maternal & fetal components. Fetal part
derived from chorionic plate. Maternal part from uterine endometrium. Formation of Fetal partchorionic plate
(Trophoblast+Extraembryonic mesoderm) 11th batch Medicine. Primary villi formation - by the beginning of 3rd
week, appearance of cytotrophoblastic core covered with syncytial layer. Secondary villi formation- During
further development, mesodermal cells penetrate the core of primary villi & grow towards decidua. Tertiary
villi formation- mesodermal cells in the core of villi begin to differentiate into blood cells & small blood vessels.
They form villous capillary system. Capillaries in tertiary villi join with developing capillaries of chorionic plate &
connecting stalk. Establishing contact with intraembryonic circulatory system. Outer cytotrophoblastic shell
formation -meanwhile cytotrophoblastic cells in villi penetrate into syncytium & cover it. This shell gradually
surrounds trophoblast entirely& attaches the chorionic sac firmly to the maternal endometrium. Anchoring
villi/Stem villi – villi that extend from chorionic plate to decidua basalis of maternal endometrium. Stem villi
branch from the sides to form terminal/free villi in to intravillous spaces. Exchange of nutrients & other factors
occur through these free villi. By 19th-20th day embryo is attached to its trophoblast shell by connecting stalk.
Presented by 15th Batch 285 FHCS | EUSL
By the beginning of 2nd month trophoblast has a great number of secondary &tertiary villi. By the 4th month
cytotrophoblastic & some connective tissue cells disappear in the villi, forming larger villi. Thereby placental
barrier formed by syncytium & endothelium.
Villi covers entire surface of chirion.as pregnancy advanced, villi on embryonic pole grow & expand to
form chorion frundosum. Villi on abembryinic pole degenerate. Maternal part development-spiral
artery delivered maternal blood to placenta. They erode &release blood in to intervillous spaces.
Invasion of maternal spiral arteries by cytotrophoblastic cells. Creating hybrid vessels containing both
maternal & fetal cells. This process transforms small diameter high resistance vessels to large diameter,
low resistance vessels. The decidua of endometrium over chorionic frundosum is decidua basalis. This
connects the chorion tightly. During 4th &5th months decidua forms a number of decidual septa, which
project in to intervillous spaces divide the placenta in to cotyledons. Together with decidua basalis,
chorionic frundosum makes up the placenta.
1.3. Explain how you would perform karyotyping to detect any chromosomal anomalies of a new born.(30
marks)
• Karyotyping is a technique that uses a picture of an individual’s chromosomes to analyse & detect
chromosomal abnormalities.
• Can be used to analyse the size, structure & number of chromosomes in a sample.
• Chromosome cultures are performed in rapidly dividing cells from metaphase stage of Mitosis.
1. Collecting samples-Cells from new born. Eg:WBC
2. Separating cells
3. Culturing cells-adding culture medium, harvested cells are cultured, at body temperature
(37°) for 3 days.
4. Cell sample is treated with Colchicine to arrest the cells in Metaphase.
5. Centrifuge the cells. It concentrates the cells at the bottom of the tube.
6. Addition of hypotonic saline-to swell up & more apart the cells.
7. Lysed cells are prepared, fixed using alcohol & acetic acid. Placed on a microscopic slide.
8. Digest with trypsin & stain with Giemsa. 11th batch Medicine
9. Analyse metaphase spread. G banding appearance can be seen in Giemsa staining. Series of
lightly &darkly stained bands in chromosomes. Dark bandstend to be heterochromatic, late
replicating & AT rich Light bandseuchromatic, early replicating GC rich.
10.The good spreads are captured using photomicroscope.
11.Individual chromosomes cut & arranged in pairs in decreasing order of size. Numbered from 1-22 &
sex chromosomes separately.
12.The photographic representation of entire somatic chromosomes complement is karyotype.
1. Transcription- is RNA production, by copying part of DNA. Started & controlled by RNA polymerase
enzyme. RNA polymerase doesn’t need primers to begin synthesis of RNA. This enzyme binds to DNA
double strand at promoter site. Unwinding that region & reads the template DNA strand from 3’ to 5’
direction. RNA is synthesized antiparallel To DNA strand.
2. RNA processing- Post transcriptional Modification. Introns are removed by splicing. Exons come
together. 5’ capping by 7 methyl guanosine. Addition of poly A tail produces mature RNA molecule.
3. Translation- synthesis of a protein molecule from m RNA.;Ribosome attaches to mRNA molecule;
tRNA molecule arrives. It brings an amino acid to first three bases (codon) in mRNA; Start codon is AUG.
Anticodons in tRNA link up with codons on mRNA; Ribosome reads the mRNA; Another tRNA comes
with AA; Peptide bond forms between AAs;As Ribosome moves, tRNAs comes with AAs.
Elongation of polypeptide chain.
When stop codon (UAA,UGA,UAG) is reached, termination occurs.Result is polypeptide chain.
4. Post translational modifications- trimming Covalent attachment (phosphorylation, glycosylation,
hydroxylation) Protein folding Protein degradation
07. Karyotyping
A) Helps to detect chromosomal disorders.
B) Can determine Edward syndrome.
C) The cells that used are in the telophase of meiosis.
D) Can be performed from chorionic villi cells of the fetus.
E) Helps to identify the defect in the chorionic myeloid leukemia.
12. On ultrasound examination, it was found that a fetus had well-developed facial and thoracic regions, but caudal
structures were abnormal. Lumbar and sacral vertebrae were missing, and the hind limbs were fused. What process
may have been distributed to cause such defects?
A) Neurulation.
B) Gastrulation.
C) Folding of embryo.
D) Notochord formation.
E) Placentation.
13. Placenta is the fetomaternal organ compensation the natural demand as the fetal grows. Which of the
following statement correctly define the placental barrier?
A) It acts as a barrier permeability harmful virus to enter the fetus.
B) It is matured by incorporating the cytotrophoblast.
C) It can allow the IgG immunoglobulins to escape from the maternal blood.
D) It can allow the maternal thyroxin to diffuse rapidly into the fetal circulation.
E) It is composed of endothelial lining of the maternal vessels with the syncytium.
15. Which of the following type of genetic heterogeneity in the affected population,
A) X linked recessive
B) Y linked dominant
C) X linked dominant
D) Autosomal dominant
E) Autosomal recessive
1.
1.1. Describe the oogenesis process. (30 marks)
Answers
01
A) F = primordial germ cells are diploid cells.
B) T
C) T
D) F = It can undergo only mitotic cell division.
E) F = 5th week.
02
A) T
B) T
C) F = cortical oocyte granules are released.
D) T
E) F = 2nd polar body release.
03
A) T
B) F = before implantation, in the early blastocyst stage zona pellucida is removed.
C) F = epithelial defect during the menstrual phase. During the implantation
epithelium
has proliferated.
D) T
E) T
04
A) F = during the 3rd week of development. (during the embryonic period)
B) T
C) T
D) T
E) F = during the 5th month.
05
A) T
B) T
C) F
D) T
E) F
06
A) T = It causes cardiac defect & ear abnormalities.
B) F
C) T = It causes microcephaly.
D) T = It causes vision loss, hearing loss & anaemia.
E) T = It causes mental retardation & leukaemia.
07
A) T
B) T = 47,XY, +18 or 47,XX, +18.
C) F = metaphase of mitosis.
D) T
E) F
08
Presented by 15th Batch 290 FHCS | EUSL
A) T
B) F = both sees are equally affected.
C) T
D) T
E) F = It is a single gene disorder. Short arm deletion is chromosomal
mutation
09
A) F = 45,X.(monosomy)
B) T
C) T
D) F = can be seen in females (only).
E) F = detected through the karyotype.
10
A) F = more females are affected than males.
B) F = never transmit to sons, but to all daughters.
C) T
D) T
E) T
11
A) T = form the placenta
B) F
C) F
D) F
E) F
12
A) F
B) T = Sirenomelia.
C) F
D) F
E) F
13
A) F = harmful viruses can pass through the placenta to fetus.
B) F
C) T
D) F = thyroxine slowly cross & steroid hormones rapidly cross the placenta.
E) F
14
A) F
B) F
C) T = mitochondrial disorder (specially transmitted from mother).
D) F
E) F
15
A) F
B) F
C) F
D) T
E) F
14th seq
01)
1.1 )
•Oogenesis Is the sequence of events by which germ cells oogonia differentiate into mature oocytes.
chromosomal abnormalities.
• overian cycle is changes in a female’s ovary
• From sexual maturation to menopause
• Usually lasts 28 days
• During cycle primordial follicle resumes maturation and prepared
for ovulation
this the growth follicle
Ovulation
• Due to stimulation of LH & FSH secondary follicle grows.
• With final development of 2ndry follicle a sudden increase of LH
occurs
• LH causes the completion of meiosis I and help to enter the pre-
ovulatory phase
• Meiosis II started but arrested in metaphase.
• Surface of ovary bulge and at the apex an avascular stigma appears.
1.2)
*It provides consistency of the chromosome number from generation to generation.
*It allows random assortment of maternal and paternal chromosome between gametes.
*It relocates segments of maternal and paternal chromosomes by crossing over of chromosomes segments and
produce a recombination of genetic material.
1.3)
*intra embryonic mesoderm seperates the ecto & endoderm except prochordal plate, claocal membrane & the
place occupied by notochord.
At the edges of the disc it continues as extra embryonic mesoderm
IEM has 3 parts are,
1. Para axial mesoderm:
2. Lateral plate mesoderm:
3. Intermediate mesoderm:
in the 3rd week Paraxial mesoderm
- Organized in to segments; They somatomeres.
- Formation proceeds cephalocaudally.
- Somatomeres of head region associated with the segmentation of
neural plate called neuromeres, which forms the mesodermal cells
of head.
- In the occipital region somatomeres organised into somites.
• First pair of somite arise in the occipital region
• New somites arise in craniocaudal direction in a rate of 3pairs/per day.
• Formation starts around 20th day and ends in 5th week.
• A total of 42- 44 pairs get formed.
- occipital: 4
- cervical: 8
- thoracic: 12
- lumbar: 5
- sacral: 5
- coccygeal : 10
• 1st occipital and last 5- 7 coccygeal somites later disappear.
• Remaining somites forms the axial skeleton.
• As the somites develops in a specific period, the age of the embryo can
be accurately determined by counting the somites
Differentiation of somites
- Initially somites exist as ball of mesoderm cells.
1.4)
Point Mutation
• Sickle Cell disease is the result of one nucleotide substitution
• Occurs in the hemoglobin gene
Point mutation can be seen in 3 types
1.Silent mutation
*No aminoacid change * redundancy in code
2.Missense
*change amino acid
3.Nonsense
*change to stop codon
*A sequence of 3 bases repeated in tandem will become amplified in number causing Too many copies of triplet
*if happen in coding region
The protein will have extra copies of amino acid
resulting in Trinucleotide repeat expansion
Eg: Huntington disease
Frameshift Mutation
• Inserting or deleting one or more
nucleotides
• Changes the “reading frame” like
changing a sentence
• Proteins built incorrectly bcz Amino Acid Sequence is Changed Frameshift Mutation
Eg:- Tay sachs disease
3) Events that occur during 11th and 12th days of gestation include,
A. Large lacunae appears in syncytium
B. Blastocyst is completely embedded in endometrial stroma
C. Uteroplacental circulation is established
D. Exocoelomic cyst forms ventrally
E. Primary villi appeared among the cytotrophoblasts
7) Turner’s syndrome,
A. is a trisomy
B. shows gonadal dysgenesis
C. Survive after few months of birth
D. Can be detected by cytogenetic techniques
E. Both sexes are affected
9) Which of the following abnormalities correctly paired with the mutation that can lead to,
A. Frameshift mutation – Tay Sachs disease
B. Missense - hemolytic anemia
C. Disjunction - hemophilia
D. RNA splicing disorders - Myotonic dystrophy
E. Trinucleotide repeat expansion- Huntington disease
SBR
11) What is the relationship among DNA, a gene & a chromosome?
A. a chromosome contains genes, which are composed of DNA
B. a chromosome contains hundreds of genes, which are composed of protein
C. a gene contains hundreds of chromosomes, which are composed of DNA
D. a gene contains hundreds of chromosomes , which are composed of protein
E. a gene is composed of DNA but there is no relationship to chromosome
15) High level of alpha- fetoprotein in maternal serum after 16 weeks is indicative of which following condition?
A. Trisomy 21
B. Marfan syndrome
C. Anencephaly
D. Monosomy x
E. Klinefelter syndrome
1.
1.1. Explain briefly the changes that takes in the mesoderm located closely to the central axis and state its
derivatives. (30 marks)
1.2. Write accounts on
1.2.1. Autosomal recessive inheritance. (20 marks)
1.2.2. Amniotic fluid. (20 marks)
1.3. Briefly describe the structure and functions of chromatin in eukaryotic cells. (30 marks)
Answers
MCQ
2.
True Cells of the inner cell mass or embryoblast also differentiate into two layers:
A (1) a layer of small cuboidal cells adjacent to the blastocyst cavity, known as
the hypoblast layer (2) a layer of high columnar cells adjacent to the amniotic
cavity, the epiblast layer
B Since a layer of epiblast dorsally and hypoblast ventrally is formed Dorso
ventral polarity is established during this stage.
C the cranial—caudal embryonic axis is established near the time of
implantation
D False hypoblast cells are specified to form the anterior Visceral endoderm (AVE)
E
3.
4.
5.
True At the end of the fourth week, when the embryo has approximately 28
A somites, the main external features are the somites and pharyngeal arches
During 2nd month external appearance of embryo is changed
Presented by 15th Batch 299 FHCS | EUSL
B True • Head size
C False • Limbs, face, ears, nose & eyes formed
D
E True
6.
A True
B True
C CVS is usually carried out between the 11th and 14th weeks of
pregnancy, although it's sometimes performed later than this if
necessary.
D False The AFP blood test is called MSAFP (maternal serum AFP). The AFP can
also be measured in the amniotic fluid, called AFAFP
E True
7.
False It is a monosomy
A
B True characterized by the absence of ovaries (gonadal dysgenesis) and short
stature.
C False 55% die within 2 months 5% survive for beyond 1 year
D True
E False Only females
8.
True
A
B false 47, XXY
C True
D False An example of microdeletion
E False Deletion of short arm at 5th chromosome
9.
true
A
B True Change of amino acid occurs in sickle cell anemia. Sickle cell anemia leads to
hemolytic anemia.
C False Mutation cause by pathogenic variants or deltion
D
E true Huntington's disease is caused when the trinucleotide repeats in the
HTT gene expand beyond the normal range
10.
False In contrast to DNA polymerase, RNA polymerase does not need a primer to
Presented by 15th Batch 300 FHCS | EUSL
A begin synthesis
B True the transcription is initiated by the binding of RNA polymerase to a specific
region of DNA double helix. This site is called promoter site or promoter
region.
C True RNA polymerase will read the information sequence on DNA template
from 3′ → 5′ direction
D
E False Amino acyl tRNA synthetase Required for attachment of aa to their
corresponding tRNAs
11. A
12.
A
B In XXY they have female external genitals but no gonads.
C Gene for hemophila is located on the x chromosome
D True
E Y chromosome is acrocentric while x chromosome is submetacentric.
14. E cells, derived from yolk sac cells, form a fine, loose connective tis- sue, the extraembryonic
mesoderm
15. C
SEQ
1.1
The mesoderm beside the midline thickens to form the paraxial mesoderm. This is formed by the cells
migrating from the lateral edges of the primitive node. Initially the mesoderm exisits as thin line of cells on
eother side of the midline. By the 17th day it is thickened to form the paraxial mesoderm.
At the beginning of 3rd week the paraxial mesoderm is organized into segments known a somatomeres
these appear cephalocaudally. In the head region these get associated with segments of the neural plate to
turn into neuromeres. This contributes to the formation of mesoderm of the head.
In the occipital region the first somite appears at day 20. They appear at a rate of 3 per day for 15 days in
the cranio caudal direction. By the end of 4th week a total of 42-44 pairs get formed.
occipital: 4 - cervical: 8 - thoracic: 12 - lumbar: 5 - sacral: 5 - coccygeal : 10
Out of this the first occipital and the last 5-7 coccygeal disappear later.
The remaining somites form the axial skeleton. They are used to determine age of the embryo.
Development of somites
The somites initially occur as a ball of cells which then undergo epithelialization to forma lumen, The
smites now have a donut shape.
Cell located in the ventral & medial walls of the somite become mesenchymal and shift to surround the
neural tube & notochord and become the sclerotome
Cell in the dorsomedial and ventrolateral edges become muscle precursor cells.
Cells between these 2 group of cells become the dermatome.
Cells from the muscle precursors cells migrate beneath the dermatome to form the dermomyotome.
Sclerotome forms the vertebra and ribs.
Dermatome forms the dermis of the back.
And myotome forms the segmental muscular compartment.
Presented by 15th Batch 301 FHCS | EUSL
Each somite and subsequently its derivatives, retain their segmental innervation, giving rise to specific
dermatome and myotomes which are exclusively supplied by a specific spinal segment.
Each dermatome and myotome has its own segmental nerve component, and retains it no matter where
the cells ultimately migrate.
In addition to axial structures, the paraxial mesoderm gives rise to the muscles of the limbs.
1.21
Two copies of the gene must be mutated for a person to be affected by an autosomal recessive disorder.
• An affected person usually has unaffected parents who each carry a single copy of the mutated gene and are
referred to as "carriers".
• Each parent with a defective gene normally do not have symptoms.
• Two unaffected people who each carry one copy of the mutated gene have a 25% risk with each pregnancy of
having a child affected by the disorder
Special features
1. Carrier states are usually normal but may show half normal enzyme levels such as in hexosaminidase A levels in
Tay Sach disease.
2. Consanguinity increases the chance of affected births
3. Screening is needed in certain racial groups carry higher frequency of recessive genes.
4. The sickle cell carriers develop sickling in high altitudes
Eg:
Homocystinuria
Cystic fibrosis
Sickle cell anaemia
Thalassaemias
1.22
The amniotic cavity is filled with a clear, watery fluid that is produced in part by amniotic cells but is derived
primarily from maternal blood.
The amount of fluid increases from approxi- mately 30 mL at 10 weeks of gestation to 450 mL at 20 weeks
to 800 to 1,000 mL at 37 weeks.
During the early months of pregnancy, the embryo is suspended by its umbilical cord in this fluid, which
serves as a protective cushion.
The fluid
(1) absorbs jolts,
(2) prevents adherence of the embryo to the amnion, and
(3) allows for fetal movements.
The volume of amniotic fluid is replaced every 3 hours.
From the beginning of the flfth month, the fetus swallows its own amniotic fluid, and it is estimated that it
drinks about 400 mL a day, about half of the total amount. Fetal uriñe is added daily to the amniotic fluid in
the flfth month, but this uriñe is mostly water because the placenta is functioning as an exchange for
metabolic wastes
1.3
The DNA is extensively folded & packed into a protein. This DNA coplex is what is known as chromatin
• Chromatin is the complex combination of DNA that makes up chromosomes (mesh work of thread)
05. Congenital anomalies that commonly occur during the first trimester include
A. Cleft lip
Presented by 15th Batch 303 FHCS | EUSL
B. Cryptorchidism
C. Finger webbing
D. Neural tube defect
E. Sirenomelia
SBR
11. Which of the following genetic disorders show reduce penetrance/ variable expressivity?
A. Autosomal dominant
B. Autosomal recessive
C. X linked recessive
D. X linked recessive
E. Y linked dominant
13. On ultrasound examination, it was found that a fetus had well-developed facial and thoracic region, but caudal
structures were abnormal. Lumbar and sacral vertebrae were missing, and the hind limbs were fused. What process
may have been distributed to cause such defects?
A. Formation of fate map
B. Formation of lateral folding
C. Gastrulation
D. Formation of neural tube
E. Notochord formation
1.
1.1. Write accounts on 1.1.1. Structural chromosomal disorders. (25 marks)
1.1.2. Ultrasound scanning techniques (15 marks)
1.1.3. Neurulation & fate of neural crest cells. (30 marks)
1.2. Draw and label the structure of eukaryotic gene (15 marks)
1.3. Briefly explain the principles of regulation of gene expression. (15 marks)
01.
A. True Capacitation involves epithelial interaction between sperm and mucosal surface of the uterine tube.
B. True
C. False Not by androgens. It’s an effect of epithelial interaction which leads to removal of a glycoprotein coat
and seminal plasma proteins from plasma membrane that overlies the acrosomal region of sperm.
D. False. This conditioning process lasts approximately 7hours.
E. True. Only capacitated sperm can undergo the acrosomal reaction.
03.
A. True
B. True Invagination process
C. True. Initially, streak is vaguely defined. But, in 15to 16 day, it is clearly visible as a narrow
groove.
D. False. There is notochord between primitive streak and prechordal plate
E. True. Extra embryonic mesoderm has 2 sources. 1. Primitive yolk sac (hypoblast). 2. Epibalst cells migrating
through the caudal most part of primitive streak.
04. Initially placental membrane/barrier has 4 layer-: endothelial lining of fetal vassels, connective tissue of
villous core, cytotrophoblastic layer, syncytium.
From 4th month on, placental barrier(mature) has 2layers-: endothelial lining of fetal vassels and syncytium.
A. T
B. F
C. F
D. F
E. T
05.
A. T. 2to 3rd month of pregnancy
B. F. Testes are present in scrotum before birth in 97% male new born. In most of the remainders, descent will
be completed during first 3rd month of postnatal period.
C. T. Finger webbing/syntactyly
D. T defect in neural tube formation
E. T sirenomelia/caudal dysgenesis os caused by problems with gastrulation.
06.
A. T
B. T
C. F Haploid chromosomes are formed in spermatids. Then, spermatids are changed into spermatozoa by
spermiogenesis process
D. F After fertilization (page no. 42)
E. T
07.
A. F Autosomal dominant
B. T
C. F x linked
D. F Autosomal dominant
08.
A. F It is one of microdeletion syndrome. Not chromosome disorder.
B. T
C. F. Klinfelter-47,XXY
D. T
E. F Turners- 45,XO
09.
A. F proximally
B. T
C. T remnant of alantois.
D. F. Amniotic fluid is not a content of umbilical cord since it is out of umbilical ring. E. T
10.
A. T
B. F chromosomal mutation
C. T
D. T
E. T
SBR
11. A
They are special features of autosomal dominant inheritance.
Reduced penetrance- It is a clinical term which is an expression of the number of individuals who have the gene
and show the trait. It is an all or nothing phenomenon and if the frequency is less than 100%, reduced
penetrance exists.
Variable expressivity – It refers to the degree of expression of a trait. The individuals in a family may show mild
to moderate to severe forms of the disease and need to be examined carefully in order not to miss the
diagnosis. Multiple Neurofibromatosis and Marfan syndrome are good examples of reduced penetrance and
variable expressivity.
12. E
Can detect by prenatal diagnosis such as amniocentesis.
13. C
Page no-67
14. D
15. D
SEQ
These include deletions, ring chromosomes, duplications, isochromosomes, inversions, and translocations. The
acentric, centric fragments and dicentric chromosomes which may arise are unstable at cell division. These are
lost and result in cell death. However, stable alterations are also seen.
Deletion: A deletion leads to a loss of chromatin. In terminal deletions the break is at the end of an arm. Cri du
chat syndrome has the tip of the short arm of chromosome No. 5 deleted (5p-). They have a low birth weight
and have failure to thrive. These children have a round face, low set ears, with profound learning disability,
hypertelorism [widely spaced eyes), epicanthic folds and have malformed larynxes which cause them to cry
with a sound like a cat. Cri du chat (CAT LIKE CRY). In interstitial deletions two breaks occur within the
chromosome arms, the broken segment is lost and the ends unite to form a short arm. In ring chromosome
disorders, two breaks occur at the tips of both arms of the chromosome, acentric pieces are lost from the ends,
and the two broken arms curl towards each other and fuse to form a ring.
In microdeletion syndromes, very small deletions take place and are detected only by special high resolution
banding. An example is Prader-Willi syndrome involving the long arm of chromosome 15. When the deletion is
in the maternal chromosome 15, Angelman syndrome results. This is an example of imprinting involving genes.
Duplication: It results in a gain of chromosome material by doubling, in a particular region.
Isochromosome: It is formed when a chromosome with two chromatids splits at right angles to the normal
lengthwise separation seen at cell division. The resulting chromosomes will have either both short arms or both
long arms. The net loss will be an entire long arm or an entire short arm. Twenty percent of Turner syndrome
results from having one normal X and another X which is an isochromosome for the long arm, essentially lacking
a short arm (short at monosomy and long arm trisomy). Most isochromosomes cause a spontaneous abortion.
Inversion: In this category chromosomes break at two points and the intervening broken segment turns round
180 degrees to reverse the order of chromatin. If the break points are on the same arm, it is called a paracentric
inversion and if it is on either side of the centromere, including it in the broke segment, it is referred to as a
pericentric inversion. The medical significance is that it leads to chromosomally unbalanced gametes
Translocation: A translocation involves the exchange of chromosomal material between chromosomes. There
are three types of these exchanges.
1. Centric fusion or Robertsonian translocation: This results from a fusion of whole arms of acrocentric
chromosomes. The breakpoints are at or near the centromeres of both chromosomes. The fused long arm
chromosome survives while the fused short arm chromosome is lost. This loss produces no effect as the short
arms of acrocentrics contain genetically inert material or RNA genes. Basically the chromosome complement
here will be 45 and the carrier will be quite healthy as these are all acrocentric chromosomes that carry
multiple copies of RNA genes in their short arms whose loss produces no serious consequences. Eg: 4% of Down
syndrome.
2. Reciprocal Translocation: This results from breakage and exchange of segments between chromosomes.
The points of exchange may be anywhere along the chromosome. The reciprocal translocations show exchange
of segments with no loss of chromosome material. Eg: In 90% of patients with chronic myeloid leukemia (CML),
and in 10-15% of acute lymphocytic leukemia (ALL), the bone marrow shows the presence of the Philadelphia
chromosome (Ph).
3. Insertional translocation: This involves the insertion of a deleted segment of a chromosome interstitially or
inside another chromosome following a break at that point. This is very rare but if present may produce
chromosomally unbalanced offspring.
1.1.2
Ultrasound scanning
Presented by 15th Batch 308 FHCS | EUSL
Ultrasonography is a relatively non invasive technique that uses high-frequency sound waves reflected from
tissues to create images. The approach may be trans abdominal or trans vaginal, with the latter producing
images with higher resolution. In fact, the technique, which was first developed in the 1950s, has advanced to a
degree whereby detection of blood flow in major vessels, movement of heart valves, and flow of fluid in the
trachea and bronchi are possible. The technique is safe and commonly used.Important parameters revealed by
ultrasound include:
• Characteristics of fetal age and growth
• Presence or absence of congenital anomalies
• Status of the uterine environment, including the amount of amniotic fluid. Placental position and
umbilical blood flow Whether multiple gestations are present.
All of these factors are then used to determine proper approaches for management of the pregnancy.
Determination of fetal age and growth is crucial in planning pregnancy management, especially for low-birth-
weight infants. In fact, Studies show that ultrasoundscreened and managed pregnancies with low-birth-weight
babies. Reduced mortality rate by 60% compared with an unscreened group. Fetal age and growth are assessed
by crown-rump
length during the 5th to the 10th weeks of gestation. After that, a combination of measurements—including the
Biparietal diameter (BPD) of the skull, femur length, and abdominal circumference—are used. Multiple
measurements of these parameters over time improve the ability to determine the extent of fetal growth.
Congenital malformations that can be determined by ultrasound include:
• The neural tube defects( anencephaly and spina bifida)
• Abdominal wall defects (omphalocele and gastroschisis)
• Heart and facial defects ( cleft lip and palate)
1.1.3
Neuralation and fate of neural crest cells.
Neuralation
At the beginning of the third week of development, the ectodermal germ layer has the shape of a disc that is
broader in the cephalic than in the caudal region. Appearance of the notochord and prechordal mesoderm
induces the overlying ectoderm to thicken and form the neural plate. Cells of the plate make up the
neuroectoderm, and their induction represents the initial event in the process of neurulation. Neurulation is the
process whereby the neural plate forms the neural tube. One of the key events in this process is lengthening of
the neural plate and body axis by the phenomenon of convergent extension, whereby there is a lateral to
medial movement of cells in the plane of the ectoderm and mesoderm. The process is regulated by signaling
through the planar cell polarity pathway and is essential for neural tube development. As the neural plate
lengthens, its lateral edges elevate to form neural folds,and the depressed midregion forms the neural groove.
Gradually, the neural folds approach each other in the midline, where they fuse. Fusion begins in the cervical
region (fifth somite) and proceeds cranially and caudally. As a result, the neural tube is formed. Until fusion is
complete, the cephalic and caudal ends of the neural tube communicate with the amniotic cavity by way of the
anterior (cranial) and posterior (caudal) neuropores, respectively. Closure of the cranial neuropore occurs at
approximately day 25 (18- to 20somite stage), whereas the posterior neuropore closes at day 28 (25-somite
stage). Neurulation is then complete, and the central nervous system is represented by a closed tubular
structure with a narrow caudal portion, the spinal cord, and a much broader cephalic portion characterized by a
number of dilations, the brain vesicles.
1.2
1.3
Briefly explain the principles of regulation of gene expression.
Gene expression is the information from a gene, is used in the synthesis of a functional gene product. These
products are often proteins, but in non- protein coding genes such as rRNA genes or tRNA genes, the product is
a functional RNA. The genes can be turned on and off as a result of interaction with regulatory proteins. The
expression of these genes can be controlled in many steps,
1) Transcriptional control
2) RNA processing control 3) RNA transport control 4) Translational control.
Feedback inhibition: When enough amount of product is made, the system will shuts down.
Gene expression: Genes are only expressed when needed. This is often regulated at transcription.
8TH BATCH
8TH PROPER MCQ
SBR
03) Which embryonic tissue is correctly match to its ultimate derivative?
A) Epiblast- connective tissue
B) Endocyte- connective tissue
C) Hypoblast- Endoderm
D) Mesoderm- Epithelium of urinary tract
E) Ectoderm- Epithelium of nasal cavity
06) Notochord
A) Is the early umbilical cord
B) Is precursor of vertebral column
C) lies dorsal to the spinal cord
D) Remains in the adults as nucleus pulposes
E) Formed by hypoblast
3.
3.1. Define the embryonic period (05 marks)
3.2. List 2 major events that take place immediately after the implantation (05 marks)
3.3. Describe the process of one of the event stated for Q 3.2 (30 marks)
3.4. State the effects of teratogens by giving 5 examples (20 marks)
3.5. Describe the formation of placenta (40 marks) (4.0.)
4.
4.1. List 5 prenatal diagnostic techniques that are performed (15 marks)
4.2. Write an account of a non-invasive prenatal diagnostic technique which is used frequently (35 marks)
4.3. Give a brief account on
4.3.1. Karyotyping (25 marks)
4.3.2. Downs syndrome
Answers
MCQ
01)
A) F
Teratogen is an agent that can produce a permanent alteration of structure or function in an organism after
exposure during embryonic or fetal life.
B)
C) T A mutagen is an agent that can alter the structure of a chromosome or the DNA it carries. Unlike teratogens
they may affect at any stage of life.
D) T
E) F Rh- negative mothers.
02)
A) T
Mature placental barrier consists only of
1. Syncytium
2. Endothelium
B) F
C) T
D) F
E) F
SBR
03) ANSWER – E
Sensory epithelium of ear, nose and eye is derived from the ectoderm.
04) ANSWER – B
Cystic fibrosis – autosomal recessive
Sickle cell anemia – autosomal recessive
Marfan syndrome - autosomal dominant
Homocystein urea – autosomal recessive
Thalassemia –autosomal recessive
06)ANSWER - D
SEQ-PROPER
2.0
.2.1.
2.1.1. Define the term “Monosomy” in chromosomal abnormality (05 marks)
Presence of only one member of a chromosome pair
.(ex-turners’ syndrome)
2.2.
An affected son was born to a normal male & a heterozygous female. The affected trait was linked to the sex
chromosome.
2.2.1.
State the Mendelian pattern of the above condition. (10 marks)
X linked dominant inheritance
2.2.2.
Describe the features of the above inheritance pattern by giving 3 examples. (30 marks)
their daughters.
Presented by 15th Batch 315 FHCS | EUSL
rome and
incontinentia pigment.
Affected heterozygous female and normal male,
4.0.
4.1. Outline the process of spermatogenesis
(30 marks)
erty.
• Both teratogens and mutagens cause alteration in the structure an functioning of the body, but the mechanisms
differ.
• Teratogens cause damage by altering the embryonic/ fetal development directly.
• Mutagens in contrast cause changes within the genetic material, restriction of growth or death of the embryo
/fetus.
➢It may also lead to inherited diseases if the germ cells are affected
➢Lead to cancer if somatic cells are involved.
SEQ-REPEAT
3.
3.1 Define the embryonic period (5)
3..2 list 2 major events that take place immediately after implantation
(5)
entiate
Blastodisk development
3.3. Describe the process of one of the event stated for Q 3.2
In the area over the embryoblast, the trophoblast has differentiated into two layers:
(1) an inner layer of mononucleated cells, the cytotrophoblast, and
(2) an outer multinucleated zone without distinct cell boundaries, the syncytiotrophoblast.
Mitotic figures are found in the cytotrophoblast but not in the syncytiotrophoblast.
Thus, cells in the cytotrophoblast divide and migrate into the syncytiotrophoblast, where they fuse and lose their
individual cell membranes.
The trophoblast shows considerable progress in development, particularly at the embryonic pole, where vacuoles
appear in the syncytium.
Presented by 15th Batch 318 FHCS | EUSL
When these vacuoles fuse, they form largelacunae, and this phase of trophoblast development is thus known as
the lacunar stage.
4.0.
4.1. List 5 prenatal diagnostic techniques that are performed
1. Amniocentesis
2. Chorionic villi sampling
3. Cordocentesis
4. Fetoscopy
5. Ultrasonography
4.2. Write an account of a non-invasive prenatal diagnostic technique which is used frequently
• Ultrasonography
Ultrasonography is a relatively noninvasive technique that uses high-frequency sound waves reflected from tissues
to create images.
The approach may be transabdominal or transvaginal, with the latter producing images with higher resolution .
In fact, the technique has advanced to a degree whereby detection of blood flow in major vessels, movement of
heart valves, and flow of fluid in the trachea and bronchi are possible. Important parameters revealed by
ultrasound include
characteristics of fetal age and growth;
presence or absence of congenital anomalies;
status of the uterine environment, including the amount of amniotic fluid ;
placental position and umbilical blood flow;
and whether multiple gestations are present .
All of these factors are then used to determine proper approaches for management of the pregnancy.
Fetal age and growth are assessed by crown-rump length during the 5th to the 10th weeks of gestation. After that,
combination of measurements—including the biparietal diameter (BPD) of the skull, femur length, and abdominal
circumference—are used .
Multiple measurements of these parameters over time improve the ability to determine the extent of fetal growth.
Congenital malformations that can be determined by ultrasound include the
neural tube defects anencephaly and spina bifida
abdominal wall defects, such as omphalocele and gastroschisis
heart and facial defects, including cleft lip and palate
Ultrasound can also be used to screen for Down syndrome and some other chromosome related abnormalities
through a test called nuchal translucency.
Presented by 15th Batch 321 FHCS | EUSL
The test is performed at 11 to 14 weeks of pregnancy.
Information from this test, combined with maternal serum screening test results and the mother’s age, can be
combined to provide a risk estimate for Down syndrome.
9TH BATCH
9TH PROPER MCQ
02) Day and appropriate events of the maturating embryo are correctly matched
A) Day 8 – differentiation of inner cell mass into 2 layers
B) Day 9 – formation of primitive yalk sac
C) Day 13 – formation of secondary villi
D) Day 11 – blastocyst completely embedded in the endometrial stroma
E) Day 16 – chorionic cavity
03) Oogenesis
A) Is started before birth
B) Primary oocyte may get mutation with age
C) Preovulatory phase is stimulated by LH
D) Granulosa cells contribute to produce zona pellucida
E) 2nd meiotic division finished immediately before ovulation
SBR
07) Fusion of male and female gemmates is called fertilization. What happened immediately after fertilization?
A) Formation of pronucleus
B) Restoration of diploid number of chromosomes
C) Second meiotic division of oocyte
D) Restriction reactions around zona pellucida
E) Degeneration of tail of spermatozoa
09) The structure of umbilical cord that is remaining active in late pregnancy
A) Allantois
B) Vitelline vessels
C) Yolk sac
D) Umbilical vessels
E) Vitelline duct
4.
4.1. A young woman who has missed two menstrual periods complains of intense abdominal pain. She was
suspected of having ectopic pregnancy & thereafter admitted to the hospital for further investigations.
4.1.1. Explain the term “Ectopic Pregnancy” & list 3 common sites where it can occur (15 marks)
4.1.2. Describe the sequence of events during fertilization (35 marks)
4.2.
4.2.1. Describe the formation of placenta (35 marks)
4.2.2. Draw & label the structure of fully developed placenta (25 marks)
Answers
MCQ PROPER
01)
Special features of autosomal dominant
- new mutation - Variable expressively - variation in age on set - reduce penetrance - variation in severity
dependent on sex
02)
A) T
B) T
C) F 3 rd week
D) T
E) F in 11th and 12th days
O3)
A) T
B) T
C) F By FSH
D) T
E) F Immediately after fertilization.
04)
A) T Langman’s pg. 69 – table 6.1
B) T
C) T
D) T
E) T
05)
A) T
B) T
C) F
D) T
E) F
06)
A) T
B) T By forward growth from primitive streak
C) T In 3rd week of development
D) F Not caudal its cranial end
E) T Fibroblast growth factor 8
07)
ANSWER- B
3 main results of fertilization;
1. Restoration of the diploid number of chromosomes.
2. Determination of the sex of the new individual.
3. Initiation of cleavage.
08)
ANSWER – B Biconcave shaped
09)
ANSWER-D
1. Uterine tubes
2. Ovary
3. Abdominal cavity
4.1.2)
tes
fuse, occurs in the ampullary region of the uterine tube.
1. Capacitation
-Period of conditioning
-Takes approximately 7 hours.
-Removal of glycoprotein & seminal plasma membrane that overlies acrosomal region of spermatozoa.
2. Acrosome reaction
-Occurs after binding to the zonapellucida.
-Release enzymes
which are needed to penetration of egg.
,
1. Restoration of the diploid number of chromosomes.
2. Determination of sex of new individual
(“X” carrying sperms produce female baby & “Y” carrying sperms produce male baby)
3. Initiation of cleavage.
4.2)
4.2.1)
acilitates nutrient & gases exchange between the maternal & fetal compartments.
uteroplacental circulation.
the endometrium.
ow resistance vessels.
thin & large pieces containing several nuclei breakdown & drop into intervillous blood
lake.
um & desidua basalis which is over the chorionic frondosum together form the placenta
4.2.2
immediately prior to ovulation, follicles are quite swollen and are called
graffian follicles.
4.2
The process by which the male and female gametes unite to give rise to
zygote which occurs in the ampullary region of the fallopian tube.
Ampullary region is the widest part of the tube & is close to the ovary.
Only 1% of sperm deposited in the vagina enter the cervix sperm become
less motile once they reach the isthmus .At ovulation sperm again
4.3
Write a brief account on Down,s syndrome.
(30 marks)
Down syndrome is caused by an extra copy of chromosome 21 [trisomy 21]. Features of children with Down
syndrome include growth retardation; varying degrees of intellectual disability;
craniofacial abnormalities, including upward slanting eyes, epicanthal folds [extra skin folds at the medial corners
of the eyes], flat facies, and small ears;
cardiac defects;
Hypotonia
These individuals also have an increased chance of developing leukemia, infections, thyroid dysfunction, and
premature aging
Furthermore, an increased frequency and earlier onset of Alzheimer disease is observed among persons with Down
syndrome.
26) Oogenesis
A) Corona radiata formed from granulosa cells
B) Diplotene stage is most suitable to protect oocytes
C) Oocyte complete second meiotic division at fertilization
D) Primordial follicle formed during prenatal life
E) Primary oocyte is vulnerable to damage as they advance in age
SBR
58) What is the most appropriate statement for notochord?
A) It consist of ectodermal cells
B) It develops during 2nd week
C) It is established out tail region
D) It is the precursor of spinal cord
E) It serve as the axial skeleton
04.
4.1 Caudal dysgenesis (sirenomelia) is one of the major effect caused by teratogens during the 3rd week of
development.
4.1.1 List the sequence of events takes place during the 3rd week of development. (10 marks)
4.1.2 Describe the most characteristic event occurring at the beginning of the third week of gestation. (30
marks)
Presented by 15th Batch 332 FHCS | EUSL
4.2 Write an account on
4.2.1. Oogenesis (25 marks)
4.2.2. Turner’s syndrome (20 marks)
4.2.3. Sex determination (15 marks
Answers
MCQ-PROPER
26)
A) T Langman pg. 23 fig 2.19
- Granulosa cells surrounding the oocyte remain intact and from cumulus oophorus. -Corona radiate is the
innermost layer of cumulus Oophorus and is directly adjust to zona pellucida.
B) T
whether the diplotene stage is the most suitable phase to protect the oocyte against environmental influences is
unknown.
C) T
oocyte finishes its second meiotic division immediately after the entry of spermatozoon.
D) T
at birth – primary oocyte is formed.
E) T
27)
Abnormal karyotype are seen in chromosomal abnormalities.
Abnormal number or abnormal structure.
A) T deletion of short arm of chromosome 5 – structural abnormality
B) F X - linked recessive.
C) F X - linked recessive.
D) T 45, X monosomy
Numerical abnormalities
E) T 47, XYY trisomy
28)
A) T
Day 11 & 12
- Capillaries get congested and dilated to form sinusoids.
_ results in establishment of uteroplacental circulation.
B) T
Day 13
Extraemryonic coelom—chorionic cavity.
C) T
day 12
D) F
Presented by 15th Batch 333 FHCS | EUSL
third week.
E) F
third week.
29)
A) F
Somatomeres = organized segments of Paraxial Mesoderm.
B) F
Cells
C) F
Paraxial Mesodermal segments 02.
1. Somatomeres - Head region
2. Somites – Occipital region caudally.
D) T
Somites
Sclerotome cells
Vertebrae
E) T
30)
A) F
though it’s a barrier not an absolute barrier.it facilitate gas and
nutrient exchange between maternal and fetal compartments.
B) T
because the maternal blood in the intervillous space is separated
from fetal blood by a chorionic derivative, the placenta is called
hemochorial type.
C) T
approximately 30 min after delivery.
D) T
1 In the core of the villous.
2. Thickening of basement membranes in fetal capillaries.
3. Obliterative changes in small capillaries of the villi.
4. Deposition of fibrinoid on the surface of the villi in the
junctional zone.
- By these changes reduced exchange between two circulations.
E) F Full term placenta -
Weight - 500-600g
Diameter - 15-25cm
Thickness - 3cm
31)
A) F
7 th day
B) T
C) F
Diploid
D) F
All 3 layers from the epiblast.
E) T
Neural crest cells extend throughout the neural tube
Migrate laterally
Dorsal Root Ganglion.
Presented by 15th Batch 334 FHCS | EUSL
32)
A) T
X linked recessive -
B) T
C) F autosomal recessive
D) F deletion of short arm of chromosome 5
E) F autosomal recessive
33)
Klinefelter syndrome – 47, XXY trisomy
genitalia.
and
varicose ulcers.
A) T
B) F
C) F
large teeth seen in Jacob’s disease (XYY)
D) T
E) T
58)
ANSWER – E
It underlies the neural tube and serve as basis for axial skeleton.27
Repeat campaign 2K18 presented by 10th wms
59)
ANSWER – D
60)
ANSWER – C
A) F
contain 37 coting sequence.
B) F
UGA
C) T
D) F
only from mother.
E) F
low percentage of introns.
4.1.2.
ess that establishes all 3 germ layers (ectoderm,
mesoderm,endoderm) in the embryo.
epiblast.
4.2.
4.2.1.
Oogenesis
nia differentiate into mature oocytes.
immediately prior to ovulation, follicles are quite swollen and are called
graffian follicles.
4.2.2.
Turner syndrome
– 45X
Presented by 15th Batch 336 FHCS | EUSL
de chest
4.2.3.
Sex determination
ing a pair of sex
chromosomes.
includes a X chromosome.
22)oogenesis
A) is started before birth
B) primary oocyte may get mutation with age
C) preovulatory phase is stimulated by LH
D)granulosa cells contribute to produce Zona pellucida
Presented by 15th Batch 338 FHCS | EUSL
E) Second meiotic division finished immediately before ovulation
SBR
44. which of the following is not true regarding male infertility
A) undescendent testis
B) acrosomal defect
C) Sperm count is less than 20 million/ ml
D) Lesion of pelvic nerve
51. The structure of umbilical cord that is remaining active in late pregnancy
A) Allantois
B) Vitelline vessels
C) Yolk sac
D) Umbilical vessels
E) Vitelline duct
3.4 Explain the differences in meiosis during gametogenesis in male and female gonads. (25)
4. 4.1 Describe the stages in the process of fertilization. (35)
4.2 Briefly describe the composition of fetal membranes in monozygotic twins. (25)
4.3 Observe the diagram given below.
Answers
PROPER-MCQ
22 A True
B True
C true granulosa cells and the oocyte secretea layer of glycoproteins on the
surface of the oocyte, forming the zona pellucida
D false Pre ovulatory phase-follicular phase-stimulated by FSH
E False
23 A True
B faLSE Connects amniotic cavity and yolk sac cavity
C
D
E True
24 A
B Neural crest cells contribute to the craniofacial skeleton
C F
D T sclerotome that will differentiate into the vertebrae and ribs
E T Each myotome and dermatome retains its innervation from its
segment of origin, no matter where the cells migrate. Hence, each
somite forms its own sclerotome (the tendon cartilage and bone
component), its own myotome (providing the segmental muscle
component), and its own dermatome, which forms the dermis of the
back. Each myotome and dermatome also has its own segmental
nerve component.
25 A F
B T
C T
D
E F 500-600g
26 A False Reciprocal translocation of chromosome
B True Deletion in maternal chromosome 15
C True Deletion in small part of chromosome 22
D False Reciprocal translocation
E True Deletion in long arm of chromosome 15
28 A T
B T
C F
D T
E T
29 A T Important parameters revealed by ultrasound include
th
Presented by 15 Batch 341 FHCS | EUSL
characteristics of fetal age and growth;
presence or absence of congenital anomalies;
status of the uterine environment, including the amount of amniotic
fluid ;
placental position and umbilical blood flow;
and whether multiple gestations are present .
biparietal diameter (BPD) of the skull, femur length, and abdominal
circumference
B T Ultrasound can also be used to screen for Down syndrome and some
other chromosome related abnormalities through a test called nuchal
translucency.
C T
D T
E T
44 Answer
D
51
PROPER-SEQ
3.4
4.
In the question male affected, female normal. If it is x linked 2 female children must be
carriers and 2 male children normal. Since they are normal, cannot be x-linked
30. Spermiogenesis
a. Transformation of spermatids into spermatozoa
b. Include formation of acrosome
c. Include development of cytoplasm
d. 74 days required for produce mature spermatozoa from spermatogonia
e. 30 million sperms produced daily
SBR
47. Which of the following genetic disorder show reduce penetrance/ variable expressivity?
a. X- linked recessive
b. Autosomal recessive
c. Autosomal dominant
d. Y-linked dominant
e. X-linked dominant
22.Regarding spermatozoa
A. abnormal spermatozoa has lack normal motility
B. Capacitation occurs in female reproductive tract
C. acrosomal reaction helps to penetrate the zona pellucida
D. tail is left behind when it penetrate the ovum
E. spermatogenesis longer than oogenesis
2.1. An ultrasound of a pregnant mother at 7 months gestation shows excessive fluid accumulation in the amniotic
cavity.
2.1.1.State the clinical condition of the above and mention 2 causes for it.
2.1.2.Write an account on amniotic fluid.
2.2. Write accounts on
2.2.1.Autosomal recessive inheritance
2.2.2.Gastrulation
45. When primary oocyte finished its 1st meiotic cell division?
A) After fertilization
B) During ovulation
C) Just before ovulation
D) After ovulation
E) Just before fertilization
2. 2.1. Describe the formation of neural crest cells and list five (05) derivatives of those cells.
2.2. Write an account on monozygotic twins and the composition of their fetal membranes and placenta.
2.3. List four (04) clinical features of Turner syndrome
50) On ultra sound examination it was determined that a fetus had fell developed facial and thoracic regions, but
caudal structures were abnormal. Kidneys were absent, lumbar and sacral vertebrae were missing, and the hind
limbs were fused. What process may have been disturbed to cause such defects?
A. Formed of fate map
B. Fusion of lateral folding
C. Gastrulation
D. Neural tube formation
E. Notochord formation
51) Following these answers which genetical disorder shows genetic heterogeneity in the affected populations
A. AutosomalDominance
B. Autosomal Recessive
C. X linked Dominance
D. X lined Recessive
E. Y linked Dominance
Answers
SEQ
1.1
1.2
branes in respiratory tract.Mucous trap pathogenCan be sneezed coughed away By ciliary action.
Macrophages in respiratory epithelium phagocyte
Pharynx there is waldayers ring collection of lymph follicles.
MUCUS PROTECTS. Mucus moistens and warms inhaled air and keeps the mucus membrane cells and the little hairs
called "cilia" lubricated
1.3
▪ White blood cells enter the tissue spaces by diapedesis and move through tissue spaces by ameboid motion.
▪ Granules contain a variety of toxic substances that kill or inhibit growth of bacteria and fungi.
▪ Macrophages are the end-stage product of monocytes that enter the tissues from the blood.
▪ respiratory burst- causing the release of reactive oxygen species.
▪ lysosome contain digestive enzymes (proteolytic enzymes)
▪ macrophage produce chemokines- recruit other cells to the site of infection.[
▪ Neutrophils and macrophages – bactericidal agents/oxidizing agents
▪ basophils release histamine- important in the defence against parasites and play a role in allergic reactions, such
as asthma.
▪ Natural killer cells (NK cells) destroy compromised host cells, such as tumor cells or virus-infected cells
▪ Dendritic cells (DCs)-f antigen presentation, link between the innate and adaptive immune systems.
01) Blood,
a) Express serum when it clots.
b) From a higher percentage of body weight in obese than in thin people.
c) Has low blood volume per kg body weight in neonate than in adults.
d) Make up about 7% body weight.
e) Volume rise after drink water.
09) Which of the following cells are involved with pathogenesis of acute immune decrease,
a) B-Lymphocyte
b) Mast cell
c) Neutrophils
d) Natural killer cell
e) T-Lymphocyte
SBR
11) Catabolism of Hb involves,
a) De-oxidative cleavage of the porphyrin ring.
b) Is the sole source of bilirubin.
c) Occur in RBC.
d) Synthesis bile salt.
e) Albumin is not considered for this.
14) A 5 years old boy suffering from diarrhoea for about 2 week and recently becomes sever. From further
investigation it is observed that feces contain parasites. What’s the type of leukocyte that might increase?
a) Basophils
b) Monocyte
c) Neutrophil
d) Eosinophil
e) Lymphocyte
15) All the following cells are antigen presenting cell, Except
a) B – Lymphocytes
b) Dendritic cell
c) Cooper cells
d) Microglial cell
e) Neutrophil
1. A mother of six-month-old boy cannot stop his bleeding after he hits his mouth on the kitchen floor. His mother
said that the boy’s cousin had a similar bleeding problem. He was admitted to the hospital and the investigation
gave the following results.
-Hb-9.6g/dl,
-Platelets-352000/ml
Presented by 15th Batch 354 FHCS | EUSL
-Bleeding time 7.5 min. (Normal 1-9Min)
-PT-12 sec (10-12 sec)
-APTT> 120 sec (28-35 sec)
1.1 What are the 3 processes involved to stop bleeding. (30 marks)
1.2 What coagulation disorder is likely to be present? (20 marks)
1.3 Explain briefly how you come to the above diagnosis. (35 marks)
1.4 How will conform your diagnosis (15 marks)
2
2.1
2.1.1. Briefly describe the function of albumin. (25 marks)
2.1.2. Explain the formation of Bilirubin from Heme. (25 marks)
2.2
2.2.1. List five characteristic difference between innate and acquired immunity. (20 marks)
2.2.2. Explain briefly the immunopathogenesis of allergy resulting from type 1 hyposensitivity. (30 marks)
Answers
MCQ
1. A) T
B) F
C) F
D) T ( 6‰- 8%)
E) F
2. A) F
B) T
C) F
D) T
E) F
B) F while transfusion of blood, antegen doner and antibody of the recipient are considered
C) T
D) F
E) F
C) T
E) T
5. A) F
B) T
C) T
D) F ( Bi, BB)
E) T
6. A) T
B) T
C) F vasoconstriction by tromboxane A2
E)
7. A) T
C) T
D) F
B) T
C) T
D) T
E) F
9. A) T
B) F
C)
D)
E) T
10. A) T
C) F
12. Answer A
13.
14. D
15. E
11 th SEQ
1.
Vasoconstriction
• After injury the blood vessel constricts and decreases the loss of blood from damaged Portion.
• Arterioles constrict.
➢ Nervous reflexes
▪ Pain fibers reach the CNS (spinal cord)
May effected by
hereditary Hemophilia
• Hemophilia A
(low CF 8) or
Hemophilia B (low CF 9)
1.3
• APtt is high
• Which shows abnormal intrinsic pathway
• So the patient should have decreased production or action of Factors in intrinsic pathway
(Clotting factor 12, 11, 9 or 8)
• Which tend to be Hemophilia A (low CF 8) or Hemophilia B (low CF 9)
2.1.1.
• Act as a transporter of the plasma
▪ Albumin carries insoluble substance in the plasma ( Steroid and thyroid hormones, Unconjugated bilirubin,
Drugs, Enzymes)
• Maintain the osmotic pressure of the blood
▪ Albumin is the highest found plasma protein in blood
▪ Cannot pass through the capillary membrane easily and remain in the blood.
2.1.2.
During hemolysis in the reticular endothelial system (Spleen, Liver),
• Hemoglobin will be broken down
• Heme + globulin proteins result
• Heme breaks and iron + Porphyrin result
• Porphyrin will be broke down into Biliverdin
• Catalyst by Heme oxidase
• Biliverdin will be converted into Unconjugated bilirubin (UCB)
Conjugation
of the UCB,
•
Unconjugated
Bilirubin will
be conjugated in the hepatocytes
• Glucuronidyl- transferase catalyze the reaction
• Conjugated bilirubin (CB) is formed
UPTO THIS IS MORE IMPORTANT
In biliary system,
• CB is released to the bile canaliculi
• Which lead to bile duct
• Open into the Duodenum
In intestine,
• CB is converted into urobilinogen
• By bacterial action
• Some urobilinogen reach the entero-hepatic circulation
• Remaining will be converted into stercobilin( Brown)
• Eliminate in faces
In Kidney,
• Urobilinogen in the blood reach the kidney
2.2.2.
• Type 1 hypersensitivity is IgE mediated
✓ anaphylactic reactions
Anaphylaxis
• The allergen react with basophils and mast cells
• Basophils and mast cells have been sensitized by attachment of IgE antibody
• allergic reaction occurs throughout the vascular system and closely associated tissues
• This reaction is called anaphylaxis
Urticaria
• Antigen entering specific skin areas and causing localized anaphylactic reactions.
• Histamine released locally causes
✓ Vasodilation that induces an immediate red flare
✓ Increased local permeability of the capillaries that leads to swelling of the skin
• The swellings are commonly called hives.
.Hay Fever
Difficulty in breathing
3) Albumin,
a. 40% in normal blood.
b. Initially synthesize as preproalbumin
c. It is a conjugated protein
d. Single polypeptide chain
e. It is served as source of amino acids to protein synthesizes
5) Eosinophil,
a. It's present in high concentration in pus
b. Secrete proteolytic enzymes
c. Increase in infections
d. Involve in allergy reactions
e. Lifespan is 4-5 weeks
Select the most appropriate answer from each question & mark them inside the cages
11) What is the protein responsible for iron transportation?
a. Apo ferrin
b. Apo transferin
C. Ceruloplasmin
d. Ferritin
e. Hepatalmoglobulin
13) Kamala has classic hemophilia. What was the most probable one?
a. It was autosomal dominant
b. Father had hemophilia and mother was a carrier
c. Mother was a hemophilia carrier
d. One of new x chromosome lead for deficiency if factor viii
e. She is suffering from von willebrand
14) A person who don't like to eat vegetables because he thinks it is poisonous, clinically
diagnosed his MCV 122,WBC= 10000
a. Hemorrhage
b. Aplastic anemia
c. folic acid deficiency
d. hemolytic anemia
e. sickle cell anemia
1. 25 year old mother with a history of previous abortions delivered her first baby. Same Day, the baby developed
jaundice and admitted to baby care unit. The following Investigations were done ;
• Hb-6.4 g/di
• WBC-8500,
• platelet 290000
• Serum Bilirubin Total 23.5 mol/dl
• Direct Bilirubin total 3.5 mol/dl
• Mothers blood group B negative. Fathers blood group AB positive
1.1 What is the more likely condition of this baby? (15 marks)
1.2 Explain the pathophysiology of the above condition. (40 marks)
3. What are the possible phenotype/s and genotype/s of the baby's blood group? (20 marks)
4. What is/ are the blood groups that you will consider for the transfusion of baby's condition? (10 marks)
5. How will you prevent the above condition in the next pregnancy? (15 marks)
2.1
1. Explain the biochemical basis behind the "neonatal jaundice and phototherapy (35 marks)
2.Briefly explain "deficiency of alpha 1 antitrypsin causes emphysema' (15 marks)
2.2
3. List one clinical example in each type or hypersensitivity reaction.
4. List three clinical conditions causing secondary Immune deficiency.
5. State the difference between allograft and xenograft. (10 marks)
6. State the difference between primary and secondary immune response. (10 marks)
7. State the difference between active and passive immunity giving examples. (14 marks)
Answers
MCQ
1)
A-T Hemolysis will occur
B-T Liver spleen
C-F Globin will be reutilized
D- In children can accumulate bilirubin in brain because of immature blood brain barrier. In adults cannot
accumulate
E-T
2)
A-F Refer heam metabolism notes.
B-T
C-T
D-F
E-F
3)
A-T Refer plasma protein notes.
B-T
C-F
D-T
SEQ
01)
1.1)
➢ Hemolytic disease of the new born (Erythroblastosis fetalis)
➢ Because of very low Hb & normal WBC & platelets
Normal new born Hb level 14-22g/dL
➢ Serum bilirubin is also in a high level
1.2)
➢ Fathers blood group is Rh positive & it is dominant than Rh negative.
➢ Therefore, every child of this parents will get Rh+ blood group.
➢ Rh incompatibility arises when an Rh-mother carries an Rh+
fetus.
➢ Small amount of fetal blood leak in to maternal circulation at the time of delivery & some
mothers develop significant titers of anti-Rh agglutinins during the postpartum period.
➢ During the next pregnancy the mother’s agglutinins cross the placenta to the fetus.
➢ In addition, there are some cases of fetal-maternal hemorrhage during pregnancy &
sensitization can occur during the pregnancy
➢ In any case, when anti-Rh agglutinins cross the placenta to Rh+ fetus, it can cause hemolysis
& various form of hemolytic disease of new born.
➢ If the hemolysis in the fetus is severe the infant may die in utero or may develop anemia,
severe jaundice & edema (Hydrops fetalis)
➢ Hemolysis release free bilirubin in to the blood.
➢ Large quantities of bilirubin in the ECF, either direct or indirect cause jaundice, which is
yellowish discoloration of skin, sclera, mucous membrane & deep tissue due to the
deposition of the of bilirubin.
➢ Here, elevation of indirect bilirubin is due to the increase production of unconjugated
bilirubin and bilirubin conjugating system not yet mature.
1.3)
Possible phenotypes of baby -AB /A/B
Possible genotypes of baby -AB+ /BB+/ AO+/BO+
1.4) O-
1.5)
Neutralize the mother’s blood by administrating anti-Rh antibodies in the form of Rh
immune globulin (Rhogam) during the postpartum period.
02)
2.1)
2.1.1)
➢ The majority of the new born infants (60% of full-term & 80% of pre-term) show a rise in
unconjugated bilirubin (UCB) in the 1st postnatal week
A transient physiologic jaundice because the activity of the hepatic bilirubin UDP-
glucuronosyltransferase (Bilirubin UGT) is low at birth.
(It reaches adult level in about 4 weeks)
Presented by 15th Batch 368 FHCS | EUSL
➢ Elevated UCB, in excess of the binding capacity of albumin(20-25mg/dL), Can diffuse into the
basal ganglia, cause toxic encephalopathy (Kernicterus) & a pathologic jaundice
➢ Therefore, newborns with significantly elevated bilirubin levels are treated with blue
fluorescent light (phototherapy)
➢ This phototherapy converts bilirubin into more polar & therefore water-soluble isomers.
➢ These photo isomers can be excreted in to the bile without conjugation to glucuronic acid.
(because of solubility differences, only UCB crosses the BBB & only CB appears in urine)
2.1.2 )
➢ Normally, α1-AT protects the lung form injurious effects by binding with the proteases.
Eg- active elastase
➢ Neutrophil elastase -Produced by WBC to break down harmful bacteria
-potentially damaging to the lungs if exposed
➢ A particular methionine (358 residue) is involved in binding with the protease
Active elastase + α1-AT->Inactive elastase: α1-AT complex->No proteolysis of lungs & No tissue damage
α1-AT deficiency
• α1-AT trapped in liver
• causing liver damage
• smoking oxidizes the methionine (358 residue) & inactivate the protein α1-AT
α1-AT cannot bind to the protease active elastase
Proteolysis of lungs & tissue damage occurs
accelerating the development of emphysema
2.2
2.2.1)
➢ type 1 – Utricaria
➢ type 2 – Rh incompatibility
➢ type 3 – Post streptococcal glomerulonephritis
➢ type 4 – post primary tuberculosis
➢ type 5 – Myasthenia gravis
2.2.2)
➢ Leukemia
➢ Post splenectomy
➢ AIDS
2.2.3 )
➢ Allograft – organ transplanted from a donor to a recipient of the same species who is not
genetically identical
➢ Xenograft – organ transplanted from a donor to a recipient of the different species
2.2.4 )
➢ Primary immune response – occurs when the antigen comes in contact to the immune
system for the first time
➢ Secondary immune system – Occurs when the 2nd (3rd/4
2.2.5)
Ex- Ex-
Measles-infection Natural- through the breast milk
Colostrum, Ig A
Artificial – injection of anti-rabies serum
3)thromboplastin is secreted by
A.
B. Blood vessels and surrounding tissues
C. Liver
D. Mass cells
E. Platelets
6) hematocrit
A. Is increased in people live in high altitude
B. Is measuring using Westergren's tube
C. Is used in the calculation of MCH
D. Refers to total number of red cells in 100ml of blood
E. Refers to volume percentage or cells In 10Uml or blood
7)Hemolytic anemia
A. Is accompanied by dark colored urine
B. Accompanied with increased number of reticulocytes
C. Accompanied with jaundice
D. Is associated with hereditary spherocytosis
E. Is characterized by high MC
8) Platelets
A. Destroyed in spleen
B. Produced in megakaryocytes of red bone marrow
C. Life span is about 120 days
D. Have bilobed nucleus none
E. Deficiency results in bleeding disorders
Single Best
14) What is the following plasma proteins produce outside the liver
A. Albumin
B. Gamma globulin plasma cells
C. Fibrinogen
D. B globulin
E. prothrombin
Answers
MCQ
1)
2)
A-T After 20 yrs (RBC are produced from membranous bones like vertebrae, sternum, ribs, scapula, iliac bone skull
bones & end of the long bones).
B - T Upto 20 years old red bone marrow of all bones [ long bones & all flat bones] (sembulingam pg 68).
C- F Site of erythropoiesis in fetal life.
D- F Shaft consists in yellow bone marrow
E - F Site of erythropoiesis in fetal life
1. Mesoblastic stage - Yolk sac(1st 2 months)
2. Hepatic stage - Liver (from 3 month)
3. Myeloid stage - Spleen
3)
A-
The thromboplastin is secreted by platelets that
catalyze the reaction which converts prothrombin
in to the thrombin. When tissue injury release thromboplastin [tissue
thromboplastin (factor iii)]
B- T
C - F
D - F
E-T
4)
B-F
C-T
D-T
E-T
5)
6)
C- F [ MCHC, MCV]
MCV= PCV in 100ml * 10/RBC count in million /cumm
MCH= Hb in gram per 1000ml of blood/RBC count in million /cumm
MCHC= Hb in grams/ 100ml of blood *100/ PCV in 100ml of blood
D- T
E- F
7)
08)
A True Platelets are destroyed by tissue macrophages system in spleen
Increasing destruction of platelets-> Splenomegaly->Decrease number of platelets in circulation
B True
C True 10-12 days
D True No nucleus
E True
09)
A True
B False MCHC 30 -38%
C False Antigens present.
D True
E False Osmolarity RBC=NaCl
10)
A False
B True
11)
B
Platelets are derived from megakaryocytes
Megakaryocytes not in peripheral blood
Normally, human blood contains 4000-11000
WBC per microliter, of these, the granulocytes
(Polymorphonuclear leucocytes, PMNs) are
the numerous. [ganong 554pg]
12) E
Vascular spasm-> Platelet plug formation - > Blood coagulation
13) D
A lots of clotting factors are produced in liver
So some liver disease cause to decrease of
production of clotting factor
So clotting & bleeding time will be increased
14) B
B lymphocytes/ plasma cells
15) D
Acute phase protein
Non specific changes in certain plasma
proteins in response to acute inflammatory
stages or secondary to certain chronic tissue
damages.
Positive acute phase protein - C reactive protein, Ceruloplasmin, alpha 1 antitrypsin, alpha 2 macroglobulin
Negative acute phase protein - Albumin, prealbumin, retinol binding protein, transferrin
SEQ
01) 1.1)
Rh incompatibility occurs when the mother’s blood type is Rh negative and her fetus blood type is
Rh positive.
If some of the fetus blood passes into the mother’s blood stream, her body will produce antibodies
against Rh factor in response.
Presented by 15th Batch 375 FHCS | EUSL
This is known as iso-immunization. Mother’s immune system becomes sensitized for Rh positive
antigens.
If these antibodies enter to a fetus via placenta it can harm fetus red blood cells and cause
hemolysis. (Hemolytic disease of neonates/HDN)
Factors that determine HDN are
o Genetic makeup of the mother and fetus
o Trans placental hemorrhage
o Can occur at any stage of pregnancy
o Injury, abortion, c section, placental removal, ectopic pregnancy, amniocentesis,
chorionic villus sampling, umbilical blood sampling, external cephalic version may also
leads to iso-immunization of Rh negative mother.
o Fetomaternal hemorrhage
o Volume
o Degree of maternal immune response (antigenicity of fetal RBC and type of IgG)
o Concurrent ABO incompatibility.
Mother’s iso immunization is the main risk factor for HDN. Normally first Rh positive baby escapes if
the delivery is normal and there were no any above complications during pregnancy period. Because
normally mixing of fetomaternal blood occurs during delivery when the placenta separates. But, subsequent Rh
positive babies may affect if the mother is previously sensitized. Anti Rh
antibodies fro, 1st pregnancy remain in maternal blood and cross the placenta during delivery and
cause hemolysis in Rh positive fetus.
2.1)
There are 2 main types of lymphocytes that play a major role in acquired immunity
T cells – in cellular immunity
B cells – in humoral immunity
• T cells
4 Types
a. Helper T cells
i. Helper-1
Secrete 2 substances
✓ Interleukin 1 – activates other T cells
✓ Gamma interferon – stimulates phagocytic activity of cytotoxic
cells, macrophages and natural killer cells
ii. Helper-2
Secrete
✓ IL-4 & IL-5 – Activates B cells
Proliferation of plasma cells
Production of antibodies by plasma cells
b. Cytotoxic T cells
✓ Receptors of cytotoxic T cells bind the antigen tightly.
Enlarge and release cytotoxic substance. (lysosomal
enzymes)
Destroy invading organisms.
✓ Also destroy cancer cells, transplanted cells, body’s own
tissues which are affected by foreign bodies (virus)
c. Suppressor T cells
✓ Suppress the activity of killer T cells.
✓ Prevent the t cells from destroying the body’s own tissue
along with invaded organisms.
✓ Suppress the activity of the helper T cells
d. Memory T cells
✓ When body exposed to some organism for 2nd time, memory
T cells identify them and immediately activate other T cells
and destroy them quickly.
• B cells
Proliferated B c ells transformed to
1. Plasma cells
✓ Produce the antibodies called immunoglobulin (IgG, IgA, IgM, IgD, IgE) and release in
to circulation. Produce antibodies until the end of the life span.
2. Memory B cells
✓ In the second exposure for an organism, memory cells are activated and produce
more antibodies.
2.3)
2.4)
SBR
11. Human blood in normal condition,
A) Eosinophil is the most common type of WBC
B) Lymphocytes are greater than neutrophils
C) There are more platelets than RBC
D) There are more WBC than RBC
E) Iron mostly in hemoglobin
15. Peyer's patches are aggregates of lymphatic nodules found in the area of the,
A) Small intestine
B) Appendix
C) Terminal ileum
D) Jejunum
E) Large intestine
01.
1.1. Name the two most important hormones involved in the regulation of blood volume. (20 Marks)
1.2. Briefly explain the mechanisms by which the above hormones restore blood volume following severe diarrhea
and vomiting. (80 Marks)
02.
2.1. A healthy young male living in Colombo, and having an RBC count of 4.92 million/ cm3. He went to spend his
holiday in Horton Plains (altitude - 2,300 m, 7,546 ft) in Nuwara Eliya District. At the end of his one month vacation
his RBC count was found to be 6.11 million/ cm3.
Explain the physiologic mechanisms leading to a significant increase in RBC count after one month stay in Horton
Plains. (50 Marks)
2.2. Briefly describe about sickle cell anemia. (35 Marks)
2.3. Draw and label the microscopic features of a lymph node. (15 Marks)
Answers
EME
MCQ
01 A- T theory
B- T theory
C- F capable of returning leaked protein from plasma.
D- F single layer of endothelium.
E- T theory
02 A- F adaptive immunity
B- T innate immunity
C- T innate immunity
D- T innate immunity
E- F adaptive immunity
03 A- T
B- T
C- T functioning as antigen presenting cells.
D- T
E- F lifespan is uncertain check wheatear’s histology
04 A- F
B- F 8%
Presented by 15th Batch 381 FHCS | EUSL
C- F
D- T
E- T
05 A- T Serum contain all the components as same in plasma except clotting factors
B- F
C- T
D- T
E- F
06 A- F 25.4mmHg
B- T theory
C- T
D- F isotonic loss so not much change in plasma osmolarity.
E- F their contribution is normally less than 2mOsmoles.
07 A- F not all factors ii,vii,ix,x
B- F not all EX: calcium ion 4th factor.
C- F some are produced by tissues during damage and platelets.
D- T
E- T refer the clotting cascade pathway
08 A- T theory
B- F more than 85%.
C- F some are found in some specific organs as well. HLA antigen
D- F after birth produced in response to gut bacteria. Refer Ganong’s physiology
E- F no blood group is safe universal transfusion. proper matching transfusion is
always safety suggestive.
09 A- T theory
B- T theory
C- F it induces rather than decreases.
D- F occurs in mitochondria
E- T theory
10 A- T Substances transported by albumin - Hormones( steroids & thyroid), Fatty
acids, Bile salts, Bile acid, Bilirubin, Ca2+, Mg2+, Cu, Zn, Drugs, plasma
tryptophan
B- T
C- T
D- T
E- T
11 E could be the answer
12 E could be the answer.
13 A is the answer.
14 D is the answer. Reduced production of ceruloplasmin which is important for copper
transport. Its due to the deficiency of ATPase enzyme. So, this causes accumulation of
copper in kidney liver and brain due to the transporter deficiency.
15 C is the answer. theory
SEQ
01)
1.1) ADH & Aldosterone
7) Lymphatic capillaries,
A. Are blind ended
B. Are present in bone marrow
C. Have anchoring filaments
D. Have overlapped endothelial cells
E. Permeability less than blood vessel
SBR
11) Largest contribution to blood viscosity is by?
A. Electrolytes
B. Plasma protein
12) The vitamin closely associated with synthesis of clotting factors is?
A. Vitamin A
B. Vitamin B12
C. Vitamin C
D. Vitamin E
E. Vitamin K
13) There are differences and similarities between Primary and secondary immune response. Which of the
following is similar to Primary and secondary immune responses?
A. Amount of Antibody formation
B. Occurs after antigen exposed
C. Predominant IgG
D. Predominant IgM
E. Time taken to Antibody formation
01.
1.1 Briefly describe the important changes that are seen during the transformation of a haemopoietic stem cell into
a mature Red Blood Cell. (15marks)
1.2 Name the factors essential for maturation of the stem cell into a normal red cell and their specific role(25marks)
1.3 Briefly describe the role of platelets in hemostasis. (10 marks)
1.4 Explain why damage to one eye (exposure of lens protein to immune system) leads to development of
cataract in the other eye. (20 marks)
1.5 Write a brief note on Thalassemia
Answers
1)
A. T
Presented by 15th Batch 386 FHCS | EUSL
Total body water -> Radioactive H2O, Deutarium, Antipyrine
ECF -> Radioactive Na, Radioactive Cl, Iothalamate
Plasma -> Evans blue dye
B. F
8%
C. T
D. T
E. T
RAAS
2)
A. F
Unconjugated bilirubin will increase
B. T
RBC will decrease
PCV will increase
C. F
Normocytic
D. T
To compensate reduced RBC, Reticulocytes enter into the blood.
E. T
3)
A. F
D antibodies only can cross
B. T
C. F
Antigens present in RBC membrane
Antibodies present in plasma
D. F
+
Rh dominant
E. T
F.
4)
A. F
0.8 – 2 %
B. T
C. F
Low affinity to 2,3 BPG
D. T
Increase O2 affinity than Hb A1 / Hb A
E. T
Hb A,C -> Glycosylated Hb
5)
A. F
B. T
6)
A. T
B. T
Antigen presenting cells - B Lymphocytes
- Macrophage
- Dendritic cells
C. T
D. F
E. F
7)
A. T
B. F
Absent in avascular structures, Brain, Spinal cord, Spleenic pulp, Bone marrow
C. T
D. T
E. F
More permeable
8)
A. F
B. F
C. T
D. T
E. F
Primary -> Stem cells divided and become immunocompetent
Secondary -> Show immune response
9)
A. F
B. F
C. F
D. T
E. T
Normal Hb -> Hb A, Hb A2, Hb F, Hb A1C
Abnormal Hb -> Hb S, Methemoglobin
10)
A. T
B. T
Maintain Acid Base balance.
C. T
D.
11) D
12) E
13) B
14) D
Acute phase protein -> Positive -> C reactive protein, Ceruloplasmin, α1 Antitrypsin,
α2 Macroglobulin
-> Negative -> Albumin, Prealbumin, Transferrin, Retinol binding protein
15) B
SEQ
1)
1.1.
The process of transformation of RBC from proerythroblast to mature RBC is erythropoiesis. It takes 7 days.
Uncommited stem cell -> Committed stem cell -> Proerythroblast -> Early normoblast ->
Intermediate normoblast -> Late normoblast -> Mature RBC
Proerythroblast -> Synthesis of Hb starts.
Early normoblast-> Nucleoli disappears.
Intermediate -> Hb start to appears.
Late normoblast-> Nucleus disappears.
Reticulocyte -> Reticulum contain and cell enter to the capillaries from the site of production.
Mature RBC -> Reticular network disappears. And cell become biconcave.
Usually erythropoiesis occurs in bonemarrow in adults.
1.2.
Maturation factors – Vitamin B9, B12, Intrinsic factor
Vitamin B9 -> Syenthesis and maturation of nucleic acids.
-> Essential for maturation of RBC.
-> Absence of B9 - Decrease synthesis of DNA -> Decrease DNA replication -> Decrease cellular
replication -> Large cells and reduced no of cells -> Megaloblastic anemia.
Vitamin B12 -> Synthesis of DNA in RBC.
-> Deficiency – Decrease maturation of RBC -> Decrease DNA replication -> Decrease cellular
replication -> Macrocytic anemia
Intrinsic factors of castle -> Essential for absorption of Vitamin B12 .
-> Deficiency leads to poor absorption of Vitamin B12.
1.3.
Platelets produced in bonemarrow. Hemostasis means active sequence of responces that stop bleeding.
Hemostasis -> Vascular spasm
-> Platelet plug formation
-> Blood coagulation
Vascular spasm - Platelets release thromboxane A2. It helps to vasoconstriction.
Platelet plug formation - To form platelet plug, platelet adhesion, platelet activation & aggregation have to occur .
Platelet adhesion occurs when VWF connects exposed collagen to platelets. Then platelets release ADP,
Thromboxane A2. They help to activation of platelets by changing the shape of platelets and form pseudopodia.
1.5.
Thalassemia is a hereditary haemolytic disease. It’s an autosomal recessive disease caused by insufficient synthesis
of α / β chains in globulin protein. In thalassemia total absence of either chains α / β occur. It is a disorder formed
by mutation in the genes. They may include entire gene deletion/ substitution/ deletion of one to many nucleotides
in DNA.
2 types -> α Thalasemia – Insufficient production of α globulin chain.
-> β Thalasemia - Insufficient production of β globulin chain.
Lack of coordination in α & β chains -> Impaired Hb syenthesis -> Formation of insoluble aggregates of excess
chain -> Damages the RBC -> Reticuloendothelial cells detect those damaged RBC -> Destruction of damaged RBC ->
Hemolytic anemia
35) Lymphocytes
A) Absolute count increase in pyogenic infection
B) Has a lobulated nucleus
C) Has longest lifespan out of white cells
D) is smaller in size than basophils
E) Transformed to plasma cells before making antibodies
36) IgG
A) Antibody formed against Rh factor when sensitized
B) Monomer
C) Neutralize the toxins
D) Response is greater in primary immune response than secondary immune response
E) Involves in type ІІІ hypersensitivity reactions
38) Platelets
A) Derived from megakaryocytes
B) Activate and release thromboxane A2
C) Prevent ischemic heart disease
D) Reduced in thrombosthenia
E) Are activated and attach to collagen
SBR
41) IgG
A) Present in mucosal cell barrier
B) Crosses the placenta
Presented by 15th Batch 391 FHCS | EUSL
C) Action is high in primary exposure than secondary
D) Produced by activated T lymphocytes
E) Contributes for Rh incompatibility
54) Edema is a feature in lymphatic obstruction. What is the main disturbance to starling forces in lymphatic
edema?
A) Decrease hydrostatic pressure interstitial space
B) Increase capillary hydrostatic pressure at arteriolar end
C) Increase capillary hydrostatic pressure at venular end
D) Decrease oncotic pressure in interstitial space
E) Decrease plasma oncotic pressure
55) Interstitial fluid is different to plasma though both are belongs to ECF. What is the most significant feature seen
in plasma?
A) Contain more Na+ than interstitial fluid
B) Higher buffering capacity than interstitial fluid
C) Has more protein than interstitial fluid
D) Electrically more negative than interstitial fluid
E) More viscous than interstitial fluid
57) 15-year-old boy came with fever and swelling on his leg after Trauma) His total WBC is 20 × 109/L, with
Neutrophil-18 × 109/L, with Lymphocytes- 2 × 109/What is the diagnosis?
A) Parasitic infection
B) Pyogenic bacterial infection
C) Tetanus
D) Tuberculosis
E) Viral infection
58) Mother is having O- while the blood group of the child is A+. What is the blood group of the father?
A) A-
B) AB-
C) A+
D) B+
E) O+
59) 35-year-old female came with having Hb% of 9g/dL & hypochromic microcytic blood picture. Her iron binding
capacity is higher than normal. What is the most likely diagnosis?
A)
B) Iron deficiency anemia
C)
D) Sideroblastic anemia
E)
60) A 40-year-old female presented with yellowish discoloration of sclera with anemia. It was suspected that she is
having hemolytic anemia. What is the most suitable test for conform the diagnosis of hemolytic anemia?
A) Hb%
B)
C)
D)
E) Urinary urobilinogen
01)
1.1.State 3 main groups of plasma proteins (10 marks)
1.2.Describe the role of plasma protein in,
1.2.2.Blood coagulation
02) Explain why,
2.1.PCV higher than normal in cyanotic heart diseases (30 marks)
2.2.Rhin compatibity is not seen in baby born to 𝑅ℎ+ mother (40 marks)
2.3.Macrocytic anaemia seen in folic acid deficiency (30 marks)
01)Explain why ,
1.1.Polycythemia is a feature of people living in high altitude. (30 marks)
1.2.Low dose aspirin is given to treat ischemic heart disease (40 marks)
36) A)T Ig G can cross the placenta. As a result, the Rh incompatibility may occur in Rh positive
fetus in sensitised Rh negative mothers.
B)T
C) T Tetanus and diphtheria
D) F Primary immune response- IgM
Secondary immune response – IgG
E) T In type III hypersensitivity reactions IgG, IgA and IgM
38) A- T
B- T Thromboxane A2 – promotes platelet aggregation
C- F
D- F Thrombasthenia – defective function of platelets.
Thrombocytopenia – reduction in platelet count.
E- T Platelet plug formation
40) A-T
B- T
C- F Erythropoietin is synthesized by the peritubular capillaries of the kidney
D- T Due to hypoxic stimulation
E- T Due to hypoxic stimulation
SBR
41) Answer- B/E
54) Answer- E
In lymphatic obstruction, the filtered proteins do not enter the blood reducing the
the plasma oncotic pressure.
55) Answer- C
57) Answer - B
WBC – 4.11 × 109
Neutrophil – 5.4 × 109
Lymphocytes – 2.7 × 109
Neutrophil count increases in bacterial infection
58) Answer- C
59) Answer- B
60) Answer- E
01)
1.1• Albumin
• Globulin
• Fibrinogen
1.2
02)
2.1cyanotic heart disease refers to a group of many different heart defects that are present at birth
(congenital). They result in a low blood oxygen level.
So Count of RBC increase to increase the oxygen capacity as a result PCV increase.
PCV is the proportion of blood occupied by RBCs expressed in percentage.
20) Erythrocytes
A) Erythrocytes are formed in bone marrow
B) Lifetime is 120 days
C) Production is stimulated by erythropoietin
D) Is dumbell in shape
E) is red in color
33) T/F
A) Erythrocyte production is increased in high altitude
B) Reticulocytes are destructed by reticuloendothelial system
C) Haemopoisis in adult confined to red bone marrow
D) Average lifespan of erythrocyte in a normal human is 10 days
E) Conjugated bilirubin level is increased in hemolytic jaundice
34) IgG
A) Produced in primary hypersensitivity
B) Is a dimer
C) Cross placental barrier
D) Important in producing antibodies against tetanus virus
E) Most abundant
SBR
47) There is an increase in eosinophil count elevate a serum IgE. What is the most likely condition?
A) Insect bite
B) Bacterial infection
C) Fungi infection
D) Parasitic infection
E) Viral infection
01.
1.1 Define “Natural immunity”. (20 marks)
1.2 Briefly describe three different mechanisms of natural immunity. (30 marks)
1.3 Explain the Physiological principle involved in vaccination and the benefits of vaccination. (50 marks)
20)
A) T
B) T
C) T Erythropoietin is produced in the kidney
D) T In cross section
E) T
21)
Presented by 15th Batch 398 FHCS | EUSL
A) F
B) F Sympathetic cholinergic receptors
C) T
D) F Normal body temperature is 37C
E) F Cold pale skin
29)
A) T Acute phase proteins, A class of proteins whose plasma concentration increase (+ APPs) or decrease (-APPs) in
response to inflammation
B) T Reactive protein C by opsonin on microbes
C) F synthesized in liver
D) T APPs causes increased plasma viscosity, increased ESR
E) T C reactive protein, serum amyloid A, ferritin, Interleukins
30)
A) F In adult only red bone marrow, In embryo in liver, spleen, lymph nodes and bone marrow.
B) T Ganong pg. 640
C) T Liver and spleen
D) F 2 alpha chains and 2 beta chains.
E) F Oxygen saturation in venous blood = 70%, Oxygen saturation in arterial blood = 97%
31)
Packed cell volume = Hematocrit (Red cell volume fraction)
PCV= 45%/ hematocrit = 0.45
A) T High altitude ->polycythemia -> PCV increases
B) T PCV low in ->Cell destruction , Blood loss , All types of anemia
C) T PCV increase in Polycythemia, Dehydration
D) F PCV measure using winthrobe tube ESR can measure using Winthrobe tube / westergen tube
E) T MCHC= MCH/MCV= Hb%*100/pcv (Hemoglobin concentration in one RBC)
33)
A) T In high altitudes, hypoxic stimulation increases the erythropoietin Production in the kidneys.
B) T If the reticulocyte count is increased in blood they are destructed By the reticuloendothelial system.
C) T
D) F 120 days
E) F Unconjugated bilirubin level
34)
A) F Ig G
B) F Is a monomer
C) T
D) T Important in preventing tetanus and diphtheria as those diseases Are caused by toxins produced by bacteria.
E) T GAMED
47) D
54) ANSWER – D
Serum is Blood plasma without fibrinogen
55) ANSWER – B
56) ANSWER-C In chronic renal failure monocytes increase and neutrophils Decrease.In acute renal failure due to
sudden tissue necrosis neutrophil Count increase.
58) ANSWER-C Arterial O2 is 97.5% saturated due to the presence of Physiological shunts.
Answers
MCQ
20)
A) T
B) T
D) T In cross section
E) T
21)
A) F
C) T
29)
C) F synthesized in liver
Increasing in ESR
30)
31)
Blood loss
Dehydration
33)
A) T In high altitudes, hypoxic stimulation increases the erythropoietin production in the kidneys.
B) T If the reticulocyte count is increased in blood they are destructed by the reticuloendothelial system.
C) T
D) F 120 days
34)
A) F Ig G
B) F Is a monomer
C) T
D) T Important in preventing tetanus and diphtheria as those diseases are caused by toxins produced by bacteria.
E) T
47)
ANSWER- D
Presented by 15th Batch 401 FHCS | EUSL
55)
ANSWER – B
56)
ANSWER-C
In acute renal failure due to sudden tissue necrosis neutrophil count increase.
58)
ANSWER-C
SEQ (PROPER)
Question
ANSWERS
06)
6.1)
- Due to obstruction of bile duct bile cannot enter to the small intestine from Gall bladder
- Bile contains many substances including bile pigments & bile salts.
Process Of Hemolysis
Within the macrophages RBC breakdown into globin chain & heme ring
⬇️
⬇️
⬇️
⬇️
⬇️
⬇️
Taken up by liver hepatocytes
Within liver unconjugated bilirubin conjugated by glucuronyl transferase forming conjugated bilirubin
⬇️
⬇️
⬇️
⬇️
Remaining urobilinogen breakdown into urobilin & stercobilin. They excreted with feces.
⬇️
⬇️
- In this condition increase the plasma conjugated bilirubin due to obstruction of bile duct.
6.2)
- As mentioned above due to obstruction of bile duct conjugated bilirubin level in plasma increases.
- Bile pigments give the unique colour of feces. Due to biliary obstruction bile pigments cannot enter to small
intestine.
6.3)
- As mentioned in 6.1 biliary obstruction leads to prevent bile acid entry to small intestine.
SEQ (REPEAT)
01)
1.1)
Natural immunity is also called innate or , native immunity First line of defence system which blocks the
entry of microbes into the body.
Once microbe enters body – phagocytosis, NK cells, some proteins, cytokines mechanisms will be activated.
This immune system founds in all multi-cellular organisms including plants.
Always present; ready to recognize and eliminate microbes
Do not react against non-microbial substances
Less specific
Powerful early defence mechanism
Cross-talk with adaptive immunity
1.2)
Physical and chemical epithelial barriers are the very early defence system which prevent microbes from entering
into the body.
–Skin, GI, RS
Once microbes enter into the body some other natural immune mechanisms are activated.
Phagocytes have a major role in natural immune system
–Neutrophils –Monocytes/macrophages- NK cells- dendritic cells.
Complement system and cytokines are main mechanisms which are activated to response to pathogens.
1- Complement system
• Collection of circulating and membrane associated proteins
• Chemoattraction
• Lectin pathway
2- Cytokines
• Dendritic cells, macrophages and other cells secrete
1.3)
Vaccines work by stimulating a response from the immune system to a virus or bacterium. This creates a ‘memory’
in the immune system. This immune memory allows the body to ‘remember’ a specific virus or bacterium, so that it
can protect itself against this virus or bacterium and prevent disease that it causes.
Most vaccines contain a weakened or an inactivated (killed) form of a virus or bacterium, or a small part of the virus
or bacterium that cannot cause disease. This is called an antigen.
When a person gets a vaccine, their immune system recognises the antigen as foreign. This activates the immune
cells so that they kill the disease-causing virus or bacterium and make antibodies against it.
It also activates immune cells - called T-cells and B-cells - in the blood, in the bone marrow and throughout the
body.
Later, if the person comes into contact with the actual virus or bacterium, their immune system will remember it.
It can then produce the right antibodies and activate the right immune cells quickly, to kill the virus or bacterium.
This protects the person from the disease.
Different vaccines bring about different levels of protection. How long protection lasts also depends on the disease
it protects against. Some vaccines can only protect against a disease for a short period and may need booster doses;
for others, immunity can last a lifetime.
10TH BATCH
10TH BATCH (PROPER) – MCQ
Answers
MCQ
19)
They are formed from interstitial fluid collected via lymph capillaries.
B) F composition of lymph is similar to ECF except the presence of high concentration of protein in lymph.
C) F is extracellular
D) F
E) T functions of lymph,
B) T
E) F
36)
A) F jaundice = yellowish discoloration of skin, sclera and mucus membrane due to increase in bilirubin
concentration in blood more than 2mg/dl. Normal plasma concentration less than 0.5mg/dl.
No hemolysis. So No jaundice.
⬆️ Respiration - - >SOB
Macrocytic (MCV > 100) eg; - B12 and folic acid deficiency anemia.
37)
Normal MCHC
B) F
C) F
⬇️
38)
A) T
D) T Glucose binds to a β chain Valine residue. RBC have 120 days life-span. So it remains in blood longer.
E) T
39)
A) T
B) T
40)
A) F
B) T
C) F
D) T
E) F
55)
ANSWER – E
56)
ANSWER – A
B cells—humoral immunity
57)
ANSWER – A
SEQ (PROPER)
6.
6.1.
Polycythemia means the excess cells in blood. Due to this action more RBC produces than normal and
polycythemia occur (6 -7 million/ mm3)
6.2.
SEQ (REPEAT)
04)
4.1)
It is the process of formation of red blood cells. 2 million per second are produced in a healthy person. In adults it
occurs in the bone marrow. The pluripotent haemopoietin stem cells are divided to form committed stem cells.
Then into colony forming unit of erythrocytes. Erythropoietin normally acts on this level. CFUs form
proerythroblasts. They have no Hb but have nucleoli. Then convert to Early normoblasts( Basophil erythroblast)
Nucleolus disappear and Hb starts appearing. Then to Intermediate normoblasts.After multiplication, convert into
Late normoblast. Hb is increased. It takes2-7 days for pro erythroblasts to mature into late normoblast. There are 4
mitotic divisions in between forming 16 cells. Not all are good, so bad cells are destroyed. These form reticulocytes,
mature for 2-3 days in bone marrow and are released into blood stream by diapedesis. These form 1% of RBC.
There, after 2-3 days, convert into mature RBC.
4.2)
• Tissue oxygenation
• Erythropoietin
• Fe
11TH BATCH
11TH BATCH (PROPER) – MCQ
01) Blood,
a) Express serum when it clots.
b) From a higher percentage of body weight in obese than in thin people.
c) Has low blood volume per kg body weight in neonate than in adults.
d) Make up about 7% body weight.
e) Volume rise after drink water.
09) Which of the following cells are involved with pathogenesis of acute immune decrease,
a) B-Lymphocyte
SBR
11) Catabolism of Hb involves,
a) De-oxidative cleavage of the porphyrin ring.
b) Is the sole source of bilirubin.
c) Occur in RBC.
d) Synthesis bile salt.
e) Albumin is not considered for this.
14) A 5 years old boy suffering from diarrhoea for about 2 week and recently becomes sever. From further
investigation it is observed that feces contain parasites. What’s the type of leukocyte that might increase?
a) Basophils
b) Monocyte
c) Neutrophil
d) Eosinophil
e) Lymphocyte
15) All the following cells are antigen presenting cell, Except
a) B – Lymphocytes
b) Dendritic cell
c) Cooper cells
d) Microglial cell
e) Neutrophil
05.
5.1.
5.1.1. List two function of lymph. (10 Marks)
5.1.2. Describe the mechanisms that facilitate lymph flow in lymphatic vessels. (25 Marks)
5.1.3. Briefly explain why an infected wound in the leg may cause enlargement of lymph node in the groin.
(15 Marks)
5.2. Explain the physiological basis for numbness in the hand
5.2.1. after holding ice cube inside the palm for a few minutes. (25 Marks)
5.2.2. when BP cuff is kept inflated in the upper arm above systolic pressure for three minutes. (25 Marks)
06.
6.1. State two stages of haemostasis. (10 Marks)
6.2. Briefly describe each stage mentioned in 6.1. (40 Marks)
6.3. Explain why three is increased risk of intravascular thrombosis in
6.3.1. people who are immobilized for long periods. (25 Marks)
6.3.2. people with atherosclerotic disease. (25 Marks)
31. -
33. A-F
D-T VB12 bound with intrinsic factor produce by stomach and absorb in intestine
D- T
B-T Ig G
C-T Ig A
D-
E-T When anti Rh agglutinin cross the placenta to an Rh + fetus, hemolysis occurs, in
40.A-T Absent in avascular structures, brain, spinal cord, splenic pulp, bone marrow
B-F
C-
D-T
E-T
-c creative protein
-ceruloplasmin
-alpha 1 antitrypsin
-alpha 2 macroglobulin
-haptoglobin
-fibrinogen
-albumin
-transferrin
-transthyretin
5.1.
▪ Lymphatic vessels drain excess interstitial fluid from tissue spaces and
5.1.2.
▪ Most of the filtered fluid out of blood capillaries returns to them by reabsorption.
▪ Some excess filtered fluid drains into lymphatic capillaries and lymph is formed
▪ Proteins can move through the lymphatic capillaries into lymph ( Lymphatic capillary more permeable for the
protein)
▪ Lymphatic vessels return the lost proteins and plasma to the bloodstream.
▪ Respiratory pump.
o Lymph flow is also maintained by pressure changes that occur during inhalation
(breathing in).
o Lymph flows from the abdominal region toward the thoracic region
➢ when a lymphatic vessel distends, the smooth muscle in its Wall contracts
➢ Which helps move lymph from one segment of the vessel to the next.
➢ Pathogens enter in to the lymphatic Capillaries on the infected wound of the leg
➢ Pathogens reaches Superficial and Deep inguinal nodes( lymph nodes in groin)
5.2
5.2.1. • Cold receptors of the palm sense the change in temperature (Free nerve endings)
• Contraction of the smooth muscles in the wall of the Arterioles near the surface of
the palm
• Prolonged reduced blood flow lead to reduced O2 supply to the nerves of the
• Lead to numbness
5.2.2. Blood pressure is the force exerted by the blood on the walls of the vessels Systolic pressure is the
highest pressure exerted by the heart on the blood vessels
• Prevent the blood flow to the distal part of the upper limb
• Leads to numbness
▪ Vasoconstriction
• Secondary response
▪ Blood coagulation
6.2. Vasoconstriction
• After injury the blood vessel constricts and decreases the loss of blood from
damaged Portion.
• Arterioles constrict
➢ Nervous reflexes
• This factor acts as a bridge between a specific Glycoprotein present on the surface of platelet and Collagen fibrils.
• Platelets get adhered to the collagen of ruptured blood vessel • Secrete adenosine diphosphate (ADP) and
thromboxane A
• All these platelets aggregate together and form a loose temporary platelet plug
Temporary platelet plug will be converted into definite blood clot Contains following steps,
, • Deep veins of the leg carries the blood to heart against the gravity
6.3.2. • Due to the abnormal deposition of Cholesterol under the epithelium of the blood
vessels
• High velocity of blood may damage the wall of the blood vessels
• Blood clot will be formed and block the lumen of the blood vessels
12TH BATCH
6. T form of Hb
a. Deoxy form of Hb
b. Has high oxygen affinity form
c. Has low oxygen affinity form
d. H bonds and ionic bonds limit the movement of monomer
e. The binding of oxygen destabilizes some of the H bonds and ionic bonds particularly between alpha – beta dimers
44. Which of the following is responsible for the seasonal allergic reactions such as runny nose, nasal congestion
and wheezing
a. Monocytes
b. Neutrophils
c. Mast cells
d. Lymphocytes
e. Basophils
59. 25 years old women complains she has tiredness immediately when she walk on a staircase. Physician found
that she has heavy menstrual bleeding , pale conjunctiva, palpitation like signs. What is the most possible cause?
a. Jaundice
b. Shortness of breathing
c. B12 deficiency
d. Microcytic anemia
e. Thyrotoxicosis
1.
1.1. Explain spherocytosis
1.2. Explain Physiological basis of
1.2.1 Anaemia in spherocytosis
1.2.2 Enlarged spleen in spherocytosis ▪ Shortness of breath and dizziness
1.3. Role of alpha 1 – antitrypsin activity in preventing emphysema
1.4. Regulation of heme synthesis
Answers
6. A.T
C.T
D.T
E.T bind O2 move Fe , This move proximal his and its helix , moving helix alters alpha
beta interface
B.T
C. T
D.
E.
31.A.T
In children blood cells are actively produce in the marrow cavity of all the bones.by age 20 marrow in
the cavities of the long bones except for upper humerus and femur has been inactive
D. F red blood cells and most wbc and platelets produce in bone marrow T and B lymphocytes also
produce in the lymph nodes and spleen
34.A.T
B.F
C.T
D.T
35..A.T erythroblastosis fetalis means mother is sensitize for Rh factor at first Time rh+ baby delivery and in
second pregnancy preformed agglutinin cross placenta and leads to hemolysis
B.F first dosage receive 28weeks of gustation and again 72h after delivery
C.F
D.F. if hemolysis in the fetus is severe than infant may die in utero or may developed anemia jaundice
edema (hydrops fetalis) Ganong page 562
E. T
36.A.T
B.T
C.T ABO system antibodies developed shortly after birth but Rh Antibodies developed
E.F mismatch occur between donor blood cells and recipient plasma
37.A. T in hemoglobin iron atom present as fe2+ reduce formed when it get Oxidize to fe3+
B. T
38.A T normal plasma volume is about 5% of body weight gan0ng page 562
B.T
43.A
Special/Specific/Acquired
44.C
Mainly allergies are due to mast cells which releases allergic inducing substances.
49. C
The reticulocyte count rises when there is a lot of blood loss or in certain diseases in
which red blood cells are destroyed prematurely, such as hemolytic anemia.
53. D
54.A
59.D
microcytic anemia
60.A
Antigen present at the surface of RBC...but in O grp no antigen... present both antigen
1.1. Hereditary spherocytosis is an inherited blood disorder. It happens because of a problem with the RBC.In
Hereditary spherocytosis, the RBCs are very small and spherical rather than being biconcave discs. These red cells
are more fragile than biconcave Shaped RBC.
Presented by 15th Batch 427 FHCS | EUSL
1.2
1.2.1 anemia- Reduced Hb concentration in blood lower than the normal level, relative to the age ,sex,race
and physiological status (ex:pregnancy, altitude)
Different abnormalities of the RBCs, many of which are acquired through hereditary, make the cells fragile,so
they rupture easily as they go through the capillaries, especially through spleen. Even though the number of RBCs
formed may be norm1, or evenmuch greater than normalin some hemolyticdiseases, the life span of the
fragileRBCs short that cells are destroyed faster than they can be formed and serious anemia results. This is called
hemolytic anemia ( in this anemia life span ofa RBC is shortened, increase breakdown of RBC occur.)
1.3. Alpha 1 - anti trypsin - it is synthesized by liver and it is the principal protease. Inhibitor of human plasma. It is
considered as acute phase reactant.
It increase in
• inflammatory conditons
• trauma
• burns
• infarction
• malignancy
decrease in
• emphysema
• nephrotic syndrome
•hypoproteinemia
Emphysema- is a lung condition that causes shortness of breath.in people with Emphysema the air sacs in the lungs
( alveoli) are damaged.
Normaly Alpha - 1 anti trypsin coats lungs, protecting them from Neutrophil elastase.neutrophil elastase produced
by white blood cells to break down harmful bacteria. Potentially damaging to lungs if exposed.
If alpha 1 anti trypsin deficiency occur, lungs lack alpha-1 anti trypsin coating, leaving them open to damage by
Neutrophil elastase. It trapped in liver, causing liver damage. Neutrophil elastase uninhibited causing lung damage
13TH BATCH
13TH BATCH (PROPER) – MCQ
12. Serum is different from plasma. What is the main feature of serum?
A. it has higher osmolarity.
B. It is devoid of fibrinogen.
C. It has increased calcium concentration
D It has no antibodies.
E. It contains platelets.
31. 02 binding to Hb
A. Co2 reduces oxygen affinity to Hb.
B. deoxy Hb increases affinity tor protons
C. Effect low affinity to 02 at high pH
D. Hb tetramer binds 4 moles of 2-3BPG
E. affinity for H increases with the increased percentage of saturation
Single best
55. Albumin contribute most of the plasma colloid osmotic pressure. what is the most
appropriate reason for that...?
A. Albumin is the smallest particle of the colloid particles
B. Albumin Is the most numerous in the colloid particles
C. Albumin Can't filtrate through the capillary
D. Albumin is rapid turnover in Liver
E. Albumin Found in high concentration in plasma
56. Which of the following tissues has the Highest oxygen consumption under resting condition
(ml/100mg/min)
A. Brain
B. Cardiac muscle
C. Kidney
D. Liver
E. Skeletal muscle
58. Serum is different from plasma. What is the main feature of serum?
A. it has higher osmolarity.
B. It is devoid of fibrinogen.
C. It has Increased calcium concentration
D. It has no antibodies
60. The most common method or analyzing separate serum plasma protein IS
A. Biuret Test
B. Chromatography
C. Electrophoresis
D. salt precipitation
E. Spectrophotometer
3.1. Normal red blood cells placed in hypotonic saline do not hemolyze until the concentration is 0.45% (half
normal) whereas red blood cells in hereditary spherocytosis hemolyze when placed in 0.7% NaCl solution. Explain
the physiological
basis of
3.1.1 Normal BC not hemolyzing in solutions above 0.45% though hypotonic. (25 marks)
3.1.2 Increased fragility of spherocytes (25 marks)
Answers
12)
A. False
B. True
C. False
D. False
E. False
13)
A. True Drain by lymphatic capillaries
SEQ
3.1)
3.1.1)
• Water moves from lower tonicity to higher tonicity
• Rbc have higher tonicity than 0.9% Nacl
• So the water moves inside the cell accompanied by swelling of the cell
• Normal RBC is less fragile and it can tolerate the swelling until the surrounding fluid get 0.45% Nacl concentration
• After that the cell membrane will rupture
3.1.2)
• In spherocytosis RBC is already expanded, Surface/volume is decreased.
• When the RBC is placed in hypotonic solution, it will swell more.
• It will rupture in 0.7% Nacl concentration, because of increased fragility
04)
4.1)
After exercise --> Need more O2 to reduce lactic acid made during exercise (oxygen debt) --> In anaemia patients --
> O2 carrying ability of the blood is reduced-->Via chemoreceptors apneustic centre will be stimulated,
simultaneously pneumotaxic centre will be inhibited--> Causes increased inspiration and decreased expiration-->
Shortness of breath
4.2)
• RBC is the main component in maintaining viscosity of blood
• In anaemia --> RBC is reduced--> Viscosity of the blood will reduce--> Critical velocity of blood will decrease--> so
normal velocity of blood flow cause turbulent flow at the orifices of the heart--> Turbulent flow causes--> Murmurs
4.3)
In anaemia--> O2 transported by blood reduced-->O2 to organs reduce --> O2 for
brain also reduce ---> Metabolic rate in brain reduce--> Dizziness
4.4)
• Oxygenated haemoglobin gives the reddishness to the blood
• INJ anaemia O2 carrying capacity of blood reduced
• So the reddishness also reduced
• Veins in the superficial skin also contain reduced haemoglobin
• So the pale colour of the skin will prominent
12. Thymus,
A) Primary lymphoid organ.
B) Supplied by internal thoracic artery.
C) Receive afferents
D) No venous drainage.
E) Secrete Thymic hormones
20. Creatinine,
A) Level points out urinary function
B) Converted to creatine phosphate by creatine kinase
C) High molecular compound
D) Found in plant food
E) Depends on muscle mass in the body
SBR
47. Right lymphatic duct is formed by the union of,
A) Right Juglar, subclavian and bronchomediastinal
B) Right Juglar and subclavian
C) Right Juglar and bronchomediastinal
D) Right jugular left jugular
E) Right subclavian left subclavian
52. Which clotting factor deficiency will lead to the prolonged bleeding only in male?
A) Prothrombin
B) Tissue thromboplastin
C) Calcium
D) Antihemophilic factor
53. Follow the blood cells produce outside bone marrow in adults
A) RBC
B) Platelet
C) Lymphocyte
D) Eosinophil
E) Neutrophil
5.
5.1. Hypoalbuminemia leads to:
5.1.1. hyponatremia
5.1.2. hypovolemia
Explain the physiological basis of the abnormalities stated in 5.1.1 and 5.1.2. following hypoalbuminemia.
(2 x 25 marks)
6. For a Rh negative mother carrying Rh positive fetus the risk of Rh incompatibility is less than that for the second
Rh positive fetus.
6.1. Briefly describe the consequences of Rh incompatibility to the fetus and their physiological basis. (50 marks)
6.2. Explain why the second Rh positive fetus is at greater risk than the first Rh positive fetus. (50 marks)
Answers
C. F No afferent lymphatics
D. F Has corresponding veins & thymic vein drains into left brachiocephalic vein
E.T Secrete thymosin which is necessary for T cell development & production
19.A .F
Macrocytic anemia
– Low PCV, Hb
– High MCV
C .T
E.F
20.A.T Creatinine is a breakdown product of creatine phosphate from muscle and protein
metabolism
B .F
21.A .T
B .F
C. F
D. T
E. T
22.A .T
obstructive jaundice
24.A.F
B.T
C.F
D.F
E.T
reticulocytes Because the body is trying to compensate the red cell loss
infection
professional phagocytes
D. -
E.T Hyperosmotic IV has more solutes than plasma. So increase plasma solute
B .F
D.-
SBR
47.A
Neutrophil → 2 - 5 days
Basophil → 12 - 15 days
Presented by 15th Batch 443 FHCS | EUSL
Monocyte → 2 - 5 days
52.D Hemophilia A and B are fully expressed in males only. [ A is most common ]
factor [ AHF ]
53.C Bone marrow produces all of the red blood cells, 60–70 percent of the white cells
( granulocytes), and all of the platelets. Lymphatic tissues ( thymus, the spleen, and the
Major sites in human eody for heme biosynthesis are liver and erythroid cells.
Heme synthesis rate in liver shows variability according to the demand of heme protein in body. But in case of this
rate in erythroid cells it show a constant and the rate is matched to the rate of globulin synthesis.
But when the demand is low which means there is adequate amount of heme in the body.
At such conditons the rate o heme synthesis is decreased and this is known as the end product Inhibition.
In liver when heme level increases in body these heme inhibits the action of ALA synthase 1 by repression of ALA
synthase 1 and inhibition of transport ALA synthase from cytosol to mitochondria matrix. Also in liver certain drugs
such as phenobarbital induce the actvity of hepatic ALA synthase. Heme synthesis in erythroid cells are controlled
by availability intracellular ions and following enzymes.
1. Urophophyrinogen.
2. Frrochelatase.
4. Abnormal Excretion or degradation in clinical conditons like nephrotic syndrome, hemorrhage, burns
5. Congenital conditons.
5.
5.1.Hypoalbuminemia leads to
5.1.1 Hyponatremia
Hypoalbuminimia means reduce albumin level in blood. There are two types of starling forces. Hydrostatic
pressure Oncotic pressure. Hydrostatic pressure exerted by a fluid. Oncotic pressure is exerted ey proteins mainly
albumin. There are 4 forces act in both arterial and venous end of the capillary. They maintain the capillary
microcirculation. Hydrostatic pressure in the capillary Hydrostatic pressure in the interstitial space Oncotic pressure
in the capillary Oncotic pressure I the interstital space.There is a net fltration occurs in arterial end and net
reabsorption occurs in venous end. When albumin level is reduced in blood, plasma oncotic pressure is reduced in
both arterial and venous end. When oncotic pressure is reduced net fltration is increased in arterial end and net
reabsorption in reduced. So more fluid enters into interstital space from plasma. With this fluid sodium like ions
also enter into interstital space. It cause reduce sodium ion concentraton in blood. It is called hyponatremia.
As mentioned above when oncotic pressure is decreased net fltration is increased and net reabsorption is
decreased. More fluid enters into interstitial space from plasma. Fluid accumulates in interstitial space. It cause
reduce blood volume in blood vessels. It is called hypovolemia.
6.
6.1.Briefly describe the consequences of Rh incompatbility to the fetus and their physiological
basis.(50marks)
• Rh+ individuals never develop Rh antibodies in their body because they have D antigen.
• After this sensitization, if this Rh- individual is again exposed to Rh+ blood, transfusion reactions occur
•Rh incompatibility occurs when the mother’s blood type is Rh negative and her fetus blood type is Rh positive.
•In new born babies, Rh incompatbility is less commoner than ABO incompatbility
•If Rh antibodies mix with Rh+ fetus, hemolytic diseases occur in fetal blood.
RBCs.
- Now, many large and immature cells in proerythroblastic stage are released into
circulation.
1. Severe Anemia
Excessive hemolysis results in anemia and the infant dies when anemia becomes severe.
When this conditon becomes more severe, it may lead to intrauterine death of fetus
3. Kernicterus
Form of brain damage in infants caused ey severe jaundice. Increase unconjugated bilirubin
in plasma. The blood-brain barrier is not well developed in infants as in the adults. Bilirubin
6.2.Explain why the second Rh positve fetus is at greater risk than the frst Rh positve fetus.
•Rh+ individuals never develop Rh antibodies in their body because they have D antigen.
•When Rh- mother carries a Rh+ fetus as her first baby, Antigen D is present in fetal blood but not antibody D
Antigen D is not present in mother’s blood. Antibody D also not present in maternal blood because she does not
expose to Rh+ blood yet.
• At the delivery, due to the separation of placenta, some amount of Rh+ fetal blood mixes with Rh- maternal
blood [ sensitization of the maternal blood ]
• Within that time, the mixed fetal blood cells are dead.
•When the mother conceives for the second time and if the fetus happens to be Rh+ again, Rh antibody in the
maternal plasma crosses the placenta and goes to fetal circulation. Fetus is Rh+ and it has D antigens on its RBC
surface
. •Antibody D received from mother will react with D antigens and make clumps.
• So the second Rh+ fetus is at greater risk than the first Rh+ fetus.
15TH BATCH
15TH BATCH (PROPER) – MCQ
38) what in left side test correctly evaluate right side situation
A. Anemia - MCV
B. Fragility of RBC - Hematocrit
C. Hypochromic - MCHC
D. Leukocytosis - WBC count
E. Microcytosis - RBC count
39) What are the events that take place of primary stage of blood clotting?
A. Initiate vasoconstriction
B. Involve fibrin clot formation
C. Initiate platelet plug formation
D. Increase bleeding time
E. Initiate by releasing tissue factors
SBR
41) Responsible for antibody production
A. Basophil
B. Neutrophil
C. Eosinophil
D. Plasma cells
E. T lymphocyte
4.
4.1.
4.1.1. What is hyperbilirubinemia? (05 marks)
4.1.2. Differentiate between conjugated and unconjugated bilirubin. (10 marks)
4.1.3. Briefly describe how to determine direct and indirect bilirubin. (15 marks)
6. An elderly male underwent partial gastrectomy, later encountered with increasing fatigue and shortness of
breath on exertion. Full blood count test showed an RBC count of 2.8 x10/L, Hematocrit 42% and a hemoglobin
concentration of 8.4g /dl of blood.
6.1. Calculate his
6.1.1. Mean Corpuscular Volume (MCV) (15 marks)
6.1.2. Mean Corpuscular Hemoglobin (MCH) (15 marks)
6.2. Explain the physiological basis of the hematological findings of 6.1 (20 marks)
6.3. Explain the physiological basis of fatigue and shortness of breath (20 marks)
6.4. Briefly explain the immunological response against the primary and secondary exposure to pathogen (30marks)
Answers
MCQ
12) Acute intermittent porphyria
A. F
B. T
C. F
D. F
E. F
27) Mucosa associated lymphoid tissue found in
A. T
B. T
C. T
D. T
E. F
34) Features of hemolytic anemia
A. T
B. T
C. F
D. T
E. T
35) Normal circulating blood contain
A. F
B. T
C. T
D. T
E. F
36) Regarding plasma cells
SEQ
04)
4.1)
4.1.1)
Increased Bilirubin levels in the blood is called hyperbilirubenemia
4.1.2)
When the pathogen is exposed primarily it develops immunological responses. Those messages will be
saved in memory T cells and memory B cells.
Messages for cell mediated immunity is saved in memory T cells and messages for humoral immunity saved
in memory B cell.
So during secondary exposure this memory cells get activate and take fast immunological responses and
destroy pathogen quickly.